Download as pdf or txt
Download as pdf or txt
You are on page 1of 195

E hern1of40 .

VMark ® it

b
3 I
Omuaturhwwl
..

QUESHO" Id: 2503 Tutorial Lab Values Notes Calculator Reverse Color

A 48—yearrold woman is evaluated for a painless lump in the right breast, which was first noticed by the patient 3 weeks
ago. She has no prior medical problems and has had no mammograms Her aunt was diagnosed with breast cancer at
age 60. Physical examination reveals a 1 x 1.5 cm, firm, mobile mass in the upper outer quadrant of the right breast with no
I

skin or nipple changes. There are no enlarged axillary or cervical lymph nodes. Mammography shows a spiculated mass,
and fine—needle aspiration biopsy is positive for invasive ductal carcinoma. The patient undergoes right mastectomy and
axillary lymph node dissection. Tumor analysis is negative for estrogen and progesterone receptors but positive for human
.

O
H

epidermal growth factor receptor 2 (HER2) gene amplification. Adjuvant therapy containing trastuzumab is planned. Which
H
H

of the following is most appropriate prior to starting the medication in this patient?
M
.

H
DJ
H

O A. Baseline audiometIy [2%]


H
.

A
H
Ul

B . Bone density measurement [13%]


H
at
.

C . Bone marrow biopsy [2%]


H
\l
H

v0 D. Echocardiography [44%]
.

a
H
u:
N

E . Factor V Leiden testing [2%]


.

e
N
l-‘

F Pulmonary function testing [6%]


N
.

N
N
o;

G . Tuberculosis skin testing [29%]


N
.

A
N
Ul
N
or
.

om'fled Ill 44% 3 Seconds _ 01/042020 ’


N
\l
N

Barrett answer Answered correctly Time Spent E Last Updated


.

a
N
u:
u
.

Female Reproductive System 8. Breast Q


0
w
l-‘

Breast cancer Feedback End Block


w

v
.

N
E Item1of40 IVMark < D ® 5! g '

b
Omufluhwwl
..

QUESHO" Id: 2503 Previous Next Tutorial Lab Values Notes Calculator Reverse Color

Explanation
I

Trastuzumab is a monoclonal antibody often used in combination with adjuvant chemotherapy in patients with human
epidermal growth receptor 2 (HER2)—positive breast carcinoma. Cardiotoxicity is a known adverse effect of trastuzumab
that usually manifests as an asymptomatic decline in left ventricular ejection fraction; however, overt heart failure can occur.
The risk and extent of cardiotoxicity are amplified when trastuzumab is used in combination with chemotherapy agents that
.

O
H

are also cardiotoxic (eg, doxorubicin).


H
H
M
.

Prior to initiating trastuzumab, patients should undergo a baseline assessment of cardiac function by
DJ
H

echocardiography. In addition, cardiac function should be reassessed by echocardiography at regular intervals during
H
.

therapy. Therapy should be discontinued in patients who develop symptomatic heart failure or a significant decline in
H
Ul
H

ejection fraction (eg, :16 percentage points from baseline). The cardiotoxicity associated with trastuzumab is typically
at
.

reversible, with patients experiencing complete recovery of cardiac function following treatment discontinuation.
\l
H
.

(Choice A) Ototoxicity is a common side effect of platinum—based chemotherapy (eg, cisplatin, carboplatin). Therefore,
H
u:

baseline audiometry should be performed prior to initiation of these agents.


N
.

e
N
l-‘

(Choice B) Aromatase inhibitors (eg, anastrozole, letrozole) are used in the treatment of postmenopausal estrogen
N
.

receptor—positive breast cancer and increase the risk of osteoporosis. Baseline bone density scans should be obtained prior
N
o;

to treatment.
N
.

A
N

(Choice 8) Baseline bone marrow biopsies are not routinely obtained prior to the initiation of cancer therapy. However, a
Ul
N
at

biopsy to evaluate bone marrow suppression may be indicated during therapy with antimetabolite chemotherapeutic agents
.

N
\l

(eg, 5—fluorouracil, methotrexate).


N
.

(Choice E) Tamoxifen is used in the treatment of estrogen receptor—positive breast cancer and increases the risk of venous
N
u:
u
.

Female Reproductive System a Breast


0
w
l-‘

Breast cancer Feedback End Block


w
.

N
E Inern1of40 .Vmflk ® it
$3
D
Q I
Omuaturhwwl
..

QUESfiOfl Id: 2503 Tutorial Lab Values Notes Calculator Reverse Color Text Zoom

baseline audiometry should be performed prior to initiation of these agents.

(Choice B) Aromatase inhibitors (eg, anastrozole, letrozole) are used in the treatment of postmenopausal estrogen
receptor-positive breast cancer and increase the risk of osteoporosis. Baseline bone density scans should be obtained prior
I

to treatment.

(Choice C) Baseline bone marrow biopsies are not routinely obtained prior to the initiation of cancer therapy. However, a
biopsy to evaluate bone marrow suppression may be indicated during therapy with antimetabolite chemotherapeutic agents
.

O
H

(eg, 5-fluorouracil, methotrexate).


H
HM
.

(Choice E) Tamoxifen is used in the treatment of estrogen receptor—positive breast cancer and increases the risk of venous
H
DJ
H

thromboembolism. Patients with factorV Leiden mutation are at increased risk of venous thromboembolism, and testing for
H
.

the mutation is sometimes indicated prior to initiation of tamoxifen.


H
Ur
H

(Choice F) Bleomycin can cause pulmonary fibrosis; pulmonary function testing should be obtained at baseline and at
at
.

regular intervals in most patients receiving bleomycin chemotherapy.


\l
H
.

(Choice G) Tumor necrosis factor-alpha inhibitors are used in the treatment of rheumatologic disease and can cause
H
in

reactivation of latent tuberculosis. Prior to initiating therapy with these agents, tuberculosis skin testing is recommended.
N
.

e
N
l-‘

Educational objective:
N
.

Trastuzumab is used for the treatment of patients with human epidermal growth receptor 2-positive breast carcinoma and is
N
o;

associated with a risk of cardiotoxicity. Cardiac function should be assessed with echocardiography at baseline and at
N
.

regular intervals in patients treated with trastuzumab.


N
Ur
N
OI
.

References
\l
N
.

- Trastuzumab—associated cardiotoxicity.
N
u:
u
.

Female Reproductive System 8. Breast Q


0
w
l-‘

Breast cancer Feedback End Black


w

v
.

N
E Item 1 out) I V "m 6;) 5| 3‘ P €333

l
I
- 2 Question I Tulon'al Lab Values Notes Calulator Reverse Color Text Zoom
' 3 Migneredcocfit‘. '" ,2 i E L3E'idaiez‘

- 4 Exhibit Display
. 5
. 6 Chemotherapy-induced cardiotoxicity

' 7 - Associated with anthracyclines


- 8 - Myoofie necrosis & destruction (fibrosis)
. 9 Type I - Progression to overt clinical heart failure
_ 10 - Less likely to be reversible
- 11 - Associated with trastuzumab
_ 12 Myowrdial stunninglhibemation without
. 13 myocyte destruction
Asymptomatic lefl ventricular systolic
' 1“ dysfunction
- 15 More likely to be reversible
. 16
. 17
. 18
. 19
. 20
. 21
. 22
. 23
. 24
. 25
. 26
. 27
. 28
_ 29 95' Add To Flash Card
. 30
. 31 Female Reproductive System 8. Breast [% D O
v Breast cancer Feedback End Block
N
U
I
E Inem2of4o .VMark ® it 3

y
I
Oflflfltmwv—l
u.-

QUESHO" Id: 13353 Tutorial Lab Values Notes Calculator Reverse Color

A36—yearrold woman comes to the office for a routine annual visit. The patient feels well and has no health concerns. She
has regular monthly menses with mild breast tenderness a day before bleeding starts. The patient had a copper«containing
I

intrauterine device (IUD) placed 2 years ago and uses condoms consistently. She is in a monogamous relationship with a
male partner, and they are getting married next year. Pap testing was normal 2 years ago. The patient has smoked 1 pack
of cigarettes daily for the past 20 years, but she does not use alcohol or illicit drugs. Blood pressure is 120l78 mm Hg, and
BMI is 24 kg/mz. Clinical breast examination shows no palpable masses or skin changes. The abdomen is nontender with
.

O
H

no palpable masses or hernias. Pelvic examination shows 2—cm IUD strings visible at the cervix and scant physiologic
H
H

vaginal discharge. The remainder of the physical examination is unremarkable Which of the following is recommended for
M
.

this patient at this visit?


DJ
H
H
.

O A. Abdominal ultrasound [4%]


H
Ul
H
at
.

O B. Electrocardiography [2%]
H
\l
H

9/0 C. Intimate partner violence screening [57%]


.

a
H
u:

O D. Low—dose helical chest CT [3%]


N
.

e
N

O E. Mammography [9%]
l-‘
N
.

O F. Pap test with human papillomavirus cotesting [22%]


N
o;
N
.

A
N
Ul
N
at
.

om'fled Ill 57% 1 Second _ 02/120020 '


N
\l

(tinned MSW” Answered correctly TIme SDGM E L351 Updated


N
.

a
N
u:
u
.

Female Reproductive System 8. Breast Q


0
w
l-‘

_ Intimate partner violence Feedback End Block


w
.

N
= them 2 of 40
<ir> 62) it 3 '
$3
}
Oflflfltmwv—i
IVMark
u.-

Question Id: 18368 Previous Next Tutorial Lab Values Notes Calculate r Reverse Color Text Zoom

Explanation
I

Intimate partner violence


Routine annual examination
Evaluation Suspicious signslsymptoms (eg, bruising)
.

O
H

Prenatal visits
H
H
M
.

Homicide
H
DJ

Mental health disorders (eg, PTSD)


H
H
.

Consequences Unintended pregnancy


H
Ur

Pregnancy complications (eg, abruptio placentae)


H
at
.

Sexually transmitted infections


H
\l
H
.

Safety planning (eg, local shelter referral)


a

Management
H
in

Psychosocial counseling
N
.

e
N

PTSD = posttraumatic stress disorder.


l-‘
N
.

Intimate partner violence (IPV) is any type of physical, psychologic, or sexual harm committed by a partner or spouse. It
N
o;

affects all genders, ages, races, and sexual orientations and is highly prevalent, with a lifetime risk of approximately 1 in 3
N
.

A
N

for women and 1 in 4 for men. IPV results in significant morbidity (eg, physical injury, mental health disorders) and mortality,
Ur
N

accounting for needy 15% of all homicides.


OI
.

N
\l

IPV particularly affects women of childbearing age; however, it is underreported because patients are often reluctant to
N
.

disclose IPV for multiple reasons, including denial, shame, self—blame, and fear of retribution. Because of the ovenNhelming
N
u:
u
.

Female Reproductive System a Breast


0
w
l-‘

Intimate partner violence Feedback End Black


w
.

N
E Item 2 M40 < D’ Q) ii g '

}
Oflflfltmwv—l
u.-

Question I“: ”353 Previous Next Tutorial Lab Values Notes Calculator Reverse Color
' : p08 rauma IC 5 r955 IISOlI 9|:

Intimate partner violence (IPV) is any type of physical, psychologic, or sexual harm committed by a partner or spouse. It
affects all genders, ages, races, and sexual orientations and is highly prevalent, with a lifetime risk of approximately 1 in 3
I

for women and 1 in 4 for men. IPV results in significant morbidity (eg, physical injury, mental health disorders) and mortality,
accounting for nearly 15% of all homicides

IPV particular1y affects women of childbearing age; however, it is underreporlred because patients are often reluctant to
.

O
H

disclose IPV for multiple reasons, including denial, shame, self—blame, and fear of retribution. Because of the ovenrvhelming
H
H

disparity between high prevalence and low disclosure rates, screening is performed in all women of childbearing age at
M
.

routine (eg, well—woman, prenatal) medical visits.


DJ
H
H

Screening commonly includes both open—ended (eg, "How safe do you feel in your relationship?") and specific (eg, "Have
.

A
H
Ur

you ever been hit, slapped, or kicked by your parlner?") questions to improve disclosure rates. Patients who screen
H
or
.

positive should be further assessed for immediate safety and given additional resources (eg, local shelter referral) for long-
H
\r

terrn planning.
H
.

a
H

(Choice A) Abdominal ultrasound is used to screen for abdominal aortic aneurysm (AAA) in men age 65—75 with a smoking
u:
N
.

history. Because screening does not reduce AAA—related mortality in women, this test is not indicated for this patient.
e
N
l-‘

(Choice B) Electrocardiography is indicated in high-risk patients (eg, hypertension, family history of premature cardiac
N
.

N
N

death at age <50), but routine screening is not recommended due to high false—positive rates and overtreatment,
u;
N
.

(Choice D) Low-dose helical chest CT is used to screen for lung cancer in patients age 55-80 with a Z30-pack-year history
N
Ur

who currently smoke or have quit within the last 15 years. This patient is age 36 with a 20—pack-year smoking history.
N
or
.

(Choice E) Mammography screening for breast cancer begins between ages 40 and 50 in average—risk women, depending
\r
N
.

on the guidelines used.


a
N
u:

Choice F In aatients ace 230 a Pan test with human ooaillomavirus cotestinc should be erformed eve 3-5 ears for
u
.

Female Reproductive System a Breast Q


O
w
l-‘

Intimate partner violence Feedback End Block


w
.

N
E imam2or4o Ivmm G») i( g I
$3
}
Oflflfltmwv—l
u.-

QUESHO" Id: 13353 Tutorial Lab Values Notes Calculator Reverse Color Text Zoom

term planning.

(Choice A) Abdominal ultrasound is used to screen for abdominal aortic aneurysm (AAA) in men age 65—75 with a smoking
history. Because screening does not reduce AAA—related mortality in women, this test is not indicated for this patient.
I

(Choice B) Electrocardiography is indicated in high—risk patients (eg, hypertension, family history of premature cardiac
death at age <50), but routine screening is not recommended due to high false-positive rates and overtreatment.

(Choice D) Low-dose helical chest CT is used to screen for lung cancer in patients age 55-80 with a 230—pack—year history
.

O
H

who currently smoke or have quit within the last 15 years. This patient is age 36 with a 20—pack—year smoking history.
H
H
M
.

(Choice E) Mammography screening for breast cancer begins between ages 40 and 50 in average-risk women, depending
DJ
H

on the guidelines used.


H
.

A
H
Ur

(Choice F) In patients age 230, a Pap test with human papillomavirus ootesting should be performed every 3—5 years for
H
at
.

cervical cancer screening. This patient's Pap test was normal 2 years ago.
H
\I
H

Educational objective:
.

a
H

Intimate partner violence has a high prevalence and significant morbidity and mortality and is underreported. Therefore,
in
N
.

screening is required in all women of child—bearing age at routine medical visits.


e
N
l-‘
N
.

References
N
o;
N
.

- Screening for intimate partner violence, elder abuse, and abuse of vulnerable adults: US Preventive Services Task
A
N
ur

Force final recommendation statement.


N
at
.

- Intimate partner violence discussions in the healthcare setting: a cross—sectional study.


\I
N
.

a
N
u:
u
.

Female Reproductive System 8. Breast Q


0
w
l-‘

_ Intimate partner violence Feedback End Block


w
.

N
E hern30f40 [Valiant ® it 3 I

}
INH
QUESHO" Id: 12150 Tutorial Lab Values Notes Calculator Reverse Color

A35—yearrold woman, gravida 2 para 2, comes to the emergency department due to diffuse abdominal pain. Six hours ago,
Oflflamh

the pain started suddenly in the right lower abdomen and is now diffuse. The patient had been playing tennis and says
there was no trauma She also notices pain in her right shoulder. Her last menstrual period was 3 weeks ago, and menses
occur every 28 days. A few months ago, the patient was hospitalized for a deep venous thrombosis; she had taken oral
contraceptives, which were discontinued immediately, and has since taken warfan'n daily. Both of her children were born
vaginally, and the patient has no history of surgeries. She does not use tobacco, alcohol, or illicit drugs. Temperature is
.

O
H

37.2 C (99 F), blood pressure is 80/40 mm Hg, and pulse is 118lmin. Physical examination shows a rigid abdomen with
H
H

diffuse tenderness, rebound, and guarding. Laboratory results are as follows:


M
.

H
DJ
H

Hematocrit 22%
H
.

A
H

Platelets 160,000/mm3
Ul
H
at
.

Leukocytes ElOCID/mm3
H
\l
H
.

Which of the following is the most likely diagnosis for this patient?
H
u:
N
.

O A. Acute appendicitis [7%]


N
l-‘
N
.

O B. Acute mesenten'c ischemia [6%]


N
o;
N

O C. Cholelithiasis [1%]
.

A
N
Ul

O D. Diverticulitis [0%]
N
at
.

O E. Ovarian torsion [7%]


\l
N
.

F. Ruptured ectopic pregnancy [34%]


N
u:
u
.

Female Reproductive System a Breast Q


0
w
l-‘

ovarian cyst Feedback End Block


w
.

N
it 3 $3
D
= turn 3 of 40 '

INH
Question Id: 12150 Calculate r Reverse Color Text Zoom

diffuse tenderness, rebound, and guarding. Laboratory results are as follows:


Oflflamh

Hematocrit 22%

Platelets 160,000/mm3

Leukocytes 9000imm3
.

O
H

Which of the following is the most likely diagnosis for this patient?
H
H

OA.
M
.

Acute appendicitis [7%]


DJ
H
H

Acute mesentenc ischemia [6%]


.

.Iomrnpom
OOOOOOO
H
U!

Cholelithiasis [1%]
H
at
.

Diverticulitis [0%]
\t
H
.

Ovarian torsion [7%]


H
u:
N
.

Ruptured ectopic pregnancy [34%]


N
l-‘
N

. Ruptured ovarian cyst [41%]


.

N
N
o;

Splenic mpture [1%]


N
.

A
N
U!
N
at
.

Omitted
\t

I" 41% 4 Seconds _ 02/053020


N

Correct answer
Answered correctly
.

Last Updated
a

Time Spent
G
N
u:
u

Ea
.

Female Reproductive System 8. Breast


0
w
l-‘

_ ovarian cyst Feedback End Block


w
.

N
il 3 $3
b
= Item 3 of 40
IVMark < D’ '

INH
Oflflamh Question Id: 12150 Previous Next Tutorial Lab Values Notes Calculator Reverse Color Text Zoom

Explanation

Acute abdominallpelvic pain in women

Diagnosis Clinical presentation Ultrasound findings


.

O
H

Recurrent mild & unilateral mid£ycle pain prior to ovulation


Mittelschmerz Not indicated
H
H

Pain lasts hours to days


M
.

H
DJ
H

Ectopic Amenormea, abdominal/pelvic pain & vaginal bleeding


H

No intrauterine pregnancy
.

pregnancy Positive B—hCG


H
U!
H

Sudden—onset, severe, unilateral lower abdominal pain; Enlarged ovary with


at
.

H
\l

Ovarian torsion nausea & vomiting decreased or absent blood


H
.

Unilateral, tender adnexal mass on examination flow


H
u:
N

Ruptured Sudden-onset, severe, unilateral lower abdominal pain


.

Pelvic free fluid


N

ovarian cyst immediately following strenuous or sexual activity


l-‘
N
.

Pelvic
N

Fever/chills, vaginal discharge, lower abdominal pain &


o;

inflammatory : Tuboovanan abscess


N
.

cervical motion tendemess


A

disease
N
U!
N
a.
.

This patient presents with hemoperitoneum due to a ruptured ovarian cyst. Corpus luteum cysts form in the second half
N
\l

of the menstrual cycle, after ovulation has occurred, Although hemoperitoneum does not typically occur with ovarian cyst
N
.

a
N

rupture, patients who are on anticoagulation can bleed intra—abdominally and become hemodynamically unstable Typical
u:
u

Ea
.

Female Reproductive System a Breast


0
w
l-‘

ovarian cyst Feedback End Block


w
.

N
E remaorrro -\>Mark ® 5]

D
g I

INH
QUESHO" Id: 12150 Tutorial Lab Values Notes Calculator Reverse Color

This patient presents with hemoperitoneum due to a ruptured ovarian cyst. Corpus luteum cysts form in the second half
Oflflamh

of the menstrual cycle, after ovulation has occurred. Although hemoperitoneum does not typically occur with ovarian cyst
rupture, patients who are on anticoagulation can bleed intra—abdominally and become hemodynamically unstable Typical
presentation of a ruptured cyst is sudden onset of unilateral lower abdominal pain. Symptoms of hemoperitoneum include
diffuse severe abdominal pain, pleun'tic chest pain, and shoulder pain (due to phrenic nerve irritation). Physical examination
with a ruptured ovarian cyst shows unilateral lower quadrant tenderness; with hemoperitoneum, diffuse abdominal rigidity
with rebound and guarding is present. Laboratory testing shows a decreased hematocrit due to intra—abdominal blood
.

O
H
H
H

loss. Pelvic ultrasound confirms intra-abdominal and pelvic free fluid and a possible adnexal mass (if the cyst is
M
.

incompletely drained). Treatment of a hemodynamically unstable patient with hemoperitoneum due to a ruptured ovarian
DJ
H

cyst is surgery to stop the bleeding.


H
.

A
H

(Choice A) Acute appendicitis causes right-sided lower abdominal pain with leukocytosis. Although physical examination
Ur
H
at

findings of an acute abdomen may be similar to those from hemoperitoneum, hematocrit does not decrease with
.

H
\r

appendicitis.
H
.

(Choice B) Acute mesenteric ischemia results from inadequate intestinal circulation (eg, thromboembolism). The resultant
H
u:
N

intestinal infarction classically presents as a relatively normal abdominal examination despite excruciating pain.
.

e
N
l-‘

(Choice C) Cholelithiasis typically presents with nausea, vomiting, and right upper quadrant pain. Examination findings are
N
.

focal right upper quadrant tenderness without a drop in hematocrit.


N
o;
N

(Choice D) Diverticulitis typically presents as left-sided lower abdominal pain; it does not lead to a diffuse acute abdomen
.

A
N
Ur

or a drop in hematocrit.
N
or
.

(Choice E) Ovarian torsion may present with sudden lower abdominal pain; it does not typically result in an acute abdomen
N
\r
N

on physical examination and does not cause a drop in hematocrit.


.

a
N
u:

(Choice F) Ruptured ectopic pregnancy can cause an acute abdomen due to hemoperitoneum, but it is extremely unlikely
u
.

Female Reproductive System a Breast Q


0
w
l-‘

_ ovarian cyst Feedback End Block


w
.

N
E Immaoftw Ivmm Q) i)

D
g I

INH
QUESHO" Id: 12150 Tutorial Lab Values Notes Calculator Reverse Color
findings of an acute abdomen may be similar to those from hemoperitoneum, hematocrit does not decrease with
Oflflamh

appendicitis.

(Choice B) Acute mesenteric ischemia results from inadequate intestinal circulation (eg, thromboembolism). The resultant
intestinal infarction classically presents as a relatively normal abdominal examination despite excruciating pain.

(Choice 6) Cholelithiasis typically presents with nausea, vomiting, and right upper quadrant pain. Examination findings are
focal right upper quadrant tenderness without a drop in hematocrit.
.

O
H

(Choice D) Diverticulitis typically presents as left—sided lower abdominal pain; it does not lead to a diffuse acute abdomen
H
H
M
.

or a drop in hematocrit.
H
DJ
H

(Choice E) Ovarian torsion may present with sudden lower abdominal pain; it does not typically result in an acute abdomen
H
.

A
H

on physical examination and does not cause a drop in hematocrit.


Ur
H
at
.

(Choice F) Ruptured ectopic pregnancy can cause an acute abdomen due to hemoperitoneum, but it is extremely unlikely
H
\l

with a last menstrual period just 3 weeks ago.


H
.

a
H

(Choice H) Splenic rupture may cause hemoperitoneum, but it typically presents as Iefl—sided upper abdominal pain.
in
N
.

Educational objective:
N
l-‘

A ruptured ovarian cyst may present with an acute abdomen due to hemoperitoneum. Hemodynamically unstable patients
N
.

N
N

require immediate surgical intervention.


o;
N
.

A
N
ur

References
N
at
.

- Successful conservative management of ruptured ovarian cysts with hemoperitoneum in healthy women.
\l
N
.

a
N
u:
u
.

Female Reproductive System 8. Breast Q


0
w
l-‘

_ Ovarian cyst Feedback End Block


w
.

N
E Mem4of40 IVMark ® 5] g
ۤ3
y
I
OflNeturIr—A QUESHO" Id: 12524 Tutorial Lab Values Notes Calculator Reverse Color Text Zoom

A 15—yearrold girl is brought to the office due to prolonged, irregular menstrual periods. Menses occur at 2— to 4—month
intervals and last for 7-10 days with heavy bleeding and passage of large clots. The patient's menstrual cycles are
nonpainful and she has no associated dizziness, fatigue, palpitations, or abnormal vaginal discharge. Menarche was at age
14 and her last menstrual period was a month ago. She has no other chronic medical conditions or previous surgeries.
The patient has never been sexually active. Blood pressure is 110/70 mm Hg and pulse is 72/min. Height and weight are
at the 40th and 56th percentiles, respectively. The thyroid is without nodules. The skin has no discoloration or ecchymosis.
.

O
H

Breast and pubic hair development are Tanner stage V. External pelvic examination shows no lacerations, lesions,
H
H

discharge, or bleeding. The hymenal membrane is intact. Hemoglobin is 10.4 gldL. PT and aPTI' are normal. Which of
M
.

the following is the best next step in management of this patient?


DJ
H
H
.

O A. Order pelvic MRI [5%]


H
Ur
H
at
.

O B. Perform cervical nucleic acid amplification testing [1 %]


H
\l
H

O C. Perform endometrial biopsy [2%]


.

a
H
in

O D. Perform hysteroscopy [5%]


N
.

e
N
l-‘

O E. Place copper—containing intrauterine device [1%]


N
.

V O F. Start oral progesterone therapy [84%]


N
o;
N
.

A
N
Ur
N
at

Omitted '
.

C m I" 84% 2 Seconds — 03/18/2020


N
\l

F0 answer Answered correctty Time Spent E L351 Updated


N
.

a
N
u:
u
.

Female Reproductive System 8. Breast Q


0
w
l-‘

_ Abnormal uterine bleeding Feedback End Block


w
.

N
_ hem40f40 3 '
$3
}
= Question Id: 1252‘ IVMark
OONGmNi—A
-.-.--.-
a
Tutorial Lab Values Notes Calculate r Reverse Color Text Zoom
Hl-l
HO

Hormone
levels
HH
LON
NHowwMtwNHewwMOm-h
UUUNNNNNNNNNNHHHHHH

Heavy
bleeding

Endomelrial
proliferation

Tlme
©UWOr1d

Female Reproductive Sys‘em 8. Breast


_ Abnormal uterine bleeding Ea Feedback
0
End Black
E Inem4or4o .VMark ® it Q 9
$3
}
OONGmNi—A Question Id: 12524 Tutorial Lab Values Notes Calculator Reverse Color Text Zoom

During the first year after menarche, adolescents often have anovulatory cycles with heavy, irregular menstrual
bleeding due to an immature hypothalamic—pituitary axis. In ovulatory cycles, the corpus luteum produces progesterone
after ovulation; progesterone differentiates the proliferatjve endometrium into secretory endometrium. As the corpus luteum
degenerates, the decrease in progesterone leads to normal menses through a decrease in endometrial blood supply and
shedding of the lining.

In contrast, anovulatory cycles do not produce progesterone (eg, no corpus luteum development) and there is no
.

O
H

differentiation into secretory endometrium. Therefore, these patients have continued unopposed estrogen stimulation
H
H
M
.

resulting in uncontrolled proliferation of the endometrium. Bleeding occurs when the endometrial lining becomes too
DJ
H

thickened and unstable (eg, breakthrough bleeding). Progesterone treatment helps stabilize this uncontrolled proliferation
H
.

due to estrogen by causing differentiation into secretory endometrium. Cyclic progesterone withdrawal then causes
H
Ul

menstruation.
H
at
.

H
\l

(Choice A) Congenital anomalies of the reproductive tract (eg, Mullerian agenesis) typically present with primary
H
.

amenorrhea rather than abnormal uterine bleeding. In patients with suspected anomalies, a pelvic MRI can be used to
H
u:

evaluate the structure of the reproductive tract.


N
.

e
N

(Choice B) Nucleic acid amplification testing is used in the evaluation of acute cervicitis, which presents with postcoital or
l-‘
N

interrnenstrual spotting rather than heavy menses. Acute oervicitis is typically caused by a sexually transmitted infection
.

N
N
o;

(eg, Neisseria gonorrhoeae, Chlamydia trachomatis) and is unlikely in a patient who is not sexually active.
N
.

(Choice C) An endometrial biopsy is an invasive procedure that can be used to diagnose endometrial hyperplasia, a
N
Ul
N

condition that is rare in adolescents. An endometrial biopsy may be indicated in an adolescent with risk factors for
at
.

hyperplasia (eg, obesity) or if symptoms do not improve with medical management.


\l
N
.

(Choice D) A hysteroscopy is a procedure that can be used to evaluate the endometrium for structural causes of abnormal
N
u:

bleedino e endometrial no! -, submucosal leiom oma and ma be indicated if unharmacothera fails.
u
.

Female Reproductive System 8. Breast Q


0
w
l-‘

Abnormal uterine bleeding Feedback End Block


w
.

N
E hem 4 of 40 ® 5) g I
$3
}
OflNeturIwNi—A QUESfiOfl Id: 12524 ' Tutorial Lab Values Notes Calculator Reverse Color Text Zoom

amenorrhea rather than abnormal uterine bleeding. In patients with suspected anomalies, a pelvic MRI can be used to
evaluate the structure of the reproductive tract.

(Choice B) Nucleic acid amplification testing is used in the evaluation of acute cervicitis, which presents with postcoital or
intennenstrual spotting rather than heavy menses. Acute cervicitis is typically caused by a sexually transmitted infection
(eg, Neisseria gonorrhoeae, Chlamydia trachomatis) and is unlikely in a patient who is not sexually active.

(Choice C) An endometrial biopsy is an invasive procedure that can be used to diagnose endometrial hyperplasia, a
.

O
H

condition that is rare in adolescents. An endometrial biopsy may be indicated in an adolescent with risk factors for
H
H
M
.

hyperplasia (eg, obesity) or if symptoms do not improve with medical management.


H
DJ
H

(Choice D) A hysteroscopy is a procedure that can be used to evaluate the endometrium for structural causes of abnormal
H
.

A
H

bleeding (eg, endometrial polyp, submucosat leiomyoma) and may be indicated if pharmacotherapy fails.
Ur
H
at
.

(Choice E) A copper-containing intrauterine device is a long-acting, reversible form of contraception that causes heavier,
H
\l

longer menses. Therefore, it is not recommended in patients with heavy menstrual bleeding and anemia.
H
.

a
H

Educational objective:
u:
N
.

Adolescents often have anovulatory cycles with irregular, heavy menstrual bleeding due to an immature hypothalamic—
e
N
l-‘

pituitary axis. Progesterone normalizes menstmation by stabilizing unregulated endometrial proliferation.


N
.

N
N
o;

References
N
.

A
N
Ur

- Abnormal uterine bleeding and dysfunctional uterine bleeding in pediatric and adolescent gynecology.
N
at
.

- Evaluation and management of adolescents with abnormal uterine bleeding.


\l
N
.

a
N
u:
u
.

Female Reproductive System 8. Breast Q


0
w
l-‘

_ Abnormal uterine bleeding Feedback End Block


w
.

N
OUNG‘UIJEWNH
= Item 4 of 40
Question Id: 12524 Lab Value.
9.
Notes
'
Reverse Color Text Zoom
$3

Cervicltls
.

O
H
.

H
H
M
.

H
DJ
.
H
H
.

A
H
U!
.

H
.

a
H
\l
.

H
.

a
H
u:
.

N
.

e
N
l-‘
.

N
.

N
N
u;
.

N
.

A
N
U!
.

N
OI
.

N
\l
.

OLMlmll‘l

Q Zoom In Q Zoom Out 9 Reset 517 Add To Flash Card


N
.

a
N
u:
.

u
.


o

Female Reproductive Syslem 8. Breast


0
w
l-‘
.

_ Abnormal uterine bleeding Feedback End Black


w
.

N
= Item 4 of 40 9. ' $3
IQNH
cauam Question Id: 12524 Lab Value. Notes Calulator Reverse Color Text Zoom
.

O
H
.

H
H
H
N
.

H
w
H
.

A
H
U!
.

H
.

a
H
\l
.

H
.

a
H
u:
.

N
.

e
N
l-‘
.

N
.

N
N
u;
.

N
.

A
N
U!
.

N
OI
.

N
\l
.

517 Add To Flash Card


N

(*1 Zoom In a Zoom Out 9 Reset


.

a
N
u:
.

u
.


o

Female Reproductive Syslem 8. Breast


0
w
l-‘
.

_ Abnormal uterine bleeding Feedback End Black


w
.

N
' $3
y
= Item 4 of 40 9.
OONGmNH Question Id: 12524 Notes Calulator Reverse Color Text Zoom
.

O
H
H
H
M
.

H
DJ
H
H
.

A
H
U!
H
.

a
H
\l
H
.

a
H
u:
N
.

e
N
l-‘
N
.

N
N
u;
N
.

A
N
U!
N
OI
.

N
\l

[-17 Add To Flash Card


N

Q Zoom In Q Zoom Out 9 Reset


.

a
N
u:
u
.

Ea
o

Female Reproductive Syslem 8. Breast


0
w
l-‘

_ Abnormal uterine bleeding Feedback End Black


w
.

N
E hern5of40 .VMark ® it
ۤ3
p
3 I
OG‘QOIJWNH QUESHO" Id: 4142 Tutorial Lab Values Notes Calculator Reverse Color Text Zoom

A 15—yearrold girl is brought to the office for evaluation of short stature. The patient has always been short compared to her
peers, but she recently tried out for her school's basketball team and now asks "if anything can make me taller." She has
not yet undergone menarche and has never been sexually active. The patient takes no medications or oral contraceptives.
Height is <3rd percentile and weight is at the 20th percentile. Blood pressure is 130/30 mm Hg and pulse is 92/min.
Physical examination reveals clear lung fields and a 2/6 systolic ejection murmur heard loudest in the left infraclavicular
area. The abdomen is soft and nontender. Breast development is Tanner stage 1, and pubic hair development is Tanner
Hl-l
HO

stage 4. Which of the following laboratory values is most likely to be present in this patient?
HH
LON

O A. High estrogen [0%]

V0 B. High FSH U4%]


UUUNNNNNNNNNNHHHHHH
Ni—‘owwsiatm-ti—‘ewwxiatmh

0 C. High inhibinA[2%]

O D. High testosterone [5%]

O E. Lowgrowth hormone [5%]

O F. Low LH [11%]

0m med
Correct answer
Ill 74% 1 Second — 05/08/2020
Answered correctly Tlme Spent E Last Updated

Explanation

_
Female Reproductive System 8. Breast
Turner syndrome
Q
0
Feedback End Block
E Item5of40 IVMark < D ® 5! 3
$3
>
'
OG‘QOIJWNH Question Id: 4142 Previous Next Tutorial Lab Values Notes Calculate r Reverse Color Text Zoom

Explanation

Primary ovarian insufficiency

Anterior
pituitary
Hl-l
HO

No negative feedback
HH
LON

TFSH
UUUNNNNNNNNNNHHHHHH
Ni—‘owwsiaIUI-tI—‘ewwxiomh

TLH
lEslrogen
llnhibin
Nonfunctional, lProgesterone
streak ovary
Wi

No menarche
euwwu No thelarche

This patient's short stature and primary amenon‘hea are suggestive of Turner syndrome (T5). TS is a chromosomal
abnormality caused by the complete or partial loss of an X chromosome (45,X). Additional clinical features of TS include
webbed neck, broad chest with widely spaced nipples, and cubitus valgus (angling of forearms away from body), TS is

Female Reproductive System a Breast


TU mer syndrome Ea Feedback
0
End Black
E rmm5of40 IPMark < D. 6;) ii a '
$3
p
OG‘QOIJWNH Question I“: 4142 Previous Next Tutorial Lab Values Notes Calculator Reverse Color Text Zoom

This patient‘s short stature and primary amenorrhea are suggestive of Turner syndrome (T8). T5 is a chromosomal
abnormality caused by the complete or partial loss of an X chromosome (45,X). Additional clinical features of TS include
webbed neck, broad chest with widely spaced nipples, and cubitus valgus (angling of forearms away from body). TS is
associated with multiple cardiac abnormalities including aortic coarctation, identified by a systolic murmur at the left
infraclavicular area and upper extremity hypertension.

Almost all patients with TS have ovarian dysgenesis ("streak ovaries"), which causes primary ovarian insufficiency
Hl-l
HO

resulting in low estrogen and progesterone levels (Choice A). Estrogen is required for normal breast development
(thelarche); therefore, patients with TS typically have minimal or no breast development. In addition, low estrogen levels
HH
LON

cause a lack of negative feedback on the hypothalamic—pituitary—ovarian axis, which leads to elevated FSH and LH levels.
Patients typically require estrogen replacement therapy to induce sexual development, improve growth/height, and increase
UUUNNNNNNNNNNHHHHHH
Nr—‘owwsiaIm-tr—‘ewwxromh

bone mineral density.

(Choice C) lnhibin A is released by the ovaries; therefore, levels are low in patients with TS.

(Choice D) Elevated testosterone is seen in complete androgen insensitivity syndrome; patients are phenotypic women
(46,XY karyotype) with primary amenorrhea, normal breast development, and absent pubic hair. In contrast, testosterone
levels are low in patients with TS due to lack of production by the ovaries.

(Choice E) Growth hormone (GH) deficiency can cause short stature and delayed puberty but is not associated with
coarctation of the aorta. GH levels are typically normal in T8 as short stature results from loss of X chromosome genes.
GH therapy, however, is still recommended in patients with TS to improve height potential.

(Choice F) Hypothalamic or pituitary etiologies of amenorrhea cause decreased LH levels. However, this patient‘s aorlic
coarctation makes TS more likely.

Educational objective:
Primary amenorrhea in patients with short stature and aortic coarctation is suggestive of Turner syndrome. Patients

_
Female Reproductive System a Breast
TU mer syndrome
Q
0
Feedback End Block
$3
p
_ lam 5 of 40
® 3‘
'
3
. '
OG‘QOIJWNH
= . Id-. 4142
QUESNO“ Tutorial Lab Values Notes Calculator Reverse Color Text Zoom

Patients typically require estrogen replacement therapy to induce sexual development, improve growth/height, and increase
bone mineral density.

(Choice C) Inhibin A is released by the ovaries; therefore, levels are low in patients with TS.

(Choice D) Elevated testosterone is seen in complete androgen insensitivity syndrome; patients are phenotypic women
(46,XY karyotype) with primary amenorrhea, normal breast development, and absent pubic hair. In contrast, testosterone
levels are low in patients with TS due to lack of production by the ovaries.
.

O
H
H
H

(Choice E) Growth hormone (GH) deficiency can cause short stature and delayed puberty but is not associated with
M
.

coarctation of the aorta. GH levels are typically normal in T8 as short stature results from loss of X chromosome genes.
DJ
H

GH therapy, however, is still recommended in patients with TS to improve height potential.


H
.

A
H
Ur

(Choice F) Hypothalamic or pituitary etiologies of amenorrhea cause decreased LH levels. However, this patient's aortic
H
at
.

coarctation makes TS more likely.


H
\l
H

Educational objective:
.

a
H

Primary amenorrhea in patients with short stature and aortic coarctation is suggestive of Turner syndrome. Patients
ND
N
.

typically have ovarian dysgenesis (ie, primary ovarian insufficiency), which results in low estrogen levels. These low
6
N
l-‘

estrogen levels cause a lack of negative feedback on the hypothalamic—pituitary-ovarian axis, resulting in high FSH levels.
N
.

N
N
o;

References
N
.

A
N
Ur

- Turner syndrome in girls presenting with coarctation of the aorta.


N
at
.

- Clinical review: Turner syndrome: updating the paradigm of clinical care.


\l
N
.

a
N
ND
u

Ea
.

Female Reproductive System 8. Breast


0
w
l-‘

_ Turner syndrome Feedback End Block


w
.

N
at
Item 5 of 40 l ‘7 "m ‘ p ® 5' 3. '
OG‘QOIJWNH 0095110“ Id: 4142 Previous Next Tulon'al Lab Values Nata Calulator Reverse Color Text Zoom

associated with multiple cardiac abnormalities including aortic coarctation, identified by a systolic murmur at the left
Exhibit Display

l'lypomalamic-pimitary-avaflan axis
.

O
H
.

H
H
H
N
.

H
w

i
H
.

A
H
U1
.

H
.

a
H
\l
.

H
.

a
H
u:
.

N
.

e
N
l-‘
.

N
.

N
N
o;
.

N
.

A
N
U1
.

N
0.
.

N
\l
.

Q Zoom In Q Zoom Out 3 Reset 517 Add To Flash Card


N
.

a
N
u:
.

u
.

Ea
o

Female Reproductive Sys‘em 8. Breast


0
w
l-‘
.

_ Turner syndrome Feedback End Black


w
.

N
mAwNi—A
E hemtiofdt)
Question Id: 3241
.m ®
Tutorial
5‘
Lab Values
a
Notes Calculator
'
Reverse Color Text Zoom
ۤ3
A 16—yearrold girl comes to the office for emergency contraception. She had unprotected sexual intercourse last night and
is worried that she may become pregnant Her last menstrual period was 1 week ago, and she has had regular menses
since menarche. Physical examination is normal and urine pregnancy test is negative. After various emergency
contraceptive options are discussed, the patient asks for a pill option and that her parents not be notified about this visit. In
most states, which of the following is the most appropriate management of this patient?

O A. Obtain parental consent prior to providing emergency contraception [2%]


11

O B. Prescribe misoprostol (prostaglandin E1) pills [15%]


13
9/ O C. Provide levonorgestrel pill [69%]
15
O D. Reassure patient that pregnancy is unlikely and no emergency contraception is required [3%]

17 O E. Supply a short course of high-dose oral contraceptives [9%]

19

0m i'tted
21 Correct answer
Ill 69% 1 Second — 01/212020
Answered correctly Time Spent E Last Updated
23

Explanation
25

27
Emergency contraception

29 Timing after _ . . .
Method _ . . Efficacy Contraindications
31 Female Reproductive System a Breast
Contraception a, CI Feedback
0
End Block
UiAwNi—A
hemSofAIO
Question Id: 3241
IVMam <l>
Previous Next Tutorial
ii
Lab Values
3
Notes Calculator
'
Reverse Color Text Zoom
$3
Explanation

Emergency contraception

Timing after _ _ _
Method _ Efficacy Contraindications
intercourse
11
_ _ - Acute pelvic infection
Copper—containing _ _ _ _
13 _ _ - Severe uterine cavrty distortion
intrauterine 0-120 hr 299% _ V
_ . Wilson disease
deVIce _ _
15 . Complicated organ transplant failure

17 Ulipristal 0-120 hr 9896-9991: - None

Levonorgestrel 0-72 hr 59%-94% - None


19
Oral contraceptives" 0—72 hr 47%—89% . None
21
I'Combined estrogen/progestin oral contraceptives containing levonorgestrel or norgestrel‘

23
Patients with recent unprotected intercourse may be candidates for postcoital or emergency contraception (EC) for
pregnancy prevention. Pregnancy status determines eligibility for EC use:
25
- Patients with a positive pregnancy test are not candidates for EC because implantation has already occurred.
27 - In contrast, patients with a negative pregnancy test are still at risk for unintended pregnancy and are candidates for EC
(Choice D).
29
The levonorgestrel pill (ie, Plan B) is the most readily available form of EC; the high dose of progestin in these pills
31 Female Reproductive System a Breast
_ Contraception Ea Feedback
0
End Block
mAwNi—A
E men-Sam
009550“ Id: 32“
.Vmflk ®
Tutorial
5)
Lab Values
g
Notes Calculator
I
Reverse Color Text Zoom
$3
Patients with recent unprotected intercourse may be candidates for postcoital or emergency contraception (EC) for
pregnancy prevention. Pregnancy status determines eligibility for EC use:

- Patients with a positive pregnancy test are not candidates for EC because implantation has already occurred.
- In contrast, patients with a negative pregnancy test are still at risk for unintended pregnancy and are candidates for EC
(Choice D).

The levonorgestrel pill (ie, Plan B) is the most readily available form of EC; the high dose of progestin in these pills
11 prevents pregnancy by inhibiting the LH surge, thereby delaying ovulation. Additional mechanisms include cervical mucus
thickening and thinning of the endometrial lining. Levonorgestrel pills are highly effective (ie, up to 94%); however,
13 effectiveness decreases over the course of 72 hours (ie, as low as 59%). Therefore, levonorgestrel pills should be
provided as soon as possible after unprotected intercourse. Most states allow adolescents to receive confidential care for
15
contraception, pregnancy, and sexually transmitted diseases without parental consent (Choice A).

17 (Choice B) In patients with a positive pregnancy test (ie, implanted pregnancy), misoprostol (prostaglandin E1) can be
used for medical abortion or pregnancy termination by stimulating uterine contractions, Misoprostol does not prevent
19 pregnancy and, therefore, is not used for EC.

21
(Choice E) Combination oral contraceptives, commonly used for precoital contraception, may also be used for EC by
taking multiple pills simultaneously to achieve the progestin level required to delay ovulation. However, the high estrogen
23 content typically causes intolerable side effects (eg, severe nausea), and this method is less effective (ie, 4796—8996) than
levonorgestrel pills.
25
Educational objective:

27 The levonorgestrel pill (ie, Plan B) are a readily available and effective emergency contraception option that prevent
pregnancy by delaying ovulation. In most states, adolescents seeking pregnancy prevention options may receive
29 confidential medical care without parental consent.

31 Female Reproductive System a Breast


_ Contraception
Q
0
Feedback End Block
urwi—A
E hem s of!!!)
Question I“: 32“ -
Q) il
Lab Values
g
Notes Calculator
'
Reverse Color Text Zoom
$3
provided as soon as possible after unprotected intercourse. Most states allow adolescents to receive confidential care for
contraception, pregnancy, and sexually transmitted diseases without parental consent (Choice A).

(Choice B) In patients with a positive pregnancy test (ie, implanted pregnancy), misoprostol (prostaglandin E1) can be
used for medical abortion or pregnancy termination by stimulating uterine contradicns. Misoprostol does not prevent

9 pregnancy and, therefore, is not used for EC.


. 10
(Choice E) Combination oral contraceptives, commonly used for precoital contraception, may also be used for EC by
11
taking multiple pills simultaneously to achieve the progestin level required to delay ovulation. However, the high estrogen
- 12
content typically causes intolerable side effects (eg, severe nausea), and this method is less effective (ie, 4796—8996) than
13
Ievonorgestrel pills.
. 14
15 Educational objective:
. 16 The levonorgestrel pill (ie, Plan B) are a readily available and effective emergency contraception option that prevent
17
pregnancy by delaying ovulation. In most states, adolescents seeking pregnancy prevention options may receive
. 18
confidential medical care without parental consent.
19
. 20
21 References
. 22
- Emergency contraception review: evidence-based recommendations for clinicians,
23
. 24 - Adolescents and emergency contraception: update 2011.
25
- Interventions for emergency contraception.
. 26
27 - Practice bulletin no. 152: emergency contraception.
. 28
29
. 30
31 Female Reproductive System 8. Breast
_ Contraception
Q
0
Feedback End Black
E ttem7or4o [Valiant ® it Q I
DGIOMAUNH
-

QUESHO" Id: 4307 Tutorial Lab Values Notes Calculator Reverse Color

A47—yearrold woman comes to the office due to lower abdominal pain that is relieved with urination. The pain has
worsened over the past 2 months and she now urinates hourly to minimize the pain. The patient is sexually active but has
avoided intercourse recently because it has become painful. She had 4 vaginal deliveries in her 305 and a tubal ligation
u...

after her last delivery. The patient has irritable bowel syndrome and takes no medications. Vital signs are normal. BMI is
27 kg/mi. There is tenderness to palpation over the lower abdomen. 0n bimanual examination, the anterior vaginal wall is
tender to palpation, but there is no cervical motion tenderness. The remainder of the pelvic examination is normal, and
HO
HH

Valsalva does not elicit any noticeable changes. Urinalysis results are as follows:
M
.

Specific gravity 1.013


DJ
H
H

Protein none
.

A
H
Ul

Blood trace
H
at
.

Leukocyte esterase negative


\l
H
.

Nitrites negative
H
ND
N

Bacteria none
.

6
N
l-‘

White blood cells 1—2/hpf


N
.

N
N
o;

Red blood cells 1-2/hpf


N
.

A
N

A urine culture is negative. Which of the following is the most likely diagnosis in this patient?
Ul
N
at
.

O A. Cystocele[21%]
N
\l
N
.

v V B. Interstitial cystitis [56%]


N
u:
u
.

Female Reproductive System a Breast Q


0
w
l-‘

Interstitial cystitis Feedback End Block


w
.

N
DGIOMAUNH
E hem 7 of40 ® 5‘ a '
$3
-

Question Id: 4307 ' Tutorial Lab Values Notes Calculate r Reverse Color Text Zoom

Protein none

Blood trace
u...

Leukocyte esterase negative

Nitrites negative

Bacteria none
HO
HH

White blood cells 1—2/hpf


M
.

H
DJ
H

Red blood cells 1-2/hpf


H
.

A
H

A urine culture is negative. Which of the following is the most likely diagnosis in this patient?
U1
H
.

O A. Cystooele [21%]
H
\l
H
.

V0 B. Interstitial cystitis [56%]


H
u:
N

O C. Overactive bladder [4%]


.

e
N
l-‘

O D. Pelvic inflammatory disease [2%]


N
.

N
N
u;

O E. Urethral diverticulum [15%]


N
.

A
N
Ul
N
OI
.

COmrtfitied “I 55%
N

23econas _ 05/0912020
\l

Bone answer Answered conedty


N

Time Spent E Last undated


.

a
N
u:
u

Ea
.

Female Reproductive System 8. Breast


0
w
l-‘

_ Interstitial cystitis Feedback End Block


w
.

N
DGIOMAUNH
hem70f40
IVMam <ii> ® 51$ '
$3
-

Question Id: 4807 Tutorial Lab Values Notes Calculate r Reverse Color Text Zoom
Previous Next

Explanation
u...

Interstitial cystitis (bladder pain syndrome)

More common in women


Epidemiology
Associated with psychiatric & pain disorders (eg, fibromyalgia)
HO
HH

- Bladder pain with filling, relief with voiding


M
.

Clinical presentation - TUrinary frequency, urgency


DJ
H

- Dyspareunia
H
.

A
H
Ur

- Bladder pain with no other cause for 26 weeks


Diagnosis
H
at
.

- Normal urinalysis
H
\l
H

- Not curative, focus is on improving quality of life


.

a
H

- Behavioral modification, avoidance of triggers, physical therapy


u:

Treatment
N
.

Amitriptyline, pentosan polysulfate sodium


N
l-‘

Analgesics for acute exacerbations


N
.

N
N
o;

Interstitial cystitis (IC) (also known as painful bladder syndrome) is a chronic, painful bladder condition of uncertain
N
.

etiology. Patients typically have pain that is exacerbated by bladder filling and relieved by voiding, Symptom onset is
N
Ur

typically gradual and worsens over a period of months, Other clinical features include urinary frequency and urgency,
N
at
.

chronic pelvic pain, and dyspareunia. IC typically presents in women age >40 and is associated with other chronic pain
N
\l

conditions (eg, fibromyalgia, endometriosis, irritable bowel syndrome), sexual dysfunction, and psychiatric illness (eg,
N
.

depression, anxiety).
N
u:
u

Ea
.

Female Reproductive System a Breast


0
w
l-‘

_ Interstitial cystitis Feedback End Block


w
.

N
DGIOMAUNH
E hem7of40 -?Mark < D. Q) i g p
$3
-

(109550“ Id: 4307 Previous Next Tutonal Lab Values Notes Calculator Reverse Color Text Zoom

Interstitial cystitis (IC) (also known as painful bladder syndrome) is a chronic, painful bladder condition of uncertain
etiology. Patients typically have pain that is exacerbated by bladder filling and relieved by voiding. Symptom onset is
typically gradual and worsens over a period of months. Other clinical features include urinary frequency and urgency,
u...

chronic pelvic pain, and dyspareunia. IC typically presents in women age >40 and is associated with other chronic pain
conditions (eg, fibromyalgia, endometriosis, irritable bowel syndrome), sexual dysfunction, and psychiatric illness (eg,
depression, anxiety).
HO
HH

The diagnosis of IC is largely clinical; however, additional laboratory testing—including a urinalysis, postvoid residual, and
sexually transmitted infection screening—is performed to exclude other conditions (eg, cystitis, urinary obstmction,
M
.

malignancy). Management includes bladder training, fluid management, analgesics, and avoidance of any precipitating
DJ
H

agents (eg, caffeine, alcohol, artificial sweeteners).


H
.

A
H
Ur

(Choice A) Cystocele (bladder prolapse into the anterior vaginal wall) may cause dyspareunia and urinary symptoms (eg,
H
at
.

frequency). Although cystoceles are common in mulliparous women, this patient has no changes on pelvic examination
H
\l

with Valsalva (ie, no prolapse).


H
.

a
H

(Choice C) An overactive bladder presents with urinary urgency (an intense need to urinate) that can be accompanied by
u:
N

nocturia and frequency. There is no associated dyspareunia or pelvic pain relieved with voiding.
.

e
N
l-‘

(Choice D) Pelvic inflammatory disease typically presents with lower abdominal pain and dyspareunia; however, patients
N
.

typically have concomitant cervical motion or adnexal tenderness, making this diagnosis unlikely.
N
o;
N
.

(Choice E) A urethral diverticulum, an abnormal outpouching of the urethra, can cause urethral tenderness, urinary
N
Ur

frequency, and dyspareunia. However, it also typically presents with a tender anterior vaginal mass, pumlent urethral
N
at
.

discharge, and increased pain with voiding.


N
\l
N

Educational objective:
.

a
N
u:

Interstitial cystitis (painful bladder syndrome) is an idiopathic, chronic condition characterized by pain that is exacerbated by
u
.

Female Reproductive System a Breast Q


0
w
l-‘

_ Interstitial cystitis Feedback End Block


w
.

N
E turn 7 of!!!) Q) i! g '
DGIOMAUNH
-

Question Id: 4807 - Tutorial Lab Values Notes Calculator Reverse Color
gun-u. no

agents (eg, caffeine, alcohol, artificial sweeteners).

(Choice A) Cystocele (bladder prolapse into the anterior vaginal wall) may cause dyspareunia and urinary symptoms (eg,
frequency). Although cystoceles are common in multiparous women, this patient has no changes on pelvic examination
u...

with Valsalva (ie, no prolapse).

(Choice C) An overactive bladder presents with urinary urgency (an intense need to urinate) that can be accompanied by
nocturia and frequency. There is no associated dyspareunia or pelvic pain relieved with voiding.
HO
HH

(Choice D) Pelvic inflammatory disease typically presents with lower abdominal pain and dyspareunia; however, patients
M
.

typically have concomitant cervical motion or adnexal tenderness, making this diagnosis unlikely.
DJ
H
H

(Choice E) A urethral diverticulum, an abnormal outpouching of the urethra, can cause urethral tenderness, urinary
.

A
H
Ul

frequency, and dyspareunia. However, it also typically presents with a tender anterior vaginal mass, purulent urethral
H
at
.

discharge, and increased pain with voiding.


H
\l
H

Educational objective:
.

a
H

Interstitial cystitis (painful bladder syndrome) is an idiopathic, chronic condition characterized by pain that is exacerbated by
u:
N
.

bladder filling and relieved by voiding. Additional clinical features may include dyspareunia and urinary frequency and
e
N
l-‘

urgency.
N
.

N
N
o;

References
N
.

A
N
Ul

- Advances in diagnosis and treatment of interstitial cystitis/painful bladder syndrome.


N
at
.

- Treating interstitial cystitis/bladder pain syndrome as a chronic disease.


\l
N
.

a
N
tr:
u
.

Female Reproductive System 8. Breast Q


0
w
l-‘

_ Interstitial cystitis Feedback End Block


w
.

N
E memsouo
QUESHO" Id: 3352
.VMark ®
Tutorial
5]
Lab Values
g
Notes Calculator
I
Reverse Color Text Zoom
ۤ3
A26—yearrold nulliparous woman comes to the office due to severe pain during sexual intercourse. The pain is "particularly
unbearable" with deep penetration, which has caused the patient to avoid sexual intimacy for the past few months. This has
placed a significant strain on her relationship with her husband of 2 years, who has been her only sexual partner. They
have been using condoms with lubrication during intercourse, but this has not improved the pain. Menses occur every 30
days, are painful, and last for 5 days with moderate bleeding. The patient has no chronic medical conditions and takes no
daily medications. BMI is 24 kglml. \fital signs are normal. Speculum examination is normal. Which of the following is the
most appropriate treatment for this patient's condition?

O A. Cognitive—behavioral therapy [12%]

V0 B. Combination oral contraceptives [43%]

O C. Couples therapy and sex education [20%]

O D. Pelvic floor physical therapy [11%]

O E. Vaginal dilators [12%]

Omitted
c rrect I" 43% ZSeoonds — 02/190020
Bo answer Answered correctly Time Spent E Last Updated

Explanation

Endometriosis
Female Reproductive System 8. Breast
Endometriosis
Q
0
Feedback End Block
Item 3 of 40
Question Id: 3862 IVMark <ir>
Previous Next
62)
Tutorial
il
Lab Values
3
Notes Calculate r
'
Reverse Color Text Zoom
$3
Explanation

Endometriosis
Pathogenesis Ectopic implantation of endomelrial glands

Dyspareunia
Dysmenorrhea
Clinical features Chronic pelvic pain
Infertility
Dyschezia

lmmobile uterus
Cervical motion tenderness
Physical examination Adnexal mass
Rectovaginal septum, posterior cul-de—sac, uterosacral ligament
nodules

Diagnosis - Direct visualization & surgical biopsy

- Medical (oral contraceptives, NSAIDs)


Treatment
Surgical resection

NSAIDs = nonsteroidal anti-inflammatory drugs.

This patient with severe dyspareunia and dysmenorrhea likely has endometriosis, ectopic implants of endometrial glands
and stroma in the abdomen and pelvis. Like normal endometrium, these intraperitoneal implants proliferate and shed

Female Reproductive System a Breast


Endometriosis
0
Feedback End Black
E ham s of!!!) CD it 3 I
again
QUBSHO" Id: 3352 ' Tutorial Lab Values Notes Calculator Reverse Color

This patient with severe dyspareunia and dysmenorrhea likely has endometriosis, ectopic implants of endometrial glands
and stroma in the abdomen and pelvis, Like normal endometrium, these intraperitoneal implants proliferate and shed
during menses; however, because they have no outlet, endometnosis implants can cause recurrent, slowly progressive
pelvic inflammation and fibrosis. The chronic inflammation, fibrosis, and subsequent adhesion formation result in the
characteristic pain symptoms associated with endometriosis. Pain symptoms vary with implant location and severity; the
most common pain symptoms are dysmenorrhea and severe dyspareunia (ie, pain with intercourse), particularly with
deep penetration, due to the proximity of the vagina to the uterus, bladder, and rectum.

Patients with a presumptive diagnosis of endometriosis based on clinical symptoms are initially managed with nonsteroidal
anti—inflammatory drugs (which decrease inflammation) and combination oral contraceptives (which suppress ovulation
and reduce menstruation). When endometriosis does not improve with medical management, surgical evaluation via
laparoscopy is required for definitive diagnosis (eg, biopsy) and resection of implants.

(Choices A, C, D, and E) Vaginismus, also known as genitopelvic pain/penetration disorder, is a common cause of
dyspareunia. In contrast to endometriosis, vaginismus causes pain with any vaginal penetration (eg, intercourse, speculum
examination) In addition, vaginismus typically causes diffuse pain due to repeated involuntary pelvic floor muscle
contractions; this pain is unrelated to menses. Patients with vaginismus are typically treated with multiple modalities,
including cognitive-behavioral therapy, couples therapy and sex education, pelvic floor physical therapy, and vaginal
dilators.

Educational objective:
Endometriosis can present with dysmenonhea and deep dyspareunia. Symptomatic patients are offered empiric medical
therapy with nonsteroidal anti—inflammatory drugs and/or combination oral contraceptives, which treat inflammation and
suppress ovarian stimulation of endometriosis.

Female Reproductive System a Breast


Endometriosis
Q
0
v Feedback End Block
m‘de’tu‘fiUNt—l
Mem 9 oh“)
QUESHO" Id: “755
I ‘7 Mark ®
Tutorial
it
Lab Values
Q
Notes Calculator
I
Reverse Color Text Zoom
ۤ3
ASS—yearrold woman comes to the office due to breast pain. For the past 2 months, the patient has had intermittent
bilateral breast pain that usually begins the week prior to menses and subsides on the first day of her menstrual cycle The
pain is primarily in the bilateral upper outer quadrants and does not radiate to the back or neck. Six months ago, the patient
started a new job that requires long hours of sedentary work; she has gained 4.5 kg (10 lb). She has no chronic medical
conditions and takes no daily medications. The patient has regular monthly menses, and her last menstrual period was a
week ago. BMI is 25 kg/mz. \fital signs are normal. Breast examination shows bilateral, symmetric, large breasts with
H
O

slight tenderness over the upper outer quadrants. There are no skin changes, nipple discharge, masses, or axillary
H
H

lymphadenopathy. Which of the following is the best next step in management of this patient?
HH
LON

O A. Breast-reduction surgery [0%]


H
.

A
H
Ur

O B. Breast ultrasound and mammography [8%]


H
at
.

O C. Combined estrogen/progestin oral contraceptives [15%]


\l
H
.

O D. Selective serotonin reuptake inhibitor [1%]


H
u:
N

V O E. Supportive bra and nonsteroidal anti—inflammatory drugs [74%]


.

e
N
l-‘
N
.

N
N
o;

Omitted
N

II 74% 1Second — O5I21I2020


.

Correct answer I
Answered correctly Tlme Spent E Last Updated
N
Ur
N
at
.

N
\l

Explanation
N
.

a
N
u:
u
.

Female Reproductive System 8. Breast Q


0
w
l-‘

Breast cancer Feedback End Black


w

v
.

N
hem 9 of 40
IPMark < D Q) il 5 '
m‘de’ll’lfiUND—l
Question Id: 14766 Next Tutorial Lab Values Notes Calculate r Reverse Color Text Zoom
Previous

Management of breast pain

Breast pain
HO
HH
H
N

Cyclic. bilateral, difiuse Noncyclic, unilateral, focal


H
w
H
.

A
H
U!
.

H
a.
.

Mass No mass No mass Mass


H
\l
.

H
.

a
H
u:
.

N
.

Imaging Observe Imaging Biopsy.


N
l-‘
.

referral to breast
N
.

surgeon
N
u;
.

N
.

Abnormal Normal
N
U!
.

N
OI
.

N
\l
.

N
.

Biopsy Observe
N
u:
.

Ea
.

Female Reproductive Syslem 8. Breast


0
w
l-‘
.

Breast cancer Feedback End Black


w
.

N
m‘de’tu‘fiUNt—t
hem 9 of!!!)
009550“ Id: "755
I \7 Mark ®
Tutorial
ii
Lab Values
3
Notes Calculator
p
Reverse Color Text Zoom
$3
Mastalgia (breast pain) is common in women and typically classified as either cyclic or noncyclic:

- Cyclic breast pain is benign and due to estrogen and progesterone changes associated with the menstrual cycle.
The cyclic pain usually begins up to 2 weeks prior to and subsides with the onset of menses. The pain is typically
described as heavy, diffuse, dull, and bilateral. Patients may have increased pain or tenderness of the bilateral upper
outer breast quadrants; however, there are typically no examination abnormalities (eg, breast mass, skin changes,
axillary lymphadenopathy).
H
O

- Noncyclic breast pain, in contrast, is not associated with menses, often has a variable location, and involves a single
H
H

breast. Patients may have an associated breast mass. Patients with noncyclic, unilateral, or focal breast pain and
HH
LON

those with abnormal examination findings (eg, breast mass) require further evaluation with breast imaging or biopsy
(Choice B).
H
.

A
H
U1

Patients with cyclic breast pain with no associated mass are at low risk for malignancy. Therefore, they are managed
H
.

with reassurance, observation, and symptom management such as the use of a supportive bra (which minimizes breast
H
\l

movement) and nonsteroidal anti-inflammatory drugs.


H
.

a
H

(Choice A) Breast—reduction surgery may be indicated in patients with large, heavy breasts and associated chronic pain of
ND
N
.

the neck, shoulders, or back. These patients typically have noncyclic breast pain exacerbated by activity (eg, exercise); this
N
l-‘

patient's cyclic pain would not benefit from surgery,


N
.

N
N

(Choice 6) Combined estrogen/progestin oral contraceptives are a potential cause of cyclic breast pain due to alterations
o;
N

in estrogen and progesterone levels; they are not used for treatment.
.

A
N
Ul

(Choice D) Selective serotonin reuptake inhibitors are used to treat premenstrual syndrome, which can present with cyclic
N
at
.

breast pain occurring in the luteal phase of the menstrual cycle. However, the diagnosis requires both somatic and
N
\l
N

behavioral (eg, irritability, emotional lability) symptoms related to menses, which are not present in this patient.
.

a
N
u:

Educational objective:
u
.

Female Reproductive System a Breast Q


0
w
l-‘

Breast cancer Feedback End Block


w

v
.

N
m‘de’tl’lfiUNt—l
them 9 onto
QUESHO" Id: “755
I ‘7 Mark ®
Tutorial
it
Lab Values
3
Notes Calculator
I
Reverse Color Text Zoom
$3
Patients with cyclic breast pain with no associated mass are at low risk for malignancy. Therefore, they are managed
with reassurance, observation, and symptom management such as the use of a supportive bra (which minimizes breast
movement) and nonsteroidal anti-inflammatory drugs.

(Choice A) Breast—reduction surgery may be indicated in patients with large, heavy breasts and associated chronic pain of
the neck, shoulders, or back. These patients typically have noncyclic breast pain exacerbated by activity (eg, exercise); this
H
O

patient's cyclic pain would not benefit from surgery.


H
H

(Choice 6) Combined estrogen/progestin oral contraceptives are a potential cause of cyclic breast pain due to alterations
HH
LON

in estrogen and progesterone levels; they are not used for treatment.
H

(Choice D) Selective serotonin reuptake inhibitors are used to treat premenstrual syndrome, which can present with cyclic
.

A
H
Ul

breast pain occurring in the luteal phase of the menstrual cycle. However, the diagnosis requires both somatic and
H
at
.

behavioral (eg, irritability, emotional lability) symptoms related to menses, which are not present in this patient.
H
\l
H

Educational objective:
.

a
H

Breast pain is typically benign when it is cyclic (ie, associated with menses), bilateral, and diffuse with no associated
u:
N
.

examination abnormalities (eg, breast mass). Patients benefit from reassurance and symptom management (eg, supportive
e
N
l-‘

bra, nonsteroidal anti-inflammatory drugs).


N
.

N
N
o;

References
N
.

A
N
Ul

- Breast pain.
N
at
.

- Common breast problems.


\l
N
.

a
N
ND
u
.

Female Reproductive System 8. Breast Q


0
w
l-‘

Breast cancer Feedback End Block


w

v
.

N
DGNQMfiUNt—t
Mem100f40
QUESHO" Id: 12036
'VMark ®
Tutorial
5]
Lab Values
g
Notes Calculator
I
Reverse Color Text Zoom
ۤ3
A28—yearrold woman comes to the office due to discomfort during urination for the past 3 months. Antibiotics for a urinary
tract infection were prescribed at an urgent care center a few weeks ago, but symptoms have persisted despite treatment.
The patient has also noticed increasing pain with the passage of stool for the last year, parlicularty during menses. Bowel
movements occur daily and are soft and not watery. Her symptoms have not improved with the elimination of caffeine and
alcohol or an increase in dietary fiber. The patient has also had intermittent abdominal pain for the past 7 months. She
takes no daily medications and has had no surgeries. BMI is 19 kg/m2_ Examination shows suprapubic tenderness and an
o
._.

immobile, retroverted uterus. Rectovaginal examination reveals nodularity in the posterior cul-de—sac. Urinalysis and urine
H
,_.

culture are normal. Surgical biopsy of the pelvic nodules would most likely show which of the following?
HH
LION

O A. Colorectal hamartoma [1%]


u

Ih
H
U!
H

VO B. Endometrial glands and stroma [86%]


u

at
H

O C. Endometrial intraepithelial neoplasia [1%]


\l
H
u

H
3

O D. Invasive urothelial carcinoma [2%]


ND
H
u

O E. Serous epithelial ovarian carcinoma [0%]


N
6
i-‘
N

O F. Uterine myometrium [8%]


u

N
N
0.]
N
u

N
A
U!

Omitted
N

ll 86% 1Se00nd — 02/19/2020


Correct answer I
u

Answered oorreclty Tlme Spent E Last Updated


at
N
\l
N
u

N
3
ND
N

Explanation
.

u
0

31 Female Reproductive System 8. Breast


Endometriosis
Q
0
v Feedback End Block
N
U
u
hem 10 of 40
IVMam <ii> ® il 3 '
$3
DGNQMfiUND—t Question Id: 12056 Previous Next Tutorial Lab Values Notes Calculate r Reverse Color Text Zoom

Explanation

Endometriosis

Pathogenesis Ectopic implantation of endometn'al glands


o
._.

Dyspareunia
H
,_.

Dysmenorrhea
HH
LION

Clinical features Chronic pelvic pain


Infertility
H
.

A
H

Dyschezia
U!
H
or
.

lmmobile uterus
H
\l

Cervical motion tenderness


H
.

Physical examination Adnexal mass


H
u:
N

Rectovaginal septum, posterior cul-de-sac, uterosacral ligament


.

e
N

nodules
l-‘
N
.

Diagnosis - Direct visualization & surgical biopsy


N
o;
N

- Medical (oral contraceptives, NSAIDs)


.

Treatment
N
U!

Surgical resection
N
OI
.

NSAIDs = nonsteroidal anti—inflammatory drugs.


\l
N
.

Endometriosis affects women of reproductive age and is characterized by the ectopic implantation of endometrial
N
u:

lands and stroma causino chronic elvic min 8 mtoms and examination findins va den-endin on the secific
u

Ea
.

Female Reproductive System a Breast


0
w
l-‘

Endometriosis Feedback End Block


w
.

N
DGNQMfiU-INH
ltem100f40
(109550“ Id: 12955
Ivmm ®
Tutorial
5]
Lab Values
g
Notes Calculator
I
Reverse Color Text Zoom
$3
Endometriosis affects women of reproductive age and is characterized by the ectopic implantation of endometrial
glands and stroma, causing chronic pelvic pain. Symptoms and examination findings vary, depending on the specific
location of endometriotic implants.

Dysuria or suprapubic tenderness and dyschezia (pain with defecation) can be caused by implants on the bladder and
rectovaginal septum, respectively. Bowel symptoms commonly worsen with menses. Rectovaginal nodularity is highly
characteristic of endometfial implants and fibrosis of the posterior cul—de—sac. The uterus is often immobile and fixed in a
o
._.

retroverted position due to scarring from continued cyclic shedding of ectopic endometrial tissue that has no outlet. Other
H
,_.

common clinical features include an adnexal mass (endometrioma) and cervical motion tendemess.
HH
LION

Definitive diagnosis of endometriosis is by direct visualization with biopsy during surgical exploration, typically laparoscopy.
H
.

Treatment is surgical removal of visible lesions and lysis of fibrotic tissue, but recurrence is common. Conservative
H
Ur

treatment includes oral contraceptives and nonsteroidal anti-inflammatory drugs.


H
at
.

H
\l

(Choice A) Colorectal hamartomas, typically benign colonic tumors, may cause abdominal pain due to mass effect;
H
.

however, hamartomas do not worsen with menses or cause pelvic inflammation (eg, immobile uterus).
H
u:
N

(Choice 8) Endometrial intraepithelial neoplasia, atypical overgrowth of the intrauterine endometrial lining, typically occurs
.

e
N

in women with risk factors (eg, obesity, chronic anovulation). Patients typically have abnormal uterine bleeding or
l-‘
N

postmenopausal bleeding.
.

N
N
c;

(Choice D) Invasive urothelial carcinoma can cause dysuria, but patients typically have painless hematuria; this patient's
N
.

urinalysis is normal, making this diagnosis unlikely.


N
Ur
N
at
.

(Choice E) Serous epithelial ovarian carcinoma can cause abdominal pain, but the typical presentation is bloating or
N
\l

ascites in a postmenopausal patient with an adnexal mass.


N
.

a
N

(Choice F) Leiomyoma (ie, fibroids), benign tumors of uterine myometrium, may cause abdominal pain and urinary or
u:
u
.

Female Reproductive System a Breast Q


0
w
l-‘

E ndometriosis Feedback End Block


w
.

N
DGNQMfiUNt—l
Imm1flof4u
Question Id: 12955
®
Tutorial
5‘
Lab Values
g
Notes Calculator
'
Reverse Color Text Zoom
$3
however, hamartomas do not worsen with menses or cause pelvic inflammation (eg, immobile uterus).

(Choice (3) Endometrial intraepithelial neoplasia, atypical overgrowth of the intrauterine endometrial lining, typically occurs
in women with risk factors (eg, obesity, chronic anovulation). Patients typically have abnormal uterine bleeding or
postmenopausal bleeding.

(Choice D) Invasive urothelial carcinoma can cause dysuria, but patients typically have painless hematuria; this patient's
o

urinalysis is normal, making this diagnosis unlikely.


._.
H
,_.

(Choice E) Serous epithelial ovarian carcinoma can cause abdominal pain, but the typical presentation is bloating or
HH
LION

ascites in a postmenopausal patient with an adnexal mass.


u

(Choice F) Leiomyoma (ie, fibroids), benign tumors of uterine myometn'um, may cause abdominal pain and urinary or
Ih
H
Ul
H

bowel symptoms due to mass effect; however, symptomatic patients typically have an irregularly enlarged uterus, making
u

at
H

this diagnosis less likely.


\l
H

Educational objective:
u

H
3

Endometriosis is ectopic implants of endometrial glands and stroma in the abdominopelvic cavity. Symptoms are based on
ND
H
u

implant location. Bladder implants may cause suprapubic tenderness and dysuria. Rectovaginal implants may cause
N
6
l-‘
N

dyschezia, rectovaginal nodularity, and pelvic fibrosis, as evidenced by an immobile uterus on examination.
u

N
N
0.!
N

References
u

N
A
Ul
N

- Clinical practice. Endometriosis.


u

at
N

- Endometriosis.
\l
N
u

N
3
ND
N
.

u
0

31 Female Reproductive System 8. Breast


Endometriosis
Q
0
v Feedback End Block
N
U
u
DGNQU‘fiUNH
=~emmm
Question I“: 12086
.vm <
Previous
>
Next
G)
Tulon'al
ii
Lab Value.
a
Notes Calcdator
'
Revetse Color Text Zoom
$3
. Direct visualization & surgical biopsy
Exhibit Display
Pelvic endometriosis

Implants (bladder)
o
._.
H
,_.
H
N
w
H .—
.

A
H
U!
.

H
.

a
H
\l
.

H
.

a
H
u:
.

N
.

e
N
|-‘
.

N
.

N
N
u;
.

N
.

A
N
U!
.

N
OI
.

Implanls (bowel)
N
\l
.

Q Zoom In Q Zoom Out 9 Reset 517 Add To Flash Card


N
.

a
N
u:
.

u
.


o

Female Reproductive Sys‘em 8. Breast


0
w
|-‘
.

Endometriosis Feedback End Black


w
.

N
DGNQU‘fiUNH
o
._.
HH
H H
,_.
LION
=
_
Item 10 M40
Question Id: 12086 I ‘7 "a.“ Lab Vahles
3.
Nata Calculator
'
Revetse Color Text Zoom
$3
.

A
H
U!
.

H
.

a
H
\l
.

H
.

a
H
u:
.

N
.

e
N
|-‘
.

N
.

N
N
u;
.

N
.

A
N
U!
.

N
OI
.

N
\l
.

3 9'7 Add To Flash Card


N

Reset
.

a
N
u:
.

u
.


o

Female Reproductive Sys‘em 8. Breast


0
w
|-‘
.

Endometriosis Feedback End Black


w
.

N
DGNQU‘fiUNH
o
._.
H
,_.
H
N
.—
w Lab Value.
3.
Notes Calulator
'
Revetse Color Text Zoom
$3
u

Ih
H
U1
-

H
u

a!
H
\l
-

H
u

H
3
.

ND
H
u

N
6
.

N
H
u

N
N
0.!
.

N
u

N
A
U!
.

N
u

OI
N
\l
.

3 9‘7 Add To Flash Card


u

a Zoom In Q Zoom Out Reset


N
3
.

ND
N


.

u
0

.31 Female Reproductive Sys‘em 8. Breast


Endometriosis
0
v Feedback End Black
DmN‘QU‘fiUND—l
hem11of4-D
QUESHO" Id: “753
.VMark ®
Tutorial
it
Lab Values
Q
Notes Calculator
I
Reverse Color Text Zoom
ۤ3
A45—yearrold woman comes to the office for an annual well—woman visit. She feels well and has no concerns. The patient
takes a combined estrogen/progestin oral contraceptive and has regular monthly menses. She has no chronic medical
conditions and has had no prior surgeries. The patient had 2 term vaginal deliveries in her 305, and her last Pap smear with
human papillomavirus cotesting was normal 5 years ago. The patient does not use tobacco, alcohol, or illicit drugs. Her
last menstrual period started 2 days ago. Blood pressure is 110/60 mm Hg and pulse is 70lmin. BMI is 23 kglm2_ Pelvic
10 examination shows a multiparous cervix without lesions. Pap testing reveals benign-appearing endometrial cells and no
intraepithelial lesions. Which of the following is the best next step in management of this patient?

O A. Cenrical conization [0%]

O B. Colposcopy [5%]
15
16 O C. Endometn'al biopsy [20%]
17
O D. Pelvic ultrasound [4%]
18
19 VO E. Routine Pap testing [69%]
20
21
22
23
Omitted
cm III 69% 1Seoond _ o5m3r2020
24 E0 am” Answered correctly Tlme Spent E Last Updated
25
26
27 Explanation
28
29
30 Pap smear results requiring endometrial evaluation
31 Female Reproductive System 8. Breast
Endometrial cancer
Q
0
32 Feedback End Black
1 than 11 of“ G») ' €633
'
2 v ,
Question “1' “753
I \7 Mark <
Previous
D
Next Tutorial
ll
Lab Values
3
Notes Calculator
'
Reverse Color Text Zoom
3
4 Explanation

7 Pap smear results requmng endometrial evaluahon


I 6 u u u u

8 Result Group requiring endometrial sampling


9
1o - Premenopausal women with:
o Abnormal uterine bleeding 0R
Benign—appearing endometrial cells _ g .
0 Risk for endometnal hyperplasra
Postmenopausal women

Atypical glandular cells Women age 235 OR at risk for endometrial hyperplasia
15

- 16 Atypical glandular cells, favor neoplastic All women


17
. 18 This patient has benign-appearing endometrial cells on Pap testing Pap test reporting varies by age:
19
. In women age <45, endometrial cells are not reported on Pap test results because this is a common, benign finding,
. 20
21 particulany during the first 10 days of the menstrual cycle (as in this patient).

. 22 - In women age 245, endometrial cells are reported because this finding is more concerning for endometrial

23 hyperplasia or cancer, particularly in patients who are postmenopausal, symptomatic (ie, abnormal uterine bleeding),
' 24 or at high risk (eg, unopposed estrogen from obesity, chronic anovulation).
25
. 26 Although this patient is age 45, she is premenopausal (ie, regular monthly menses), asymptomatic (ie, no abnormal
27 uterine bleeding), and has no risk factors (eg, normal BMI). In addition, she is taking a combined estrogen/progestin oral
- 28 contraceptive, which decreases the risk of endometrial hyperplasia and cancer (Choice C). Therefore, she does not require

29 additional evaluation (eg, endometrial biopsy) and can continue routine Pap testing.
. 30 '
31 Female Reproductive System a Breast Q 0
. 32 . End°m91fial “"0“ Feedback End Block
1 ‘ Item 11 of40 '
. < D . €633
- 2 Question Id'' 14758 Previous Next Tutonal Lab Values Calculator Reverse Color Text Zoom
3 This patient has benign-appearing endometrial cells on Pap testing. Pap test reporting varies by age:
. 4
5 - In women age <45, endometrial cells are not reported on Pap test results because this is a common, benign finding,
_ 6 particularly during the first 10 days of the menstrual cycle (as in this patient).

7 . In women age 245, endometn'al cells are reported because this finding is more concerning for endometrial
8 hyperplasia or cancer, particularty in patients who are postmenopausal, symptomatic (ie, abnormal uterine bleeding),
9 _ _ _ _ _
10 or at high risk (eg, unopposed estrogen from obesrty, chronic anovulation).

Although this patient is age 45, she is premenopausal (ie, regular monthly menses), asymptomatic (ie, no abnormal
uterine bleeding), and has no risk factors (eg, norrnal BMI). In addition, she is taking a combined estrogen/progestin oral
contraceptive, which decreases the risk of endometrial hyperplasia and cancer (Choice 0). Therefore, she does not require
additional evaluation (eg, endometrial biopsy) and can continue routine Pap testing.
15
. 16 (Choices A and B) Cervical conization and colposcopy are used to evaluate for cervical cancer in patients with either
17 cervical intraepithelial lesions on Pap testing or gross abnormalities (eg, fungating lesion) of the cervix, which are not seen
. 18 in this patient.

. :2 (Choice D) Pelvic ultrasound can be performed in patients with postmenopausal bleeding to assess the risk of endometrial
21 cancer (ie, as an alternative to endometrial biopsy). However, ultrasound is not used to assess the risk of cancer in
. 22 asymptomatic or premenopausal patients, such as this one.

. :3; Educational objective:


25 Endometn'al cells on Pap testing are a normal finding in premenopausal women, particularly within the first 10 days of the
. 26 menstrual cycle. These patients can continue routine Pap testing and do not require additional evaluation (eg, endometrial
, 27 biopsy) unless they are symptomatic (ie, abnormal uterine bleeding) or at high risk for endometrial hyperplasia and cancer
. 28 (eg, obesity).
29
. 30 .
31 Female Reproductive System 8. Breast Q 0
v
. 32 _ Endometnal cancer Feedback End Black
E Ice-m1 om IV'M 4 > ® 5! 5‘ ' Q3
DGNQU‘fiUNH Question I“: 14753 Previous Next Tulon'al Lab Values Nata Calulator Revetse Color Text Zoom
Atypical glandular cells - Women age 235 OR at risk for endometrlal hyperplasla
Exhibit Display

Cervical cone biopsy

.10

. 15

. 16

. 17

. 18

. 19

. 20

. 21

. 22

. 23

. 24

. 25

. 26

n 27 W

' 23 Q Zoom Out 3 Reset 9’7 Add To Flash Card


. 29

. 30 ’
. 31 Female Reproductive Sys‘em 8. Breast Q 0
. 32 . E"°°"'9‘”a' “W“ Feedback End Black
E Ice-m1 om IV'M 4 > ® 5! 5‘ ' Q3
DGNQU‘fiUNH Question I“: 14753 Previous Next Tulon'al Lab Values Nata Calulator Revetse Color Text Zoom
Atypical glandular cells - Women age 235 OR at risk for endometrlal hyperplasla
Exhibit Display

Colposcopy

.10

. 15

. 16
. 17

. 18
. 19

. 20
. 21

. 22
. 23

. 24
. 25

. 26
. 27

' 23 com In Q Zoom Out 9 R858! 95 Add To Flash Card


. 29

. 30
. 31 Female Reproductive Sys‘em 8. Breast Q 0
. 32 . E"°°"'9‘”a' “W“ Feedback End Black
DRNQMfiUNt—t
E loem120f40
QUESfiOfl I“: 479'
.ek <
pmvioug
D
Next
Q)
Tutorial
ii
Lab Values
5.
Notes Calculator
I
Reverse Color Text Zoom
ۤ3
IThe following vignette applies to the next 2 items. The items in the set must be answered in sequential order. Once yil

A26-year~old woman comes to the office due to a vulvar lesion that she noticed 2 days ago. The lesion began as a papule,
but this morning the patient noticed a small ulcer in the center. No pain or discharge is present. She has mild burning with
urination but no fever, other skin lesions, or oral ulcers. The patient is sexually active and has had 5 male partners over the
NH:
HHH

past 5 years. She uses oral contraceptives to prevent pregnancy. The patient regularly uses tobacco, alcohol, and illicit
drugs. \fital signs are within normal limits. Vulvar examination shows a 2-cm ulcer with a nonexudative base and a raised,
indurated margin. There is no tenderness with palpation. Moderate, painless bilateral inguinal lymphadenopathy is present.

Item 1 of2

Which of the following is the most likely diagnosis in this patient?

O A. Behcet syndrome [0%]

O B. Chancroid[8%]

O C. Granuloma inguinale [12%]

O D. Herpes genitalis [4%]

V0 E. Syphilis [72%]

O F. Vulvar carcinoma [1%]

Owned 0
Female Reproductive System 8. Breast
Syphilis
Q
O
Feedback End Block
<> I
DRNQMfiUNl—l Previous Next Tutorial Lab Value; Notes Calculator Reverse Color Text Zoom
The following vignette applies to the next 2 items. The items in the set must be answered in sequential order. Once you
click Proceed to Next Item, you will not be able to add or change an answer.

A26-year~old woman comes to the office due to a vulvar lesion that she noticed 2 days ago. The lesion began as a papule,
but this moming the patient noticed a small ulcer in the center. No pain or discharge is present. She has mild burning with
urination but no fever, other skin lesions, or oral ulcers. The patient is sexually active and has had 5 male partners over the
past 5 years. She uses oral contraceptives to prevent pregnancy. The patient regularly uses tobacco, alcohol, and illicit
rut—Ac
HHH

drugs. Vital signs are within normal limits. Vulvar examination shows a 2—cm ulcer with a nonexudative base and a raised,
indurated margin. There is no tenderness with palpation. Moderate, painless bilateral inguinal lymphadenopathy is present.

Item1of2

Which of the following is the most likely diagnosis in this patient?

O A. Behcet syndrome [0%]

O B. Chancroid [8%]

O C. Granuloma inguinale [12%]

O D. Herpes genitalis [4%]

V0 E. Syphilis [72%]

O F. Vulvar carcinoma [1%]

Omitted
I" 72% 3Seconds — 01/20/2020
Conect answer
Answered correctly Time Spent E Last Updated
E

Female Reproductive System 8. Breast


Syphilis Ea Feedback
0
End Block
1 Exhibit Display
2
Characterlsflas 07 ulceraflve sexually rammed diseases
3
4
5
6
7
8
9

Q Zoom Out I? Add To Flash Card


IPMark < D‘ C?) ii 3 ' @233
DRNQMfiUND—l Question I“: 4791 Previous Next Tutorial Lab Values Notes Calculator Reverse Color Text Zoom
Characteristics of ulcerative sexually transmitted diseases

Causative
Disease Features of primary lesion
agent

Multiple 8. deep ulcers


Base may have gray to yellow
NH:
HHH

Haemophilus exudate
Chancloid _
ducreyl Organisms ofien clump in long
parallel strands ("school of
fish“)

Multiple, small, grouped


ulcers
Herpes Shallow with erythematous
Genital herpes simplex virus base
1 8' 2 Multinucleated giant cells &
intranuclear inclusions
(Cowdry type A)

Extensive & progressive


ulcerative lesions without
lymphadenopathy
Granuloma
IOebsiella a Base may have granulation—
inguinale
granulomatl's like tissue
(donovanosis)
Deeply staining gram—negative
intracytoplasmic cysts
(Donovan bodies)

Female Reproductive System 8. Breast Q 0


Syphilis Feedback End Black
IVMark < D Q) ii 3 ' €333
DRNQMfiUND—l Question Id: 4791 Previous Next Tutorial Lab Values Notes Calculator Reverse Color Text Zoom

0 Single, indurated, well—


circumscn'bed ulcer
Clean base
. Thin, delicate, corkscrew—
shaped organisms on
dark—field microscopy
NH:
HHH

Small & shallow ulcers


Large, painful, coalesced
Lymphogranuloma Chlamydia Fgflggslxpph nodes
venereum trachomatis
Intracytoplasmic chlamydial
inclusion bodies in epithelial
cells 8. leukocytes

OUWadd

Patients with multiple sexual partners and substance abuse are at high risk for sexually transmitted infections. This patient
has classic symptoms of primary syphilis — bilateral inguinal lymphadenopathy and a painless genital chancre. Syphilitic
chancres form at the site of direct inoculation with Treponema pallidum. After exposure (3430 days), patients develop a
single papule that tums into a shallow, painless, nonexudative ulcer with indurated edges. Chancres are exceedingly
infectious, with rates of Tpallidum transmission as high as 30%. Most chancres resolve spontaneously in 6—8 weeks (if
untreated), but the systemic spread of Tpallidum results in continued infection.

(Choices A, B, and D) Behcet syndrome, chancroid, and herpes genitalis are usually characten'zed by multiple, painful
genital lesions (not a single, painless lesion). Chancroid, caused by Haemophilus ducreyi, is most commonly found in
developing regions but is rare in the United States. Behget syndrome is thought to be a vasculitis and is marked by

Female Reproductive System a Breast Q 0


Syphilis Feedback End Black
DRNQMfiUNt—t
_...\>Ma.t
Question “1- 479‘
<I>
Previous Next Tutorial
nie'IJQB
Lab Values Notes Calculator Reverse Color Text Zoom

©UWorld

Patients with multiple sexual partners and substance abuse are at high risk for sexually transmitted infections. This patient
has classic symptoms of primary syphilis - bilateral inguinal lymphadenopathy and a painless genital chancre. Syphilitic
chancres form at the site of direct inoculation with Treponema pallidum. After exposure (3430 days), patients develop a
single papule that turns into a shallow, painless, nonexudative ulcer with indurated edges. Chancres are exceedingly
infectious, with rates of Tpallidum transmission as high as 30%. Most chancres resolve spontaneously in 6—8 weeks (if
NH:
HHH

untreated), but the systemic spread of Tpallidum results in continued infection.

(Choices A, B, and D) Behcet syndrome, chancroid, and herpes genitalis are usually characterized by multiple, painful
genital lesions (not a single, painless lesion). Chancroid, caused by Haemophilus ducreyi, is most commonly found in
developing regions but is rare in the United States. Behget syndrome is thought to be a vasculitis and is marked by
recurrent oral aphthous ulcers, often with significant systemic manifestations (including genital ulcers).

(Choice C) Granuloma inguinale is caused by Klebsiella granufomat‘is and is marked by the formation of extensive,
progressive, and painless genital ulcers, usually without lymphadenopathy. Granuloma inguinale is seen primarily in India,
Guyana, and New Guinea. In the United States, <100 infections occur annually, most in patients who have traveled to
these countries.

(Choice F) Vulvar carcinoma usually presents with a vulvar plaque, ulcer, or mass, often with significant pruritus. Vulvar
cancer would evolve over weeks or months (not 2 days).

Educational objective:
Primary syphilis is marked by the formation of a painless chancre that begins as a papule and converts into a nonexudative
ulcer with indurated borders. Mild to moderate bilateral lymphadenopathy is often present.

31 Female Reproductive System 8. Breast Q 0


. Syphilis Feedback End Block
ORNQMfiUND—l QUESHO" Id: 4792
IVMark ®
Tutorial
5‘
Lab Values
E
Notes Calculator
'
Reverse Color Text Zoom
ۤ3
Item 2 of2

Serum rapid plasma reagin and herpes simplex virus polymerase chain reaction testing are negative. Which of the
following is the best next step in management of this patient?

O A. Biopsy and culture of the ulcer [28%]

O B. Empinc acyclovir [0%]


Hl-l
HO
u

O C. Empinc cefln‘axone and azithromycin [8%]


.

VO D. Empiric penicillin [60%]


H
.

O E. Follow-up in 2 weeks to ensure resolution [1%]


Ul
h-I
H
at
H
\l
H
u

(:0d III 60% ZSeoonds _ 01/202020


H

D° am‘ Answered oonectly Time Spent E Last Updated


ND
N
u

6
N
l—‘
N
u

Explanation
N
DJ
N
u

uh
N
Ul

Syphilis - diagnostic serology


N
u

at
N
\l

. Antibody to cardiolipinMolesterol-ledthin antigen


N
u

Nontreponemal - Quantitative (titers)


N
ND

(RPR, VDRL) . Possible negative result in early infection


u

Ea
.

Female Reproductive System 8. Breast


0
w
l—‘

Syphilis Feedback End Block


N
U
u
hem13of40 < I). Q) ii g ' €633
ORNQMfiUNt—t Question I“: 4792 Previous Next Tutorial Lab Values Notes Calculator Reverse Color Text Zoom

Explanation

Syphilis - diagnostic serology

Antibody to cardiolipin—cholesteml—lecithin antigen


Nontreponemal Quantitative (titers)
Hl-l
HO

(RPR, VDRL) Possible negative result in early infection


u

Decrease in titers confirms treatment


.

Antibody to treponemal antigens


H

Treponemal
.

Qualitative (reactivelnonreacfive)
(Fl'A—ABS,
Ul
h-I

Greater sensitivity in earty infection


TP—EIA)
H
at

Positive even atter treatment


H
\l
H
u

Fl'A—ABS = fluorescent treponemal antibody absorption; RPR = rapid plasma reagn;


3

TP—EIA = Treponema pallidum enzyme immunoassay.


H
ND

0 World
N
u

6
N

This patient with a painless genital ulcer and bilateral inguinal lymphadenopathy almost certainly has primary syphilis.
l—‘
N
u

However, initial syphilis screening serology with rapid plasma reagin (RPR) was negative. Two types of serologic tests
N
N
DJ

are used in combination to diagnose syphilis:


N
u

uh

- Nontreponemal (eg, RPR, VDRL)


N
Ul
N

- Treponemal (eg, fluorescent treponemal antibody absorption)


u

at
N
\l

Although either type may be used to screen for syphilis, nontreponemal tests may have higher false—negative rates
N
u

(20%-30%) in patients with primary syphilis.


N
ND
u
.

Female Reproductive System a Breast Q B 0


w
l—‘

Syphilis Feedback End Block


N
U
u
ltem13 M40 '
ORNQMfiUNt—t Question “1:4792 Previous Next Tutorial Lab Values Notes Calculator Reverse Color Text Zoom
This patient with a painless genital ulcer and bilateral inguinal lymphadenopathy almost certainly has primary syphilis.
However, initial syphilis screening serology with rapid plasma reagin (RPR) was negative. Two types of serologic tests
are used in combination to diagnose syphilis:

- Nontreponemal (eg, RPR, VDRL)


. Treponemal (eg, fluorescent treponemal antibody absorption)

Although either type may be used to screen for syphilis, nontreponemal tests may have higher false-negative rates
Hl-l
HO

(20%-30%) in patients with primary syphilis.


u

Patients with negative screening serology and strong clinical evidence of primary syphilis (eg, chancre) should be treated
.

empirically with intramuscular benzathine penicillin G as this reduces the risk of transmission. In these patients, repeat
H

nontreponemal serology should be done in 2-4 weeks to establish baseline titers; a 4-fold titer decrease at 6-12 months
.

b
Ul
h-I

would confirm adequate treatment.


H
or

(Choice A) Treponema pallidum cannot be cultured, so biopsy is unhelpful. Awaiting biopsy results also delays treatment.
H
\l
H
u

(Choice B) Acyclovir is used to treat genital herpes simplex virus (HSV) outbreaks. HSV lesions tend to be multiple,
H
ND

vesicular, and painful; this patient has a single, painless lesion with negative HSV polymerase chain reaction results.
N
u

6
N

(Choice 6) Asingle dose of ceflriaxone plus azithromycin is used in the treatment of gonorrheal urethrifis. Neissen'a
l—‘
N
u

gonon'hoeae usually causes dysuria and a purulent urethral discharge, not a single, painless ulcer.
N
N
DJ

(Choice E) Treatment should not be delayed as the risk of transmission with syphililic chancres is as high as 30% and
N
u

uh

patients may not return for treatment (as chancres spontaneously resolve).
N
Ul
N
u

at

Educational objective:
N
\l

Syphilis is diagnosed with a combination of nontreponemal (eg, rapid plasma reagin) and treponemal serology. False—
N
u

negative nontreponemal serology is common in primary syphilis. Patients with negative initial serologies and strong clinical
N
ND

evidence of rima s hilis eo, chancre should be treated emoiricall with intramuscular benzathine enicillin G.
u

Ea
.

Female Reproductive System a Breast


0
w
l—‘

Syphilis Feedback End Block


N
U
u
ORNQMfiUND—l QUESHO" Id: 15945
IVMark ®
Tutorial
ii
Lab Values
3
Notes Calculator
'
Reverse Color Text Zoom
ۤ3
A 19—yearrold woman comes to the office due to continuous vaginal discharge. The patient has had a clear, slightly
malodorous vaginal discharge for the past few weeks but no pelvic pain, dysuria, or vulvovaginal pruritus. She is 6 weeks
postpartum from a vaginal delivery of a stillborn male infant alter 3 days of labor and 4 hours of pushing. The patient had
no prenatal care and delivered at home. Temperature is 36.7 C (96 F), blood pressure is 120/60 mm Hg, and pulse is
TDImin. Pelvic examination shows a well—healed, third degree perineal laceration. 0n speculum examination, lhere is a
small, red area of granulation tissue on the anterior vaginal wall. The cervix has no lesions and there is a pool of clear fluid
O
r.-

11 in the vagina with a pH of 6. Which of the following would most likely establish the diagnosis in this patient?

V O A. Bladder dye test [57%]

O B. Cenlical nucleic acid amplification test [5%]

16 O C. Urinalysis and urine culture [4%]


17
O D. Viral culture ofthe lesion [2%]
18
19 O E. Wet mount microscopy [30%]
20
2l
22
23
Omitted III 57% 1 Second _ 02/24/2020
24
20m “"5”” Answered correctly Tlme Spent E Last Updated
25
26
27 Explanation
28
29
30
Bladder dye test
3l Female Reproductive System 8. Breast
_ Urinary incontinence
Q
0
32 Feedback End Block
IP’Mark < D Q) il 3 ' €533
ORNQMfiUNl—t QUESHO" “'3 15945 Previous Next Tutorial Lab Values Notes Calculator Reverse Color Text Zoom

Explanation

Bladder dye test

‘2?
H
,_.

/ J;
3/1»
16 y ‘ t. ,»
17 Catheter—" g\ L' "
' 18 Tampon
. 19 fi
. 20 . Vesicovaginal Blue dye
21 J Syringe fistula on tampon
' (With dye)
. 22 oUWorId

23 This patient has a vesicovaginal fistula (WF), a complication of obstmctied labor that is common in resource-limited
' 24 areas (eg, sub-Saharan Africa) due to young maternal age (ie, small pelvis) and limited or no prenatal care, which results in
:: delayed diagnosis and labor intervention. Obstructed labor is the most common cause of WF wondwide with an estimated
27 >100,000 new cases yearly.

. 28 In these patients, WF develops because excessive fetal head compression during obstructed labor causes injury and

29 necrosis to the maternal vagina, rectum, and bladder. Tissue necrosis leads to erosion and fistula development between
. 30
31 Female Reproductive System a Breast Q B O
. 32 “"1“"! i"°°“‘i"°"°° Feedback End Block
ORNQMfiUND—l QUESHO“ I“: 15945
I \7 Mark ®
Tutorial
ii
Lab Values
6-
Notes Calculator
'
Reverse Color Text Zoom
£233
In these patients, WF develops because excessive fetal head compression during obstructed labor causes injury and
necrosis to the maternal vagina, rectum, and bladder. Tissue necrosis leads to erosion and fistula development between
proximal structures (eg, vesiccvaginal, rectovaginal), typically occurring within the first weeks postpartum. Because of the
aberrant connection between the vagina and bladder, patients with WF have a continuous vaginal discharge with an
abnormally elevated pH (ie, >45) due to urine, which may be malodorous due to surrounding necrotic tissue. Pelvic
examination typically shows vaginal pooling of urine, a visible defect, or an area of raised, red granulation tissue on the
anterior vaginal wall.
O
o

t.-

11
Bladder dye testing is performed to confirm the diagnosis, particularly in patients who have small fistulas that are not
visualized on pelvic examination. During a bladder dye test, the bladder is filled with dyed fluid and patients are monitored
for vaginal leakage of dye via speculum examination or tampon placement. Treatment of WF is with surgical repair.

(Choice B) Cervical nucleic acid amplification testing for chlamydia and gonorrhea is indicated in acute cervicitis, which
16 typically presents with cervical friability and a mucopurulent cervical discharge, not seen in this patient.
17
. 1; (Choice 0) Urinalysis and urine culture are used to diagnose urinary tract infection, which is common in postpartum
19 patients. This patient has no dysuria or suprapubic pain; therefore, these tests are not indicated.
. 20
21 (Choice D) Viral cutture is used to diagnose herpes simplex virus. Patient; typically have multiple ulcers or vesicles, not a
. 22 solitary raised, red area of granulation tissue.

23 (Choice E) Bacterial vaginosis can be diagnosed by using wet mount microscopy; it causes a thin, malodorous (fishy)
' 2" discharge with an elevated pH (eg, >4.5). Bacterial vaginosis does not cause vaginal pooling or granulation tissue, making
:: this diagnosis unlikely.

27 Educational objective:
. 28 Obstructed labor in resource-limited areas is the most common cause of vesiccvaginal fistula worldwide. Vesicovaginal

29 fistula presents with a continuous, watery vaginal discharge and an area of raised, red granulation tissue on the anterior
. 30 .
31 Female Reproductive System a Breast Q B O
. 32 ”"1“"! i"°°'“i"°"°° Feedback End Black
1 ' '
. I Mark ll 5
' 2 Question I“: 15945 V nal Lab Values Notes Calculator Reverse Color Text Zoom fig;
3 (Choice B) Cervical nucleic acid amplification testing for chlamydia and gonorrhea is indicated in acute cervicitis, which ‘
. 4
5 typically presents with cervical friability and a mucopurulent cervical discharge, not seen in this patient.

- 6 (Choice C) Urinalysis and urine culture are used to diagnose urinary tract infection, which is common in postpartum
7 patients. This patient has no dysuria or suprapubic pain; therefore, these tests are not indicated.
. s
9 (Choice D) Viral culture is used to diagnose herpes simplex virus. Patients typically have multiple ulcers or vesicles, not a
_ 1o solitary raised, red area of granulation tissue.
11
(Choice E) Bacterial vaginosis can be diagnosed by using wet mount microscopy; it causes a thin, malodorous (fishy)
discharge with an elevated pH (eg, >4.5). Bacterial vaginosis does not cause vaginal pooling or granulation tissue, making
this diagnosis unlikely.

Educational objective:
16 Obstructed labor in resource—limited areas is the most common cause of vesicovaginal fistula worldwide. Vesicovaginal
17 fistula presents with a continuous, watery vaginal discharge and an area of raised, red granulation tissue on the anterior
. 1s _ _ . . . . . .
19 vaginal wall. Dragnosrs rs With pelvrc examination and bladder dye test

. 20
21 References
. 22 _ _ _ . . . . .
23 - Obstetric fistulae In southern Mozambique: rncrdence, obstetric characteristics and treatment.

- 24 - Female voiding dysfunction and urinary incontinence.


25
26 - Out of the shadows and 6000 reasons to celebrate: an update from FIGO's fistula surgery training initiative.

27 - Social experiences of women with obstetric fistula seeking treatment in Kampala, Uganda.
. 28
29
. 30
31 Female Reproductive System 8. Breast Q B O
. 32 . WWW “WNW”? Feedback End Block
ORNQU‘fiUNH
6
g.-
Item 14 of 40
Question Id: 15946 Tulon'al Lab Value.
3. '
Revetse Color Text Zoom
$3

11

16
17
18
19
20
2l
22
23
24
25
26
27
28
a Zoom In Q Zoom Out 9 Reset 517 Add To Flash Card
29
30
3l
32 Urinary incontinence E» Feedback
0
End Black
ORNQMfiUNt—l QUESHO" Id: 13059
IVMark ®
Tutorial
il
Lab Values
3
Notes Calculator
'
Reverse Color Text Zoom
ۤ3
An 88—year—old woman is brought to the emergency department due to postmenopausal bleeding. The patient has had
bright red vaginal bleeding and pelvic pain for the past week. She lives in a nursing home and was brought by her
caregiver, who states that she is also refusing to bathe or leave her room. Vital signs are normal. BMI is 36 kglmz. The
abdomen is soft and nontender. The vulva is edematous and tender and there is a friable posterior perineal laceration. The
vagina is dry and pale, and the cenlix is stenotjc. The remainder of the physical examination is unremarkable. On
ultrasound, the uterus is small with a 3-mm endometrial lining, the ovaries are not visualized, and no adnexal masses are
.

O
H

present. Which of the following is the best next step in management of this patient?
H
H
H...
can

O A. Endometrial biopsy [7%]

Q B. Pelvic MRI [0%]


H
b
.

Ur
.

O C. Pessary placement [0%]


at
.

H
\l
...

#0 D. Sexual abuse screening [77%]


t-l
an

O E. Topical corticosteroids [2%]


...
in
.

N
e

O F. Vulvar biopsy [12%]


l-‘
N
.

N
N
o;
N

Omitted
.

N
A

II 77% 186a — 02/12/2020


Correct answer I
Answered oonecu, Tlme Spent E Last Updated
U!
N
at
.

N
\l
N
.

N
a

Explanation
u:
N
O
N
.

31 Female Reproductive System 8. Breast


Sexual assault
Q
0
v Feedback End Block
N
U
u
1 -
mm 15 one I \7 Mark < D Q) .l g ' $33
2 Question I“: 18059 Previous Next Tutorial Lab Values Notes Calculator Reverse Color Text Zoom
3
4

5 Explanation
6 l
7
3 Elder abuse
9
1o - Advanced age (>80)

11 Risk factors ' woman


12 . Cognitive impairment (eg, dementia, depression)
13 . Physical impairment (eg, hip fracture, stroke)
H
b

Unexplained injuries/bruising at atypical locations (eg, trunk, thighs)

I
._.
U1

Clinical . Nonosteoporotic fractures (eg, spiral fractures of long bones)


H
at

17 findings . Signs of neglect (eg, malnutrition, pressure ulcers)


18 - Signs of sexual abuse (eg, anogenital trauma)
19
20 Management . Report to adult protective services immediately

21 This elderly woman has postmenopausal bleeding (PMB), which can occur due to a variety of etiologies but is most
22 concerning for endometrial cancer, particularly in obese patients (such as this one). Initial evaluation for endometrial cancer
23
24 is with pelvic ultrasonography, which in this patient revealed a normal endometrial thickness (ie, 54 mm), indicating a low
25 likelihood of cancer and therefore no additional need for endometrial biopsy (Choice A). At this point, other causes of PMB
26 require consideration.

27 In the setting of a friable perineal laceration and multiple risk factors (ie, age >30, female, nursing home resident), this
:: patient's PMB is concerning for possible elder sexual abuse. For multiple reasons (99, fear, cognitive impairment), elderly
30 patients do not typically report their abuse, Instead, they commonly present with somatic symptoms (eg, pelvic pain) and ,

31 Female Reproductive System a Breast Q 0


32 , Sexual assault Feedback End Block
1 A

-
2 “"95”“ "l: ”“59 . ‘7 "ark nal Labelluai his Calculator Revegcdor €633
3 ' anagemen 'epo o aou pro ec Ive servrces immeia e y

z This elderly woman has postmenopausal bleeding (PMB), which can occur due to a variety of etiologies but is most
6 concerning for endometrial cancer, particularly in obese patients (such as this one). Initial evaluation for endometrial cancer
7 is with pelvic ultrasonography, which in this patient revealed a normal endometrial thickness (ie, 54 mm), indicating a low
8 likelihood of cancer and therefore no additional need for endometrial biopsy (Choice A). At this point, other causes of PMB
9 require consideration.

it: In the setting of a friable perineal laceration and multiple risk factors (ie, age >80, female, nursing home resident), this
12 patient's PMB is concerning for possible elder sexual abuse. For multiple reasons (eg, fear, cognitive impairment), elderly
13 patients do not typically report their abuse. Instead, they commonly present with somatic symptoms (eg, pelvic pain) and
- 14 vaginal bleeding (due to trauma to atrophic vulvovaginal tissue). Other red flags include unexplained behavioral changes,
particulaiiy surrounding hygiene (eg, refusing to bathe), and signs of genital trauma (eg, perineal laceration, vulvar
' 16 edema).
17
13 Concerning findings should prompt sexual abuse screening and be reported immediately to adult protective services.

19 (Choice B) Pelvic MRI can be used to evaluate for causes of pelvic pain (eg, endometliosis) but is not a first-line test for
:2 PMB. In addition, further imaging is not necessary for this patient with a normal pelvic ultrasound.

22 (Choice C) A pessary can be used to treat pelvic organ prolapse (POP), which may present with PMB if prolapsed tissue
23 (eg, vaginal mucosa) encounters friction and becomes denuded or friable. However, patients with FOP typically have a
:: bulge or mass at the vaginal introitus, which is not seen in this patient.

25 (Choices E and F) Pen'neal lesions may be caused by vulvar lichen sclerosus, which is treated with topical corticosteroids,
27 or vulvar cancer, which requires vulvar biopsy for diagnosis. However, these conditions are typically preceded by months to
28 years of vulvar pruntis and plaque~like vulvar lesions, not acute—onset vulvar edema and tenderness, as seen in this patient.

:3 Educational objective: .

31 Female Reproductive System a Breast Q 0


32 _ Sexual assault Feedback End Black
ORNQMfiUNt—t QUESHO" I“: ”“59
IVMark ®
Tutorial
5‘
Lab Values
5
Notes Calculator
'
Reverse Color Text Zoom
€633
edema).

Concerning findings should prompt sexual abuse screening and be reported immediately to adult protective services.

(Choice B) Pelvic MRI can be used to evaluate for causes of pelvic pain (eg, endomeln‘osis) but is not a first—line test for
PMB. In addition, further imaging is not necessary for this patient with a normal pelvic ultrasound.

(Choice 8) A pessary can be used to treat pelvic organ prolapse (POP), which may present with PMB if prolapsed tissue
(eg, vaginal mucosa) encounters friction and becomes denuded or friable. However, patients with FOP typically have a
Hl-l
HO

bulge or mass at the vaginal introitus, which is not seen in this patient.
H
N

(Choices E and F) Perineal lesions may be caused by vulvar lichen sclerosus, which is treated with topical corticosteroids,
w
...

or vulvar cancer, which requires vulvar biopsy for diagnosis. However, these conditions are typically preceded by months to
H
b

years of vulvar pruritis and plaque—like vulvar lesions, not acute—onset vulvar edema and tenderness, as seen in this patient.
._.
U1
H
at

Educational objective:
ray-
mu

Sexual abuse of the elderty may present with postmenopausal bleeding, pelvic pain, behavioral changes, and signs of
genital trauma (eg, perineal laceration). Concerning findings should prompt sexual abuse screening and be reported
H
u:

20 immediately to adult protective services.


21
22
References
23
24 - ACOG committee opinion no. 568: elder abuse and women's health.
25
26 - The prevalence of elder abuse in institutional settings: a systematic review and meta-analysis.

27 - Elder abuse.
28
29
30
31 Female Reproductive System 8. Breast Q 0
32 . Sexual assault Feedback End Black
ORNQMfiUNl—l
Item 15 of 40
Question Id: 18059 I V "ark Tulon'al Lab Value.
9‘
Notes Calcdator
'
Revels-e Color Text Zoom
$3

Apical (0106M) DI’OIIIIOO common with p.89!!!


Hl—l
HO
H
w N
...
H
b
NNNNNNNNNNHHHHH
wwMQm-tI—‘ewwxlo‘m

(*1 Zoom In Q Zoom Out 9 Reset [-17 Add To Flash Card


O
N

31
v
Female Reproductive Syslem 8. Breast
Sexual assault E» Feedback
0
End Black
N
U
ORNQMfiUND—l
Item 15 of 40
Question Id: 18059
4 > il 9‘ an an '
Previous Next Tulon'al Lab Value. Nola Calcdator Reverse Color Text Zoom

Exhibit Display

Lichen sclerosus
.

O
H
.

H
H
H
Nw
...
b
.

H
U1
-

H
0‘
.

H
u

\l
H
a
.

H
-

H
'D
u

6
N
l—‘
.

N
u

N
N
DJ
.

Uchcn sclerosus
u

uh
N

Pale thin tissue 8. perianal


U1
.

thickening with fissures


u

a!
N
\l
.

a Zoom In Q Zoom Out 517 Add To Flash Card


u

9 Reset
3
N
.

ND
N
0


.

.31 Female Reproductive Syslem 8. Breast


Sexual assault
0
v Feedback End Black
ORNQMfiUNt—l QUESHO" Id: 3353
IVMark ®
Tutorial
ii
Lab Values
3
Notes Calculator
'
Reverse Color Text Zoom
ۤ3
A34—yearrold woman, gravida 1 para 1, comes to the office for infertility evaluation. She has been trying to conceive for the
past year, but her cycles have become increasingly irregular, with the last menstrual period more than 3 months ago.
Menses previously occurred every 27 days and lasted 4 days. The patient feels fatigued and has been waking up at night
due to feeling too warm. She has been married for 6 years and has a 4—year—old daughter who was delivered vaginally
without complications. The patient has hypothyroidism, for which she takes levothyroxine. She has no previous surgeries.
The patient smokes a pack of cigarettes a day but does not use alcohol or illicit drugs. Both of her parents have type 2
r-I
.

diabetes mellitus. BMI is 24 kglmz. \fital signs are normal. Pelvic examination shows normal external genitalia, a small
H
H

mobile uterus, and normal bilateral ovaries. TSH is normal and a pregnancy test is negative. Which of the following would
.

rd
N

most likely be seen in this patient?


...
u:
Ht—l
ura

GnRH FSH Estrogen


.

at
.

>-

OA i 1 l
xr
u—a
.

[15%]
t-l
an

«O B. r r l
...
in
.

N
e

[76%]
l-‘
N

O C. T Normal 1‘
.

N
N

[0%]
o;
N

O D. Normal Normal Normal


.

N
A
Ur
N

[3%]
or
.

O E. 1 1 T
\r
N

[4%]
.

N
a
u:
N
O
N
u

31 Female Reproductive System a Breast


Amenorrhea
Q
0
v Feedback End Black
N
U
u
1 . Mark <1 r> I'l E ' I‘ H Al
2 Question I“: 3368 IV Previous Next nal Lab Values Notes Calculator Reverse Color Text Zoom fig;
3
4 Explanation
5
6
7 GnRH FSH Estrogen
s
9 Hypothalamic hypogonadism J, J J,

10 Primary ovarian insufficiency T T J


11
12 Polycystic ovary syndrome 1‘ Normal 1‘

13 Normal ovulation Normal Normal Normal


14
15 Exogenous estrogen use J, J T
0‘
>-

Primary ovarian insufficiency (POI), a form of hypergonadotropic hypogonadism, is the cessation of ovarian function
H
NJ

at age <40. The condition is characterized by amenorrhea or oligomenorrhea and symptoms of decreased estrogen (eg,
1: hot flashes, fatigue). Initial presentation is with irregular menses or infertility. Patients typically have a history of an
20 autoimmune disorder (eg, hypothyroidism) or Turner syndrome. The decreased ovarian function results in low estrogen
21 levels; this prevents the physiologic negative feedback mechanism, thereby causing increased levels of GnRH and FSH
22 at the level of the hypothalamus and pituitary, respectively. Infertility treatment for POI consists of either in vitro fertilization
:: or oocytelembryo donation.

25 (Choice A) Hypothalamic hypogonadism is characterized by low GnRH secretion and resultant low levels of FSH and
26 estrogen. Typical presentation is also with oligomenorrhea or amenorrhea, but the condition is more likely in the setting of
27 relative calon'c insufficiency from decreased caloric intake (eg, eating disorders) or strenuous exercise. Patients with
2‘3 hypothalamic hypogonadism do not have associated menopausal symptoms.

:3 (Choice C) Polycystic ovary syndrome (PCOS) causes infertility by anovulation. Patients with PCOS have elevated GnRH, .
31 Female Reproductive System a Breast Q 0
32 _ Amenorrnea Feedback End Block
ORNQMfiUNt—t (109550“ Id: 3353
I \7 Mark <
Previous
D
Next
C?)
Tutorial
ii
Lab Values
3'
Notes Calculator
'
Reverse Color Text Zoom
$3
Primary ovarian insufficiency (POI), a form of hypergonadotropic hypogonadism, is the cessation of ovarian function
at age <40. The condition is characterized by amenorrhea or oligomenorrhea and symptoms of decreased estrogen (eg,
hot flashes, fatigue). Initial presentation is with irregular menses or infertility. Patients typically have a history of an
autoimmune disorder (99, hypothyroidism) or Turner syndrome. The decreased ovarian function results in low estrogen
levels; this prevents the physiologic negative feedback mechanism, thereby causing increased levels of GnRH and FSH
at the level of the hypothalamus and pituitary, respectively. Infertility treatment for POI consists of either in vitro fertilization
or oocytelembryo donation.
.

O
H
r-o
H

(Choice A) Hypothalamic hypogonadism is characterized by low GnRH secretion and resultant low levels of FSH and
.

rd
N

estrogen. Typical presentation is also with oligomenormea or amenorrhea, but the condition is more likely in the setting of
no
r—

relative caloric insufficiency from decreased caloric intake (eg, eating disorders) or strenuous exercise. Patients with
HH
ura

hypothalamic hypogonadism do not have associated menopausal symptoms.


.

a)
.

(Choice 0) Polycystic ovary syndrome (PCOS) causes infertility by anovulation. Patients with PCOS have elevated GnRH,
>-
\I
h-l
.

normal FSH, and elevated estrogen. In patients with PCOS, these occur due to increased peripheral conversion of
an
H

androgens to estrone. This leads to persistently elevated (is, continuous high-frequency pulses) GnRH levels at the
i--
in

hypothalamus, which causes a preference for LH production (and normal to decreased FSH levels) at the pituitary. These
.

N
e

patients typically have a history of irregular menses, obesity, and signs of hyperandrogenism (eg, hirsutism, acne). Physical
l-‘
N

examination may show bilaterally enlarged ovaries.


.

N
N
o;
N

(Choice D) Normal hormone levels would indicate ovulation, unlikely in a patient with irregular menses. Likely etiologies of
.

N
A

infertility with normal laboratory values and normal menses are tubal blockage or male factor.
Ur
N

(Choice E) Exogenous estrogen use decreases GnRH and FSH levels through negative feedback, thereby preventing
Or
.

N
\l

ovulation. This patient has signs of hypoestrogenism (eg, hot flashes), not hyperestrogenism (eg, breast tenderness).
N
.

N
a

Educational objective:
u:
N
.

u
o

Female Reproductive System a Breast


0
l—‘
w

Amenorrhea Feedback End Block


N
U
u
ORNQMfiUNt—t QUESfiOfl Id: 3353
IVMark ®
Tutorial
il
Lab Values
a
Notes Calculator
'
Reverse Color Text Zoom
$3
estrogen. Typical presentation is also with oligomenorrhea or amenorrhea, but the condition is more likely in the setting of
relative caloric insufficiency from decreased caloric intake (eg, eating disorders) or strenuous exercise. Patients with
hypothalamic hypogonadism do not have associated menopausal symptoms.

(Choice C) Polycystic ovary syndrome (PCOS) causes infertility by anovulation. Patients with PCOS have elevated GnRH,
normal FSH, and elevated estrogen. In patients with PCOS, these occur due to increased peripheral conversion of
androgens to estrone. This leads to persistently elevated (ie, continuous high—frequency pulses) GnRH levels at the
.

O
H

hypothalamus, which causes a preference for LH production (and normal to decreased FSH levels) at the pituitary. These
H
H

patients typically have a history of irregular menses, obesity, and signs of hyperandrogenism (eg, hirsutism, acne). Physical
.

rd
N

examination may show bilaterally enlarged ovaries.


...
u:

(Choice D) Normal hormone levels would indicate ovulation, unlikely in a patient with irregular menses. Likely etiologies of
HH
Uta
.

infertility with normal laboratory values and normal menses are tubal blockage or male factor.
at
.

>-

(Choice E) Exogenous estrogen use decreases GnRH and FSH levels through negative feedback, thereby preventing
\l
u—a
.

ovulation. This patient has signs of hypoestrogenism (eg, hot flashes), not hyperestrogenism (eg, breast tenderness).
t—l
an
...
in

Educational objective:
.

N
e

Primary ovarian insufficiency, cessation of ovarian function at age <40, may present with infertility, irregular menses, and
l-‘
N

menopausal symptoms. It is characterized by elevated gonadotropin-releasing hormone and FSH levels and a low
.

N
N

estrogen level.
o;
N
.

N
A
Ul
N

References
at
.

- Primary ovalian insufficiency.


\l
N
.

N
a
u:
N
O
N
u

31 Female Reproductive System 8. Breast


Amenorrhea
Q
0
v Feedback End Black
N
U
u
ORNQMfiUNt—l QUESHO" Id: “197
IVMark ®
Tutorial
ii
Lab Values
3
Notes Calculator
'
Reverse Color Text Zoom
ۤ3
A52—yearrold postmenopausal woman comes to the office for evaluation of vulvar irritation. The patient wears sanitary
napkins due to stress urinary incontinence and occasionally notices streaks of blood on the napkin. She is sexually active
and has had some pain with sexual intercourse. The patient underwent a hysterectomy and bilateral salpingo—
oophorectomy at age 48 for adenomyosis. She has had abnormal Pap tests previously but normal colposcopy results. The
patient currently uses a nicotine patch for smoking cessation. \fital signs are normal. Pelvic examination shows multiple
vulvar exconations with surrounding erythema. There is an erythematous, fn‘able plaque on the left Iabium majus. On
.

O
H

speculum examination, the vagina has no lesions or abnormal discharge. Which of the following is the most likely diagnosis
H
H

in this patient?
M
.

H
DJ
H

O A. Condylomata acuminata [5%]


HHH
OtUI-b

O B. Contact dermatitis [8%]


.

O C. Lichen planus [12%]


H
\l
.

H
.

O D. Lichen sclerosus [14%]


N ...
u:

V0 E. Vulvar cancer [48%]


a
N
l-‘

O F. Vulvovaginal atrophy [10%]


N
.

N
N
o;
N
.

A
N

Omitted
Ul

II 48% 1Second — 01/28/2020


N

Correct answer I
Answered onrrecth/ Tlme Spent E Last Updated
at
.

N
\l
N
.

a
N
u:

Explanation
O
N
u

31 Female Reproductive System 8. Breast


Vulvar carcinoma
Q
0
v Feedback End Block
N
U
u
ORNQMfiUND—t Question Id: 14197
lm <
Previous
D
Next
Q)
Tutorial
i!
Lab Values
a
Notes Calculator
'
Reverse Color Text Zoom
$3
Explanation

Vulvar cancer

- Persistent HPV infection


Etiolo
gy - Chronic inflammation
.

O
H
H
H

- Tobacco use
M
.

- Vulvarlichen sclerosus
DJ
H

Risk factors - Immunodeficiency


HHH
OtUl-b

- Prior cervical cancer


- Vulvar/cervical intraepithelial neoplasia
.

H
\l
.

- Vulvar pruritus
H
.

Clinical features - Vulvar plaque/ulcer


N i-l
v:

- Abnormal bleeding
a
N
l-‘

Diagnosis - Biopsy
N
.

N
N

HPV 2 human papillomavirus


o;
N
.

This patient's erythematous, friable plaque on the labium majus is concerning for vulvar squamous cell carcinoma. A risk
N
Ul
N

factor for the development of Vulvar cancer is persistent human papillomavirus (HPV) infection, particularly with types
at
.

16 and 18, which are also associated with cervical cancer (as seen in this patient's prior abnormal Pap tests) While most
\l
N

HPV infections are transient, patients with either chronic tobacco use or immunodeficiency (eg, HIV) are less likely to
.

a
N
u:

clear the infection, resulting in dysplastic changes.


O
N
u

31
v
Female Reproductive System a Breast
Vulvar carcinoma Ea Feedback
0
End Block
N
U
u
turn 17 of 40
IVMark <> a. '
ORNQMfiUNt—t
at: an
Question Id: 14197 Previous Next Tutorial Lab Values Notes Calculator Reverse Color

This patient‘s erythematous, friable plaque on the labium majus is concerning for vulvar squamous cell carcinoma. A risk
factor for the development of vulvar cancer is persistent human papillomavirus (HPV) infection, particularly with types
16 and 18, which are also associated with cervical cancer (as seen in this patient's prior abnormal Pap tests). While most
HPV infections are transient, patients with either chronic tobacco use or immunodeficiency (eg, HIV) are less likely to
clear the infection, resulting in dysplastic changes.

Constant dysplastic changes over the vulvar squamous cells can result in a unifocal, friable plaque or ulcer, typically on
.

O
H

the labia majora, that produces persistent vulvar irritation and/or pain. Patients may also have intermittent bleeding and
H
H

dyspareunia (as seen in this patient) or an asymptomatic lesion found on routine examination. Diagnosis is with vulvar
M
.

biopsy, which evaluates for depth of invasion and determines management options.
DJ
H

(Choice A) Patients with condylomata acuminata are typically asymptomatic or have mild vulvar pruritis. In contrast to this
t-li-It-l
OtUl-b

patient, condylomata acuminata typically present with multiple lesions that are fungated or dome-shaped.
.

(Choice B) Contact dermatitis may occur secondary to constant initation from sanitary napkins; however, it typically does
H
\l
.

not cause a discrete, friable plaque.


H
.

a
N ...
in

(Choice 8) Vulvar lichen planus typically presents with pruritic, purple-hued plaques that are sometimes associated with
a

thin, white stn'ae around the labia and vulva (ie, Wickham sln'ae).
N
l-‘
N

(Choice D) Vulvar lichen sclerosus lesions are typically multiple white papules that converge into plaques. A single lesion
.

N
N
u;

is uncommon.
N
.

(Choice F) Women with vulvovaginal atrophy typically have thinning of the skin and fusion of the labia majora, rather than
N
U!

a distinct, friable vulvar plaque.


N
at
.

N
\l

Educational objective:
N
.

Vulvar squamous cell carcinoma often occurs secondary to persistent human papillomavirus infection, which is associated
N
u:

with chronic tobacco use. Patients with vulvar cancer often have vulvar irritation, intermittent bleeding, and a unifocal,
O
N
u

31
v
Female Reproductive System a Breast
Vulvar carcinoma Ea Feedback
0
End Block
N
U
u
ORNQMfiUNt—t QUESHO" Id: “197
IVMark ®
Tutorial
ii
Lab Values
5
Notes Calculator
'
Reverse Color Text Zoom
$3
(Choice A) Patients with condylomata acuminata are typically asymptomatic or have mild vulvar pruritis. In contrast to this
patient, condylomata acuminata typically present with multiple lesions that are fungated or dome—shaped.

(Choice B) Contact dermatitis may occur secondary to constant irritation from sanitary napkins; however, it typically does
not cause a discrete, friable plaque.

(Choice C) Vulvar lichen planus typically presents with pruritic, purple-hued plaques that are sometimes associated with
thin, white striae around the labia and vulva (ie, Wickham striae).
.

O
H

(Choice D) Vulvar lichen sclerosus lesions are typically multiple white papules that converge into plaques. A single lesion
H
HM
.

is uncommon.
H
DJ
H

(Choice F) Women with vulvovaginal atrophy typically have thinning of the skin and fusion of the labia majora, rather than
HHt-l
OiUl-b

a distinct, friable vulvar plaque.


.

Educational objective:
H
\l
.

Vulvar squamous cell carcinoma often occurs secondary to persistent human papillomavirus infection, which is associated
H
.

with chronic tobacco use. Patients with vulvar cancer often have vulvar irritation, intermittent bleeding, and a unifocal,
N ...
in

friable mass commonly located on the labia majora.


a
N
l-‘
N
.

References
N
N
o;

- Genital cancers in women: vulvar cancer.


N
.

A
N
Ul

- Key concepts in management of vulvar cancer.


N
a.
.

- Vulvar cancer: two pathways with different localization and prognosis.


\l
N
.

a
N
u:
O
N
u

31 Female Reproductive System 8. Breast


v Vulvar carcinoma
Q
0
Feedback End Black
N
U
u
ORNQU‘fiUNH
_ Item 17 of 40
Question Id: 1419? Lab Vaha
9.
Notes Calulator
'
Revetse Color Text Zoom
$3
.

O
H
H
HM
.

H
DJ
H
HHH
OUI-b
.

H
\l
.

H
.

a
N ...
u:
a
N
|-‘
N
.

N
N
u;
N
.

A
N
U!
N
OI
.

N
\l
N
.

a
N
u:
u
.


o

Female Reproductive Sys‘em 8. Breast


0
w
H

Vulvar carcinoma Feedback End Black


N
U
u
hern180f40 .VMark ® 5] g I
ORNQMfiUNl—l QUESHO" Id: 15302 Tutorial Lab Values Notes Calculator Reverse Color

A54—yearrold woman, gravida 2 para 2, comes to the office for evaluation of leakage of urine. Over the past 2 years, the
patient has had increasingly frequent involuntary loss of urine and now wears a sanitary napkin to prevent urine from
dribbling on her clothes after she voids. For the past week, she has had burning with urination but no frequency or
urgency. In the last year, the patient has had 3 urinary tract infections that were treated with antibiotics. The patient had 2
term vaginal deliveries in her 305. She has no chronic medical conditions and has had no prior surgeries. Blood pressure
is 118/64 mm Hg and pulse is 32/min. BMI is 33 kg/m? On pelvic examination, the patient does not leak urine with Valsalva
r-I
.

and the vagina is dry and atrophic. A 2-cm tender anterior vaginal mass is palpable and causes expression of a bloody
H
H

discharge at the urethral meatus. Postvoid residual is 50 mL. Which of the following is the most likely cause of this
M
.

patient's symptoms?
DJ
H
.
H
A

O A. Abnormal tract between the bladder and vagina [20%]


ur
h-l
or
H

O B. Acute infection of the lower urinary tract [4%]


\l
...
.

O C. Hypermobility of the urethra and bladder neck [6%]


.

an
>-
...
u:
.

O D. Impaired contractility of the bladder detrusor [4%]


N
o
l—‘

V O E. Localized outpouching of urethral mucosa [64%]


N
.
N
N
u;
N
.
N
A

Omitted
I" 64% 1Seoond — 02/03/2020
ur
N

Correct answer
Answered oonectty Tlme Spent E Last Updated
or
.
N
\l
N
as
.
N

Explanation
u:
N
.
u
o

31 Female Reproductive System 8. Breast


_ Urinary incontinence
Q
0
Feedback End Block
N
U
u
1 hem 1a of 40 < D
, Mark I1 E p
-A u A
2 Question I“: 15802 IV Previous Next nal Lab Values Notes Calculator Reverse Color Text Zoom fig;
3
4 Explanation
5
6
7 Urethral diverticulum
s
9 _ _ . Urethral mucosa herniated
Definition , _ ,
1o Into surrounding tissue

11 . Dysuria
12
_ _ . Postvoid dribbling
13 Clinical features ,
14 - Dyspareunia
15 - Anterior vaginal wall mass

16 - Urinalysis
’ 17 _ _ _ - Urine culture
Diagnostic testing _
19 - MRI of the pelvrs
20 . Transvaginal ultrasound

:: This patient has a urethral diverticulum, an abnormal localized urethral mucosa due to recurrent periurethral gland
23 infection along the anterior vaginal wall, The recurrent infection and inflammation of the urethral tissue creates the
24 abnormal outpouching that can collect and store urine, resulting in postvoid dribbling and recurrent lower urinary tract
25 infections (eg, dysuria). Infection of the urethral diverticulum can also result in tenderness, often presenting as dyspareunia
26 or a tender antierior vaginal wall mass with an associated expressed purulent or bloody urethral discharge Diagnosis
27 is confirmed with a pelvic MRI, and treatment is via surgical excision of the diverticulum.
28
29 (Choice A) Vesicovaginal fistula, an abnormal tract between the bladder and vagina, typically presents with constant,
30 rather than intermittent, leakage of urine due to continuous drainage of the bladder into the vagina. In addition, there is no .
31 Female Reproductive System a Breast Q 0
32 . U'inary WWW“! Feedback End Black
lem1sor4o .Pmm < > ® it a. I
ORNQMfiUNt—t QUESHO“ I“: 15802 Previous Next Tutorial Lab Values Notes Calculator Reverse Color
This patient has a urethral diverticulum, an abnormal localized urethral mucosa due to recurrent periurethral gland
infection along the anterior vaginal wall. The recurrent infection and inflammation of the urethral tissue creates the
abnormal outpouching that can collect and store urine, resulting in postvoid dribbling and recurrent lower urinary tract
infections (eg, dysuria). Infection of the urethral diverticulum can also result in tenderness, often presenting as dyspareunia
or a tender anterior vaginal wall mass with an associated expressed purulent or bloody urethral discharge. Diagnosis
is confirmed with a pelvic MRI, and treatment is via surgical excision of the diverticulum.

(Choice A) Vesicovaginal fistula, an abnormal tract between the bladder and vagina, typically presents with constant,
.

O
H
H
H

rather than intermittent, leakage of urine due to continuous drainage of the bladder into the vagina. In addition, there is no
M
.

associated anterior vaginal wall mass or purulent urethral discharge.


r-n
DJ

(Choice B) Acute infection of the lower urinary tract (ie, cystitis, urethiitis) can cause dysuria and urinary incontinence;
n—i
.

however, patients often also have urgency, frequency, and no tender anterior vaginal wall mass.
ur
h-l
at
H

(Choice C) Stress urinary incontinence, caused by hypennobility of the urethra and bladder neck, is common in
\l
...
.

multiparous and obese patients, such as this one. Patients typically have urinary leakage with increased intraabdominal
.

as
>-

pressure (eg, Valsalva), which is not seen in this case.


...
.

to
N
o

(Choice D) Overflow incontinence, which is due to impaired bladder detrusor oontractility, can occur in postmenopausal
l—‘
N

women. Patients typically have constant dribbling of urine from incomplete bladder emptying and chronic urinary retention
.

N
N

(ie, overflow). This diagnosis is unlikely because this patient has a normal postvoid residual (ie, no retention).
u;
N

Educational objective:
.

N
A
Ur

A urethral diverticulum is an abnormal localized outpouching of the urethral mucosa that can collect urine, resulting in
N
or
.

inflammation of the sunounding tissue. Patients often exhibit dysuria, postvoid dribbling, dyspareunia, and a tender anterior
\l
N

vaginal wall mass that expresses a purulent or bloody urethral discharge.


.

N
a
u:
N
.

u
o

Female Reproductive System 8. Breast Q


0
l-‘
w

Urinary incontinence Feedback End Block


.

w
N
ORNQU‘fiUNH
we“
0095110" I“: 15002
4
Previous
>
Next
G)
Tulon'al
ii
Lab Value.
9‘
Notes Calcdator
'
Reverse Color Text Zoom
@
Diagnostic testing
I I I ‘ I ‘

Ea-mibit Display
Urethral diverticulum
Hl-l
HO
HH
LON
HHHI—l
worm-h
m
H
wwwNNNNNNNNNNh-I
Ni—‘owwMaIUI-hwiui—‘ou:

Q Zoom In Q Zoom Out 9 Reset [:7 Add To Flash Card

inflammation of the surroundino tissue. Patients often exhibit d suria, ostvoid dribblino d sareunia and a tender anterior
Female Reproductive Sys‘em 8. Breast Q 0
Urinary incontinence Feedback End Black
ORNQU‘fiUNH
hem1aur4n
Question Id: 15802

Previous
> C») $‘I-3'€§3 Text Zoom
Next Lab Value. Notes Calcdator Reveise Color
Diagnostic testing
E;hibit Display

Vesicovaginal fistula
i-I
.

O
H
H
M
.

H
DJ
H
.

H
A
U!
h-l
OI
H
xi
...
.
.

ca
>-
...
.

in
N
o
i—‘
N
.

N
N
u;
N
.

N
A
U!
N
OI
.

Vagina
\i
N

a Zoom In Q Zoom Out S Resei ‘% Add To Flash Card


.

N
a
u:
N

@, Cl 0
inflammation of the surroundino tissue. Patients often exhibit d suria, ostvoid dribblin d sareunia and a tender anterior
.

u
o

Female Reproductive System 8. Breast


|-‘
w

Urinary incontinence Feedback End Black


.

w
N
hem190f40 IVMark ® it Q I
ORNQMfiUNt—l QUESHO" Id: 1235 Tutorial Lab Values Notes Calculator Reverse Color

A48—yearrold woman comes to the clinic due to dyspareunia, which has worsened overthe last 4 months. Five years ago,
the patient undenNent radiation and chemotherapy for stage II cervical cancer. She has not been sexually active for several
years due to fear of cervical cancer recurrence. Now the patient has a new partner and is using over—the-counter lubricants
during intercourse but still experiences pain with penetration. She has had no postcoital bleeding, dysuria, or abnormal
vaginal discharge. The patient has a 20—pack—year smoking history. She has not had a menstrual period since completing
radiation therapy. Vital signs are normal. Examination shows thin vulvar skin with shrinkage of clitoral tissue. The vaginal
.

O
H

introitus is narrow, and the vaginal mucosa appears pale. The cervix appears scarred consistent with postradiation changes
H
H

but has no raised, nodular, or friable lesions. Which of the following is the most likely cause of this patient's condition?
M
.

H
DJ
H

«’0 A. Atrophic vaginilis [76%]


H
.

uh
H
Ul

O B. Lichen planus [2%]


H
at

O C. Lichen sclerosus [15%]


H
\l
H
.

O D. Recurrent cervical cancer [0%]


.

NNNNNNNNNNH
qQmAwNHo-o

O E. Vulvar intraepithelial neoplasia [0%]


.

O F. Vulvodynia [5%]
.

Omitted II 76% 1Second — 01/21/2020


Correct answer I
Answered mnecm Tlme Spent E Last Updated
.
.

Explanation
O
N
.

31 Female Reproductive System 8. Breast


_ Dyspareunia
Q
0
Feedback End Black
N
U
u
hem 19 of 40
lm <ii> ® il 3 '
$3
ORNQU‘fiUNt—t Question Id: 12055 Previous Next Tutorial Lab Values Notes Calculate r Reverse Color Text Zoom

Explanation

Genitourinary syndrome of menopause

Vulvovaginal dryness, irritation, pruritus


Dyspareunia
.

O
H

Symptoms Vaginal bleeding


H
HM
.

- Urinary incontinence, recurrent urinary tract infection


H
DJ
H

- Pelvic pressure
H
.

uh
H

- Narrowed introitus
U!
H

- Pale mucosa, 1 elasticity, 1 rugae


at

Physical examination
H
\l

- Petechiae, fissures
H
.

- Loss of labial volume


G
.

wwNNNNNNNNNNH
HowwummtHo-o

Vaginal moisturizer & lubricant


.

Treatment
Topical vaginal estrogen

This patient has atrophic vaginitis (or the genitourinary syndrome of menopause) from medically—induced menopause due
to radiation and chemotherapy treatment for cervical cancer resulting in ovarian follicle depletion. Estrogen maintains the
.

moisture, blood flow, and collagen content (eg, elasticity, turgor) of the vulvovaginal tissues (eg, vagina, vulva, urethra).
Patients with low estrogen levels eventually develop dryness and decreased blood flow and elasticity in these
.

estrogen—dependent tissues
.

Symptoms of estrogen deficiency are due to thinning of the vulvar skin (99, irritation), narrowing of the vaginal introitus
.

Dyspareunia
0
Feedback End Black
N
U
.
Inem19 M40 '
ORNQMfiUNl—t Question Id: 12055 Previous Next Tutorial Lab Values Calculator Reverse Color
This patient has atrophic vaginitis (or the genitourinary syndrome of menopause) from medically—induced menopause due
to radiation and chemotherapy treatment for cervical cancer resulting in ovarian follicle depletion. Estrogen maintains the
moisture, blood flow, and collagen content (eg, elasticity, tu rgor) of the vulvovaginal tissues (eg, vagina, vulva, urethra).
Patients with low estrogen levels eventually develop dryness and decreased blood flow and elasticity in these
estrogen—dependent tissues.

Symptoms of estrogen deficiency are due to thinning of the vulvar skin (eg, irritation), narrowing of the vaginal introitus
(eg, dyspareunia), and loss of natural lubrication (eg, dryness). Physical examination findings include pale, easily-
.

O
H

denuded, retracted, atrophic vulvovaginal epithelium (eg, clitoral shrinkage). First-line treatment includes over-the-
H
H
M
.

counter lubricants or moisturizers. Persistent or severe symptoms are treated with vaginal estrogen.
H
DJ
H

(Choice B) Lichen planus is a chronic, inflammatory skin dystrophy that results in glazed, brightly erythematous, vulvar
H
.

uh

lesions with a purple hue that may be overlaid by white reticular lines (ie, Wickham striae).
H
Ul
H
at

(Choice C) Patients with lichen sclerosus have thin, wrinkled vulvar skin that can form into thickened white plaques that
H
\l

eventually obliterate the labia majora and minora, scarring the normal external landmarks. In contrast to atrophic vaginitis,
H
.

lichen sclerosus does not affect the vagina.


.

NNNNNNNNNNH
qQmAwNHo-o

(Choice D) Recurrent cervical cancer typically presents with abnormal bleeding from raised, friable cervical nodules (not
.

seen in this patient).

(Choice E) Vulvar intraepithelial neoplasia (VIN) creates asymptomatic or pruritic vulvar lesions, particularly in smokers.
VIN typically appears as raised, multifocal vulvar lesions (eg, white, erythematous, hyperpigmented), which are not seen in
.

this patient.
.

(Choice F) Vulvodynia causes dyspareunia due to a sharp, burning pain on the vulvar vestibule often triggered by touch
(eg, positive Q-tip test). Patients may have vestibular erythema but no associated vaginal tissue narrowing or clitoral tissue
.

shrinkage.
O
N

Ea
.

31 Female Reproductive System a Breast


_ Dyspareunia
0
Feedback End Block
N
U
u
Imm190f40 ® 5‘ g '
ORNQMfiUNt—t Question Id: ”“55 Tutorial Lab Values Notes Calculator Reverse Color

(Choice D) Recurrent cervical cancer typically presents with abnormal bleeding from raised, friable cervical nodules (not
seen in this patient).

(Choice E) Vulvar intraepithelial neoplasia (VIN) creates asymptomatic or pruritic vulvar lesions, particularly in smokers.
VIN typically appears as raised, multifocal vulvar lesions (eg, white, erythematous, hyperpigmented), which are not seen in
this patient.
.

O
H

(Choice F) Vulvodynia causes dyspareunia due to a sharp, burning pain on the vulvar vestibule oflen triggered by touch
H
H

(eg, positive Q—tip test). Patients may have vestibular erythema but no associated vaginal tissue narrowing or clitoral tissue
M
.

shrinkage
DJ
H
H

Educational objective:
.

uh
H
Ul

Atrophic vaginitis (genitourinary syndrome of menopause) is due to low estrogen levels, which cause decreased
H
at

vulvovaginal tissue elasticity and blood flow. Patients may have resultant dyspareunia due to narrowing of the vaginal
H
\l

introitus and dryness from loss of natural lubrication. Treatment is with lubricants or moisturizers; vaginal estrogen is used
H
.

for persistent or severe symptoms.


.

NNNNNNNNNNH
qQmAwNHo-o
.

References

- Vulvar and vaginal atrophy: physiology, clinical presentation, and treatment considerations.
.

- Genitourinary syndrome of menopause: new terminology for vulvovaginal atrophy from the International Society for the
Study of Women's Sexual Health and the North American Menopause Society.
.

- ACOG practice bulletin no. 141: management of menopausal symptoms.


.

O
N
.

31 Female Reproductive System 8. Breast


_ Dyspareunia
Q
0
Feedback End Block
N
U
u
ORNQMfiUNl—l
ltem190f40
Question Id: 1205 lm < > 5| 5‘ an on '
Previous Next Tulmial Lab Values Nata Calulator Reverse Color Text Zoom

This atient has atro hic va-initis or the oenitourina 5 ndrome of menoause from medicall —induced menoause due
Exhibit Display

Lichen sclerosus
.

O
H
.

H
H
H
N
.

H
w
u

I.
H
U1
-

H
Cl
.

H
u

\l
H
.

H
G
.

H
'0
.

N
O
u

l—‘
N
.

N
N
DJ
-

Uchcn sclerosus
u

uh
N

Pale thin tissue 8. perianal


U1
.

mickening with fissures


u

a!
N
\l
.

a Zoom In Q Zoom Out 517 Add To Flash Card


u

9 Reset
N
3
.

ND
N

Ea
.

u
0

.31 Female Reproductive Sysrem 8. Breast


_ Dyspareunia
0
Feedback End Black
hem 20 one I ‘7 Mark ® it Q I
ORNQMfiU-INH QUESHO" Id: 12320 Tutorial Lab Values Notes Calculator Reverse Color

A42—yearrold woman, gravida 2 para 2, comes to the office for evaluation of pelvic pain. The patient's menses are painful,
with heavy bleeding that requires her to change her tampon every hour during the first 2 days. She did not have painful
menstrual periods until a few years ago. The pelvic pain used to subside after menses but has become constant over the
past few months and is unrelieved by ibuprofen. Her menstrual cycles are regular, occur every 28—30 days, and last 4
days. She has had no dysuria, urinary frequency, or constipation. The patient had a tubal ligation after her last delivery.
She has not had cervical cancer screening within the past 5 years and is not sexually active. Temperature is 36.7 C (98 F),
.

O
H

blood pressure is 120/70 mm Hg, and pulse is 78/min. Examination shows a boggy uterus that is tender to palpation.
H
H

Which of the following additional findings is most likely present in this patient?
M
.

H
DJ
H

O A. Exophytic cervical mass [1%]


.
H
A
Ul
H

O B. Large, irregularly shaped uterus [23%]


at
.
H

O C. Mucopumlent cenrical discharge [1%]


\l
h-I
an
H

O D. Posterior cuI-de-sac nodularity [8%]


N h-I
.

to

V O E. Symmetlically enlarged uterus [63%]


o
.

N
l—‘
.

O F. Unilateral adnexal mass [1%]


N
N
N
cu
N
A
.

Omitted
Ul

C m I" 63% 1 Second — 03/23/2020


N

E0 “"5“” Answered correctly Tlme Spent E Last Updated


at
.

N
\l
N
a
.

N
u:

Explanation
O
N
u

31 Female Reproductive System 8. Breast


_ Dysmettorrttea
Q
0
Feedback End Black
N
U
u
Exhibit Display

Nomal uterus vs adonomyosis

CUM

Q Zoom In Q Zoom om 3 Reset 9‘7 Add To Flash Card


ii 3 '
ORNQMfiUNt—l QUESHO" Id: 12320 Tutorial Lab Values Notes Calculator Reverse Color

This patient's chronic pelvic pain and heavy menstrual bleeding are most likely due to adenomyosis, a condition in which
endometrial glands and stroma accumulate abnormally within the uterine myometrium. Adenomyosis typically occurs in
multiparous women age >40 and is characterized by new—onset dysmenorrhea due to the cyclic shedding of the
endometrium within the myometrium. The continued accumulation of endometrial tissue within the myometrium causes an
increase in the endometrial cavity surface area (resulting in heavy menstrual bleeding) and progression to chronic pelvic
pain. The entrapped endometrial tissue within the uterine myometrium results in a boggy, tender uterus on examination;
r-I
.

it also induces myometrium hypertrophy, which causes a concentric or symmetrically enlarged uterus.
H
H
M
.

The initial workup of suspected adenomyosis consists of pelvic ultrasonography and/or MRI. A definitive diagnosis is made
DJ
H

histologically after hysterectomy, which is also the treatment for patients who do not improve with conservative management
.
H
A

(eg, oral contraceptives, progestin—releasing intrauterine device).


Ur
H
at
.
H

(Choice A) An exophytic cervical mass due to cervical cancer may cause heavy vaginal bleeding; however, the bleeding is
\l
l-l

typically irregular rather than cyclic as with menstruation.


an
H

(Choice B) An enlarged, irregularty shaped uterus is common with uterine leiomyomata (fibroids). Fibroids can cause
N l-l
.

to

heavy menstrual bleeding; however, the uterus is typically firrn (not boggy) and nontender.
o
.

N
l—‘
.

(Choice 6) Pelvic inflammatory disease can cause chronic pelvic pain and mucopurulent cervical discharge It does not
N
N

cause heavy menstrual bleeding and is unlikely in patients who are not sexually active.
N
u;
N
A
.

(Choices D and F) Posterior cul—de—sac nodularity and adnexal masses (ie, endometriomas) can occur in women with
N
Ur

endometriosis due to endometrial implants outside the uterus. This condition typically presents with dysmenon’nea and
N
at
.

chronic pelvic pain in younger women (age 25-35) Because endometriosis causes endometrial implants outside (rather
N
\r

than within) the uterine musculature, it does not cause a boggy uterus.
N
a
.

N
u:

Educational objective:
O
N
u

31 Female Reproductive System a Breast


_ Dysmenorrhea
Q
0
Feedback End Block
N
U
u
ORNQMfiUNt—l QUESHO" Id: 1N2” Tutorial
it
Lab Values
a
Notes Calculator
'
Reverse Color Text Zoom
$3
histologically after hysterectomy, which is also the treatment for patients who do not improve with conservative management
(eg, oral contraceptives, progestimreleasing intrauterine device).

(Choice A) An exophytic cervical mass due to cervical cancer may cause heavy vaginal bleeding; however, the bleeding is
typically irregular rather than cyclic as with menstmatjon.

(Choice B) An enlarged, irregularly shaped uterus is common with uterine leiomyomata (fibroids). Fibroids can cause
heavy menstrual bleeding; however, the uterus is typically firm (not boggy) and nontender.
r-I
.

O
H
H

(Choice 6) Pelvic inflammatory disease can cause chronic pelvic pain and mucopurulent cervical discharge. It does not
M
.

cause heavy menstrual bleeding and is unlikely in patients who are not sexually active.
DJ
H

(Choices D and F) Posterior cul—de—sac nodularily and adnexal masses (ie, endometriomas) can occur in women with
.
H
A
Ur
H

endometJiosis due to endometrial implants outside the uterus. This condition typically presents with dysmenonhea and
at
.
H

chronic pelvic pain in younger women (age 25-35). Because endometliosis causes endometrial implants outside (rather
\l
u—a

than within) the uterine musculature, it does not cause a boggy uterus.
t—l
an

Educational objective:
N u—a
.

in
o
.

Adenomyosis typically presents in women age >40 and is characterized by dysmenoniiea; heavy menstrual bleeding;
N
l—‘
.

progressive chronic pelvic pain; and a boggy, tender, symmetrically enlarged uterus.
N
N
N
u;

References
N
A
.

N
Ur

- Adenomyosis: a clinical review of a challenging gynecologic condition.


N
at
.

- Adenomyosis and abnormal uterine bleeding (AUB—A)—pathogenesis, diagnosis, and management.


\l
N
a
.

N
u:
O
N
u

31 Female Reproductive System 8. Breast


_ Dysmenorrhea
Q
0
Feedback End Block
N
U
u
E hemzoarw ® 5! ”5
ORNQMfiUNl—l Question I“: 12020
' €633
Lab Value. Nata Reverse Color Text Zoom
II I H V '-11 v'

Progression of cervical cancer


Eady siege
Hl-l
HO
H
N
H
w
l-lh-Il-lh-‘l-l
“MOW-h

28 Q Zoom In a Zoom Out 3 917 Add To Flash Card

31 Female Reproductive Sysrem 8. Breast % O


32 _ Dysmenorrhea Feedback End Black
E hemzoarw ® 5! ”5 '
ORNQU‘fiUNH Question Id: 12020 Notes Revetse Color Text Zoom
€633
II I H V 'I
Hl-l
HO
H
N
H
w
HHI—lh-‘I-l
“MOW-h

19 Immutal

Stbmuoosal

27 oumu
2’ Q Zoom In Q Zoom Out 9 Reset 517 Add To Flash Card

31 Female Reproductive Sys‘em 8. Breast % O


32 _ Dysmenorrhea Feedback End Black
ORNQU‘fiUNH
= Item 20 of 40
Question Id: 12020
5!
Notes
'
Revetse Color Text Zoom
$3
I! I " v. -

Pelvic endometriosis

Implants (flower)

. 10
. 11

- 12
. 13

. 14
. 15

. 16
. 17

. 18
. 19

. 21

. 22
. 23

. 24
. 25

. 26
Implanls (bowel)
. 27

. 28
a Zoom In Q Zoom Out 9 @658! 517 Add To Flash Card
. 29

. 30
.

.
31

32
Female Reproductive Sys‘em 8. Breast
_ Dysmenorrhea E» Feedback
0
End Black
ORNQMfiUNt—l QUESHO" I“: 4542
IVMark ®
Tutorial
5‘
Lab Values
g
Notes Calculator
'
Reverse Color Text Zoom
ۤ3

A 42—yearrold nulligravid woman comes to the office with intermittent blood staining the left side of her bra. She is extremely
worried because her 66«year-old mother was recently diagnosed with breast cancer The patient performs monthly breast
self-examinations and has not felt any lumps. She has a history of type 2 diabetes for which she takes insulin and
metformin. The patient does not smoke cigarettes and only drinks alcohol on rare social occasions. Physical examination
shows no palpable breast masses. Mammography shows no masses or calcifications. Which of the following is the most
likely diagnosis?
.

O
H

0A.
H
H

Ductal carcinoma in situ [5%]


M
.

B. Fat necrosis [1%]


DJ
H
H
.

Fibroadenoma [0%]
Turner)
H
Ul
H
at

Fibrocystic changes of the breast [1%]


.

H
\l

Inflammatory carcinoma [1%]


H
G

@0000
N ...
‘0

Intraductal papilloma [82%]


O

. Lobular breast carcinoma [0%]


IL?)
massessusmi
N
N

_ Paget disease of the breast [5%]

Pituitary adenoma [0%]

I" 82% 1 Second _ 04/23/2020


COWBOY answer
Answered correctly Time Spent Last Updated
F

Female Reproductive System 8. Breast


Intraductal papilloma
0
Feedback End Block
ORNQMfiUNt—t
than 21 of 40
Question I“: 4542
®
Tutorial
5‘
Lab Values
g
Notes Calculator
'
Reverse Color Text Zoom
$3
Explanation

Unilateral bloody nipple discharge


No associated mass or lymphadenopathy
.

O
H
H
H

o Mammography 8. ultrasound
Management
M
.

. Biopsy, +f— excision


DJ
H

OLlWorld
H
.

A
H
U1

Pathologic nipple discharge is typically unilateral, bloody, or serous and may be accompanied by additional breast
H
at
.

abnormalities. The most common cause of unilateral bloody discharge without a coexisting breast mass is an intraductal
H
\l

papilloma. Typically, this benign condition is nonpalpable on clinical breast examination due to the small size of the
H
G

papilloma inside the duct, Although this patient's family history of breast cancer increases her risk of breast malignancy,
H
‘0

normal examination and negative imaging findings are reassuring and confirm the diagnosis.
N
.

O
N

Diagnostic workup for pathologic (eg, unilateral and/or bloody) nipple discharge should begin with mammography to mle
.

H
N
.

out carcinoma even without the presence of a palpable breast mass. With small intraductal papillomas, mammography is
N
W

often normal, Ultrasound is also indicated for evaluation of ductal pathology and may demonstrate a dilated duct due to the
N
3

space—occupying papilloma.
N
U1
N
u

at

(Choice A) Although intraductal papillomas may contain foci of ductal carcinoma in situ, this condition typically presents as
N
\l

microcalcifications on mammography, which are absent in this patient.


N
u

(Choice B) Fat necrosis ofthe breast is a benign condition usually due to previous trauma and typically presents as a firm,
N
ND
u
u

Female Reproductive System a Breast Q


0
w
l—‘

intraductal papilloma Feedback End Block


N
U
u
ORNQMfiUNt—l QUESHO" Id: 4542
IVMark ®
Tutorial
ii
Lab Values
3
Notes Calculator
'
Reverse Color Text Zoom
$3
Pathologic nipple discharge is typically unilateral, bloody, or serous and may be accompanied by additional breast
abnormalities. The most common cause of unilateral bloody discharge without a coexisting breast mass is an intraductal
papilloma. Typically, this benign condition is nonpalpable on clinical breast examination due to the small size of the
papilloma inside the duct. Although this patient's family history of breast cancer increases her risk of breast malignancy,
normal examination and negative imaging findings are reassuring and confirm the diagnosis,

Diagnostic workup for pathologic (eg, unilateral and/or bloody) nipple discharge should begin with mammography to rule
.

O
H

out carcinoma even without the presence of a palpable breast mass. With small intraductal papillomas, mammography is
H
H

often normal. Ultrasound is also indicated for evaluation of ductal pathology and may demonstrate a dilated duct due to the
M
.

space—occupying papilloma.
DJ
H
H
.

(Choice A) Although intraductal papillomas may contain foci of ductal carcinoma in situ, this condition typically presents as
H
U1

microcalcifications on mammography, which are absent in this patient.


H
at
.

(Choice B) Fat necrosis of the breast is a benign condition usually due to previous trauma and typically presents as a firm,
\l
H

irregularty shaped mass with findings of oil cysts on mammography. Nipple discharge is not typical.
G
H
‘0

(Choice C) Afibroadenoma presents as a solitary, painless, firm, and mobile breast mass averaging 2 cm in size, and
N
.

would be noted on clinical breast examination and mammography. It is not associated with nipple discharge.
N
.

H
N
.

(Choice D) Fibrocystic changes of the breast are very common in premenopausat patients. Classic presentation is a
N
W

patient with cyclic bilateral breast pain (eg, mastalgia) and diffuse nodularity on breast examination. Nipple discharge is not
N
3

typical but, if present, is nonbloody (eg, physiologic). This condition is unlikely in this patient without palpable breast
N
U1

masses and mastalgia.


N
u

at
N
\l

(Choice E) Diffuse breast erythema with edema and peau d'orange appearance are the hallmark features of inflammatory
N
u

breast cancer (IBC). Although nipple discharge may be present with inflammatory breast cancer, this patient has no other
N
ND

breast abnormalities on examination and no imaging evidence of malignancy (eg, mass, calcification, parenchymal
u
u

Female Reproductive System a Breast Q


0
w
l—‘

_ Intraductal papilloma Feedback End Block


N
U
u
ORNQMfiUNt—t Tutorial
il
Lab Values
5
Notes Calculator
'
Reverse Color Text Zoom
$3
breast cancer (IBC). Although nipple discharge may be present with inflammatory breast cancer, this patient has no other
breast abnormalities on examination and no imaging evidence of malignancy (eg, mass, calcification, parenchymal
distortion) on mammography.

(Choice G) Lobular breast carcinoma typically presents as a fixed palpable breast mass with irregular borders and can
often be bilateral. Normal breast examination and negative mammography rule out this condition due to lack of a lesion or
calcifications.
.

O
H

(Choice H) Paget disease of the breast is a form of ductal carcinoma that pnmanly presents with eczematous nipple
H
H
M
.

changes (eg, flaking, crusting skin) that also include the areola. This condition can also have bloody nipple discharge but is
H
DJ
H

unlikely in this patient without skin or nipple changes in the context of normal mammography.
H
.

A
H

(Choice I) Hyperprolactinemia caused by a pituitary adenoma would cause bilateral galactorrhea (eg, milky discharge)
U1
H

rather than unilateral bloody discharge. Headaches and visual changes signal a large lesion, and MRI may be indicated.
at
.

H
\l

Educational objective:
H
G

Unilateral bloody nipple discharge is always pathologic. Bloody discharge without a corresponding breast mass or nipple
H
‘0

changes in the setting of normal mammography is the classic presentation of intraductal papilloma.
N
.

O
N
.

H
N
.

References
N
N
W

- Management of nipple discharge and the associated imaging findings.


N
3
N
U1

- Clinician's guide to imaging and pathologic findings in benign breast disease.


N
u

at

- Papilloma on core biopsy: excision vs. observation.


N
\l
N
u

3
N
ND
u
u

Female Reproductive System 8. Breast Q


0
w
l—‘

_ Intraductal papilloma Feedback End Block


N
U
u
ORNQU‘fiUNH
=
_
Item 21 of 40
Question Id: 4542 I V "ark

Previous Next Tulon'al Lab Value.
9‘
Notes Calcdator
'
Revetse Color Text Zoom
$3
Mammo rah & ultrasound
Exhibit Display

Intraductal papilloma
.

O
H
.

H
H
H
N
.

H
w
H
.

A
H
U!
.

H
.

a
H
\l
.

H
.

a
H
u:
.

N
.

a
N
.

H
N
.

N
N
u;
.

N
.

A
N
U!
.

N
OI
.

WWI!
\l
.

517 Add To Flash Card


N

a Zoom In Q Zoom Out 9 Reset


.

a
N
u:
.

u
.


o

Female Reproductive Sys‘em 8. Breast


0
w
|-‘
.

_ Intraductal papilloma Feedback End Black


w
.

N
ORNQU‘fiUNH
anon
Lab Values Nata Calulator Reverse Color
(Choice A) Although intraductal papillomas may contain foci of ductal carcinoma in situ, this condition typically presents as
Exhibit Display
Inflammatory breast cancer
Hl-l
G
H
HH
LON
NHHHHHH
O‘DGMOUIA

lil
hJ
hJ
hJ
RdP.Ni—‘ONDGNUIUIAWN

Rah-acted mu
ha
nJ

3 Reset 9’7 Add To Flash Card


ha
nJ
u;

Female Reproductive System 8. Breast % O


Lu

. '"mdum' Papilloma Feedback End Black


u;
ORNQU‘fiUNH
Dunn
Lab Values Nata Calulator Reveise Color
(Choice A) Although intraductal papillomas may contain foci of ductal carcinoma in situ, this condition typically presents as
Exhibit Display

Mammary Puget disease


.

H
G
.

H
H
N
p.
.

UJ
pa
H
.

A
H
U!
.

H
a!
.

H
\l
.

H
.

a
H
u:
.

N
.

a
N
.

H
N
.

N
N
u;
.

N
.

A
N
U!
.

N
OI
.

N
\l
.

9 Reset 517 Add To Flash Card


N
.

a
N
u:
.

u
.

Ea
o

Female Reproductive System 8. Breast


0
w
|-‘
.

_ lntraducul papilloma Feedback End Black


w
.

N
them 22 one I ‘7 Mark ® it Q I
ORNQMfiUNt—l QUESHO" Id: 12994 Tutorial Lab Values Notes Calculator Reverse Color

A23—yearrold woman, gravida 1 para 1, comes to the office due to pain in the right lower quadrant for 2 months. The pain is
unrelieved by acetaminophen and worsens with physical activity. Menses are regular and occur every 28 days. The patient
has a history of severe dysmenorrhea and takes oral contraceptive pills daily. Two years ago, she was treated for pelvic
inflammatory disease. She has no chronic medical conditions and has had no surgeries. The patient smokes a pack of
cigarettes a day but does not use alcohol or illicit drugs. Temperature is 36.7 C (98.1 F), blood pressure is 126/74 mm Hg,
and pulse is 871mm. BMI is 36 kg/mz. The abdomen is soft and nontender. Pelvic examination reveals right adnexal
.

O
H

fullness but is limited by habitus. Pregnancy test is negative. Pelvic ultrasonography shows a 4-cm ovarian mass. The
H
H

patient undergoes laparoscopic ovarian cystectomy, and a photograph of the cyst is shown in the exhibit. Which of the
M
.

following is the most likely diagnosis?


DJ
H
.

uh
H

O A. Corpus luteum cyst [0%]


U1
H
.

H
0

O B. Ectopic pregnancy [0%]


\l
H

O C _ Endometn'oma[0%]
.

H
3
H
in

D . Mature cystic teratoma [97%]


N
O
l-‘
.

E. Mucinous cystadenoma [1%]


N
.

wwNNNNNNNN
Howwuomaww

F. Tube—ovarian abscess [0%]


.

G _ Yolk sac tumor [0%]


.

COI'Tlltleld I 97% 7 Seconds


.

— 02l15l2020
Done“ answer Answered correctly Time Spent E L31“ Updated
.

Female Reproductive System 8. Breast


ovarian cancer
Q
0
Feedback End Block
N
U
.
Item 22 of 40
ORNQU‘fiUNH Question Vahles Notes Revetse Color Text Zoom
Hl-l
HO
HH
LON
UUUNNNNNNNNNNHHHHHH
NHowwMa‘mawMD—‘owwuomh

Female Reproductive Sys‘em 8. Breast


Ovarian cancer E» Feedback
0
End Black
hem220f40 .lk ® il
ORNQMfiUND—l QUESHO" Id: 1299‘ Tutorial Lab Values Notes Calculator Reverse Color

Explanation

. 10
. 11
- 12
. 13
. 14
. 15
. 16
. 17
. 18
. 19
. 20
. 21
22
. 23
. 24
. 25
. 26
. 27
. 28
. 29

@, CI 0
. 30 '
. 31 Female Reproductive System I Bleast
. 32 _ Ovarian cancer Feedback End Black
[mm 22 0f 40 < D. '
ORNQMfiUNt—t QUESHO“ I“: Previous Next Tutorial Lab Values Notes Calculator Reverse Color
This patient has a mature cystic teratoma (ie, dermoid cyst), a benign ovarian germ cell tumor that typically occurs in
young women. Patients with teratomas are usually asymptomatic and may be diagnosed incidentally (eg, adnexal fullness)
on pelvic examination. Because teratomas have variable mass densities, they are prone to ovarian torsion because they
have an intrinsically unstable suspension across the infundibulopelvic ligaments, which contain the ovarian vessels. As a
result, sudden movement may cause intermittent ovarian vessel occlusion, leading to intermittent ovarian torsion, as
seen in this patient‘s pelvic pain that worsens with physical activity.

Teratomas typically appear as adnexal masses with multiple calcifications on ultrasound but are definitively diagnosed after
.

O
H

ovarian cystectomy, which preserves fertility and reduces the risk of torsion and malignant transformation. Teratomas
H
H
M
.

have a gross appearance consistent with the 3 germ cell layers, including ectoderrnal elements such as thick, yellow
DJ
H

sebaceous fluid and hair (arrow); mesoderrnal (eg, cartilage, adipose tissue) and endoderrnal (eg, thyroid) components
.

uh
H

may also be seen.


U1
H

(Choice A) Acorpus luteum cyst is a physiologic remnant of a ruptured follicle afler ovulation; it has a yellow appearance
.

H
0
\l

from granulosa cell lipids and pigment. This diagnosis is unlikely because this patient is on oral contraceptives (ie,
H
.

H
3

anovulatory) and corpus luteum cysts do not contain hair.


H
‘D

(Choice B) Pelvic inflammatory disease and tobacco use are risk factors for ectopic pregnancy, which grossly appears as
N
O

fetal tissue (orange arrow) implanted within a dilated fallopian tube (red arrow) in patients with a positive pregnancy test.
l-‘
.

N
N
.

(Choice C) Endometriomas may cause severe dysmenorrhea and adnexal fullness; however, this diagnosis is unlikely
N
.

because endometriomas are a collection of old blood that creates a "chocolate cyst" appearance.
N
a
N
Ur

(Choice E) Mucinous cystadenomas are benign surface epithelial ovarian masses that appear as thin-walled loculations of
N
at
.

mucinous fluid (blue arrows); there is no associated hair.


N
\l

(Choices F) Patients with pelvic inflammatory disease are at risk for tubo—ovarian abscess; however, these abscesses are
N
.

a
N

typically filled with purulent material, which is not seen in this patient.
u:
O
N
u

31
v
Female Reproductive System a Breast
Ovarian cancer Ea Feedback
0
End Block
N
U
u
Mm 22 M40 I P Mark ® 5! a '
ORNQMfiUNt—l Question Id: 12994 Tutorial Lab Values Notes Calculator Reverse Color
anovu a cry an corpus u eum cys 3

(Choice B) Pelvic inflammatory disease and tobacco use are risk factors for ectopic pregnancy, which grossly appears as
fetal tissue (orange arrow) implanted within a dilated fallopian tube (red arrow) in patients with a positive pregnancy test.

(Choice (2) Endometriomas may cause severe dysmenorrhea and adnexal fullness; however, this diagnosis is unlikely
because endometriomas are a collection of old blood that creates a "chocolate cyst" appearance.

(Choice E) Mucinous cystadenomas are benign surface epithelial ovarian masses that appear as thin-walled loculations of
Hl-l
HO

mucinous fluid (blue arrows); there is no associated hair.

(Choices F) Patients with pelvic inflammatory disease are at risk for tubo—ovarian abscess; however, these abscesses are
HH
LON

typically filled with purulent material, which is not seen in this patient.
UNNNNNNNNNNHHHHHH
owuuomAwMi—Iomwuomh

(Choice G) Yolk sac tumors are malignant germ cell tumors with gray—yellow, necrotic, and hemorrhagic areas on a gross
specimen; there is no associated hair.

Educational objective:
Mature cystic teratomas are benign ovarian germ cell tumors that contain all 3 germ cell layers and can have sebaceous
fluid and hair on gross appearance.

References

- Laparoscopic treatment of ovarian dermoid cysts is a safe procedure.

- Ovarian teratoma in routine biopsy material during a five—year period.

- Spectrums and outcomes of adnexal torsion at different ages.

31
v
Female Reproductive System 8. Breast
ovarian cancer Ea Feedback
0
End Block
N
U
Item 22 of 40 t '
ORNQU‘fiUNH
anon
Question Id: 12094 _Ca_lulat¢_)r _ Revetse Color Text Zoom

ll!:llll—.i__
Hl-l
HG
HH
LON

f 2 “mg!
OwflflafiméwNI—‘ODGNOUI-h
UNNNNNNNNNNHHHHHH

’nfihfivw
31
32 v
Female Reproductive Sys‘em 8. Breast
Ovarian cancer E» Feedback
0
End Black
Hl-l
HH
HOORNQU‘fiUNH
LON
OwflflafiméwNI—‘ODGNOUI-h
UNNNNNNNNNNHHHHHH
'
Revetse Color Text Zoom
$3

31
v
Female Reproductive Sys‘em 8. Breast
Ovarian cancer E» Feedback
0
End Black
N
U
'
ORNQU‘fiUNH
an on
Revetse Color Text Zoom
Hl-l
HO
HH
LON
OwflflafiméwNI—‘ODGNOUI-h
UNNNNNNNNNNHHHHHH

Q Zoom In a Zoom Out 9 Reset 9’7 Add To Flash Card

31
32 v
Female Reproductive Sys‘em 8. Breast
Ovarian cancer E» Feedback
0
End Black
hem 23 one I ‘7 Mark ® it Q I
ORNQMfiUNt—l QUESHO" Id: 14759 Tutorial Lab Values Notes Calculator Reverse Color

A34—yearrold woman comes to the office for evaluation of amenorrhea. Last year the patient had a missed abortion that
was surgically managed with suction cu rettage. At her follow~up visit a few weeks later she had a copper~releasing
intrauterine device placed. Her menstrual periods were initially irregular and heavy, but the bleeding became increasingly
lighter. Now the patient has not had a menstrual period in 6 months. Prior to the intrauterine device placement she had
regular menses. The patient has had no weight changes, changes in vision, or galactorrhea. She has generalized anxiety
disorder. Her older brother has severe intellectual disability due to fragile X syndrome. Vital signs are normal. BMI is 22
Hl-l
HO

kg/mz. Pelvic examination shows a small, mobile uterus with no adnexal masses. There is no tenderness over the
perineum with speculum examination. Pregnancy test is negative. TSH is normal and FSH is elevated. A progesterone
HH
LON

challenge is performed and the patient has no withdrawal bleeding. Which of the following is the most likely diagnosis in
this patient?
NNNHHHHHH
Ni—‘ONDGNOUIA

O A. Functional hypothalamic amenon’nea [4%]

O B. Intrauterine adhesions[25%]

O C. Intrauterine device—induced amenorrhea [18%]

O D. Polycystic ovary syndrome [0%]

«Q E. Primary ovarian insufficiency [50%]


NNNNNNN
Wflflatulblu

om'fled Ill 50% 1 Second _ 01/15/2020 |


Earned answer Answered correctly Time Spent E Last Updated
O
N

31 Female Reproductive System 8. Breast


Amenorrhea
Q
0
v Feedback End Block
N
U
hem 23 of 40
IP’Mark <ir> 62) ii 3 '
$3
ORNQMfiUNt—t Question Id: 14758
Previous Next Tutorial Lab Values Notes Calculate r Reverse Color Text Zoom

Explanation

Primary ovarian insufficiency


Amenorrhea at age <40
Hypoestrogenic symptoms (eg, hot flashes)
Clinical features
1 FSH
Hl-l
HO

1 Estrogen
HH
LON

Turner syndrome (45,XO)


Fragile X syndrome (FMR1 premutation)
OWGMOUI-DUNI—‘ONDGMOUIIH
UNNNNNNNNNNHHHHHH

Autoimmune oophoritis
Major causes ,
- Anticancer dmgs
- Pelvic radiation
- Galactosemia

Management - Estrogen therapy (with progestin if intact uterus)

FMR1 = fragile X mental retardation 1.

This patient has secondary amenorrhea (lack of menses 26 months in a patient with previously irregular cycles) and an
elevated FSH level suggestive of ovarian failure. In women age <40, this is consistent with the diagnosis of primary
ovarian insufficiency (POI). In POI, the lack of ovarian function results in low estrogen levels, evidenced by a lack of
withdrawal bleeding after a progesterone stimulation challenge. In these patients, the low estrogen levels limit endometfial
proliferation; therefore, there is no endometrium to slough off with progesterone stimulation.

POI is common in women who are fragile X syndrome (FMR1 gene) premutation carriers (ie, those with 50200 CGG

31 Female Reproductive System a Breast


v Amenarrhea Ea Feedback
0
End Black
N
U
item 23 of40 . P Mark Q) i) g I
ORNQMfiUNt—t QUESfiOfl Id: "759 Tutorial Lab Values Notes Calculator Reverse Color

This patient has secondary amenorrhea (lack of menses 26 months in a patient with previously irregular cycles) and an
elevated FSH level suggestive of ovarian failure‘ In women age <40, this is consistent with the diagnosis of primary
ovarian insufficiency (POI). In POI, the lack of ovarian function results in low estrogen levels, evidenced by a lack of
withdrawal bleeding after a progesterone stimulation challenge. In these patients, the low estrogen levels limit endometrial
proliferation; therefore, there is no endometrium to slough off with progesterone stimulation.

POI is common in women who are fragile X syndrome (FMR1 gene) premutation carriers (ie, those with 50-200 CGG
Hl-l
HO

repeats). These women often have associated neurobehavioral clinical features (eg, generalized anxiety disorder, autism)
and a family history of fragile X syndrome, as seen in this patient. This premutation causes an FMR1 mRNA
HH
LON

overexpression, which is hypothesized to have a cytotoxic effect on ovarian primordial follicles and result in accelerated
follicle depletion. Due to this association, women with ovarian failure at age <40 and no other obvious cause for POI (eg,
NNNHHHHHH
NI—‘ONDGNOUIA

Turner syndrome) are tested for FMR1 gene mutations.

Management of POI includes estrogen—containing therapy to reduce the risk of comorbidities associated with low estrogen
levels (eg, osteoporosis, cardiovascular disease).

(Choice A) Functional hypothalamic amenorrhea typically occurs in women with a relative caloric deficiency (eg, athletes,
eating disorders) due to decreased GnRH secretion. Patients have no withdrawal bleeding after a progesterone challenge
due to low estrogen levels; however, FSH levels are low.

(Choice B) Intrauterine adhesions (Asherman syndrome) can cause secondary amenorrhea in patients who undergo
NNNNNNN
wauorurau

intrauterine procedures (eg, suction curettage) due to endometrial damage and fibrous tissue formation. Patients with
intrauterine adhesions have no withdrawal bleeding on a progesterone challenge; however, they have normal FSH levels,
making this diagnosis less likely.

(Choice C) LevonorgestreI-releasing intrauterine devices (IUDs) can cause amenorrhea by thinning the endometrium. In
contrast, copper—releasing IUDs can cause heavy menstrual bleeding and dysmenorrhea. FSH levels are normal in both.
O
N

31 Female Reproductive System a Breast


Amenorrhea
Q
0
v Feedback End Block
N
U
ii a '
ORNQMfiUNt—l Tutorial Lab Values Notes Calculator Reverse Color

(Choice B) Intrauterine adhesions (Asherman syndrome) can cause secondary amenorrhea in patients who undergo
intrauterine procedures (eg, suction curettage) due to endometrial damage and fibrous tissue formation. Patients with
intrauterine adhesions have no withdrawal bleeding on a progesterone challenge; however, they have normal FSH levels,
making this diagnosis less likely.

(Choice 8) Levonorgestrel-releasing intrauterine devices (lUDs) can cause amenorrhea by thinning the endometrium. In
contrast, copper-releasing IUDs can cause heavy menstIual bleeding and dysmenonhea. FSH levels are normal in both.
Hl-l
HO

(Choice D) Polycystic ovary syndrome can present with abnormal uterine bleeding and secondary amenorrhea. This
HH
LON

diagnosis is unlikely because these patients have normal estrogen levels and, therefore, would have withdrawal bleeding
after a progesterone challenge.
OWGMOUl-DUNI—‘ONDGMOUIIH
UNNNNNNNNNNHHHHHH

Educational objective:
Primary ovarian insufficiency (POI) occurs due to accelerated ovarian follicle depletion in women age <40, resulting in
secondary amenorrhea, elevated FSH, and low estrogen levels. A risk factor for POI is FMR1 gene premutation for fragile X
syndrome.

References

- Genetics of primary ovarian insufficiency: a review.

- Primary ovalian insufficiency.

- Population~based estimates of the prevalence of FMR1 expansion mutations in women with early menopause and
primary ovarian insufficiency.

31 Female Reproductive System 8. Breast


Amenorrhea
Q
0
v Feedback End Block
M
U
hem 24 one I ‘7 Mark ® it 3 I
ORNQMfiUNt—l QUESHO" Id: 2507 Tutorial Lab Values Notes Calculator Reverse Color

A53—yearrold woman comes to the office with a "strange, itchy rash" on her left breast, which has been present for the last
month The patient applied over~the«counter corticosteroid ointment onto this rash with no relief of symptoms She takes
no medications. The patient's last menstrual period was 2 years ago. She has a history of hypertension that improved with
weight loss and exercise. Physical examination shows an eczematous plaque on the left nipple and areola. Which of the
following conditions is most likely associated with this patient's finding?
Hl-l
HO

«O A. Adenocarcinoma [50%]

O B. Fibroadenoma[2%]
HH
LON

O C. Lymphoma [2%]
NNNNHHHHHI—l
l—‘ONDGNOUl-h

O D. Papilloma [14%]

O E. Squamous cell carcinoma [30%]

0m med
Correct answer
Ill 50% 1 Second — 04/292020
Answered oonectty Time Spent E Last Updated
WGNJOIUl-h
NNNNNN

Explanation

Mammary Paget disease


O
N

31 Female Reproductive System 8. Breast


Breast cancer
Q
0
v Feedback End Black
N
U
ORNQMfiUNt—t
hem 24 of 40
Question Id: 2607
<
Previous
D
Next Tutorial Lab Values Notes Calculate r
'
Reverse Color Text Zoom
$3
Mammary Paget disease
.

O
H
.

H
H
M
.

H
DJ
.
H
.

H
A
U!
.

H
at
.

H
\l
.

H
.

H
3
.

ND
H

Redness, ulceration, scaling


.

N
6

& flaking of nipple


l-‘
.

N
.

N
N

.31
v
Female Reproductive System 8. Breast
Breast cancer Ea Feedback
0
End Black
ORNQMfiUNt—t
hem 24 of 40
QUBSHO" Id: 2507
<
Previous
D
Next
®
Tutorial
ii
Lab Values
3
Notes Calculator
'
Reverse Color Text Zoom
$3
© World

Breast cancer should be considered in a patient with no history of skin disease who develops a breast rash. Mammary
Paget disease is suspected when a persistent, eczematous, and/or ulcerating rash is localized to the nipple and spreads
to the areola, Other characteristic findings include vesicles, scales, bloody discharge, and nipple retraction, Patients
experience pain, itching, and burning of the affected nipple and no resolution with topical corticosteroids.

Approximately 85% of patients with Paget disease of the breast have an underlying breast cancer, although a mass is not
Hl-l
HO

always palpable. Adenocarcinoma, which refers to carcinoma that starts in glandular tissue, is generally the most
common type of breast cancer and is also found in Paget disease. The nipple changes of Paget disease are thought to be
HH
LON

caused by migration of neoplastic cells through the mammary ducts to the nipple surface. Further workup should include
mammography and biopsy.
NNNNHHHHHI—l
r—Iewuuomh

(Choice B) Fibroadenoma is a benign breast lesion that typically presents as a palpable, mobile, rubbery, firm breast mass
without nipple changes.

(Choice 6) Breast lymphoma is extremely rare and typically presents as a painless, palpable breast mass without
cutaneous manifestations in a postmenopausal woman. Cutaneous lymphomas typically involve the trunk and/or
extremities versus presenting as isolated breast lesions.

(Choice D) Breast papillomas are typically intraductal and present with unilateral bloody nipple discharge but not nipple
lesions.

(Choice E) Cutaneous squamous cell carcinoma typically results from infection with human papilloma virus (eg,
OWGMOIUl-h
wNNNNNN

genital/periungual region) or frequent sun exposure (eg, head, neck, dorsum of the extremities). Primary squamous cell
carcinoma of the breast is very rare.

Educational objective:
The hallmark of mammary Paget disease is a painful, itchy, eczematous, and/or ulcerating rash on the nipple that spreads

Female Reproductive System a Breast Q


0
._.w

32 Breast cancer Feedback End Block


ORNQMfiUNt—t
hem 24 of 40
Q "95ti 0" Id'- 2607 . <
Previous D
Next '
Tutonal Lab Values Notes Calculator
'
Reverse Color Text Zoom
$3
always palpable. Adenocarcinoma, which refers to carcinoma that starts in glandular tissue, is generally the most
common type of breast cancer and is also found in Paget disease. The nipple changes of Paget disease are thought to be
caused by migration of neoplastic cells through the mammary ducts to the nipple surface. Further workup should include
mammography and biopsy.

(Choice B) Fibroadenoma is a benign breast lesion that typically presents as a palpable, mobile, rubbery, firm breast mass
without nipple changes.
.

O
H

(Choice C) Breast lymphoma is extremely rare and typically presents as a painless, palpable breast mass without
H
H

cutaneous manifestations in a postmenopausal woman. Cutaneous lymphomas typically involve the trunk and/or
M
.

extremities versus presenting as isolated breast lesions.


DJ
H
I-l

(Choice D) Breast papillomas are typically intraductal and present with unilateral bloody nipple discharge but not nipple
.

A
Ul
i-l

lesions.
at
I-l
.

(Choice E) Cutaneous squamous cell carcinoma typically results from infection with human papilloma virus (eg,
\l
i-l

genitallpeiiungual region) or frequent sun exposure (eg, head, neck, dorsum of the extremities). Primary squamous cell
.

H
a

carcinoma of the breast is very rare.


u:
H
.

N
e

Educational objective:
l—‘
N

The hallmark of mammary Paget disease is a painful, itchy, eczematous, and/or ulcerating rash on the nipple that spreads
N
N

to the areola. The majority of patients with mammary Paget disease have an undeliying breast adenocarcinoma.
cu
.

N
N
#-
.

N
Ul
.

References
N
at

- Mammary and extramammary Paget's disease.


N
\l
N
u

a
N
O'D
N
u

31
v
Female Reproductive System 8. Breast
Breast cancer Ea Feedback
0
End Black
N
U
u
them 25 one I ‘7 Mark ® it Q I
ORNQMfiUNt—l QUESHO" Id: 3370 Tutorial Lab Values Notes Calculator Reverse Color

A28—yearrold nulliparous woman being evaluated for infertility comes to the office due to clear vaginal discharge for the past
2 days The patient and her partner have intercourse every other day and have been trying to conceive for the past 7
months. Menstrual cycles occur every 29 days with 4 days of flow. Last week, she took penicillin for a sore throat. The
patient has no medical issues and has had no surgeries. She takes a daily prenatal vitamin. Pelvic examination shows
clear mucus at the cervical 05. Which of the following is the most likely explanation for this patient's discharge?
Hl-l
HO

O A. Bacterial vaginosis [10%]

O B. Candidiasis [3%]
HH
LON

O C. Cenlical mucus plug [25%]


awNi—tewwxiour-h
NNNNNHHHHHH

O D. Chlamydia [3%]

v0 5 Ovulation [56%]
O F. Trichomoniasis [0%]

Omitted
C nect I" 56% 1Seoond — (Ki/1912020
E0 answer Answered correctly Tlme Spent E Last Updated
WGMO‘UI
NNNNN

Explanation

Physiology of the fertile window


O
N

31 Female Reproductive System 8. Breast


Ovulation
Q
0
v Feedback End Block
N
U
Exhibit Display

Physiology of file fertile window

Prim-q Summary WW
blido blink:
CFO-av
blido
(Milton Comm Rog-ulna Corp-

cb 4—Folllculuphuo—b 1—LM pin—b

Day!) Day“
ovum

Q200mln QZoomOut SReset '9AddToFIashCard


hem 25 of 40 . ‘7 Mark < D Q) it Q. I
ORNQMfiUNt—l QUESHO“ I“: 337') Previous Next Tutorial Lab Values Notes Calculator Reverse Color
Ovulation marks the transition from the follicular phase ofthe menstrual cycle to the luteal phase. This occurs midway
through the woman's cycle and is preceded by an LH surge‘ the hormone typically detected by commercial ovulation kits.
Couples trying to conceive can attempt to schedule intercourse immediately prior to ovulation without LH measurements by
monitoring physiologic factors such as cervical mucus and basal body temperature.

Cervical mucus secretion close to ovulation (late follicular phase) increases in quantity and can be perceived by patients
as vaginal discharge. This mucus is clear, elastic, thin in consistency, and described similar in appearance to an uncooked
Hl-l

egg white. It is thought to facilitate sperm transport into the utems for conception. After ovulation occurs, the mucus
HO

becomes thick and less hospitable to sperm.


HH
LON

(Choice A) Bacterial vaginosis typically causes a thin white discharge with an odor rather than clear cervical mucus.
bWNl—‘ONDGNOUI-h
NNNNNHHHHHH

(Choice B) Candidiasis typically causes a thick white to yellow discharge and mucosal erythema rather than clear cervical
discharge.

(Choice C) Acervical mucus plug serves as barrier to ascending infection during pregnancy. This brown, red, or yellowish
thick mucus is typically shed before or during labor.

(Choice D) Chlamydia cervicitis can present with a mucopurulent cervical discharge and a friable cervix rather than clear
cervical secretions.

(Choice F) Trichomoniasis causes yellow-green malodorous discharge rather than clear cenrical mucus.

Educational objective:
Cenlical mucus just prior to ovulation is profuse, clear, thin, and corresponds with an LH surge. This physiologic finding is
OWGMO‘UI
wNNNNN

not an indication of infection.

References

Female Reproductive System 8. Breast Q


0
._.w

32 Ovulation Feedback End Block


ORNQMfiUNt—l QUESHO" Id: 16394
IVMark ®
Tutorial
ii
Lab Values
3
Notes Calculator
'
Reverse Color Text Zoom
ۤ3
A35—yearrold woman, gravida 3 para 0 aborta 3, comes in for evaluation of recurrent pregnancy loss. The patient has had
3 first—trimester spontaneous abortions in the last year and is becoming increasingly worried that she will never carry a
pregnancy to full term. She has regular, monthly menses with 4—5 days of moderate bleeding and cramping. The patient
has no chronic medical conditions and takes a daily prenatal vitamin. Family history is noncontributory. Wtal signs are
normal. BMI is 24 kg/m’. 0n pelvic examination, the uterus is small, anteverted, and nontender. There are no adnexal
masses. Urine pregnancy test is negative. Transvaginal ultrasound reveals a 2-cm submucosal fibroid at the uterine
Hl-l
HO

fundus. Thrombophilia workup is negative. Which of the following is the most appropriate treatment for this patient‘s
recurrent pregnancy loss?
HH
LON

O A. Clomiphene ovulation induction [1%]


NNNNNNHHHHHI—l
urar—‘euawxratur-h

VO B. Hysteroscopic myomectomy [80%]

O C. Low—molecular—weight heparin [10%]

O D. Uterine artery embolization [1%]

O E. Vaginal progesterone [5%]

2mm Ill 80% 186a _ 12/24r2019


B“ am' Answered conectty Tlme Spent E Last Updated
O‘DGNJO‘i
wNNNN

Explanation

Female Reproductive System 8. Breast


0
._.w

32 Uterine fibroids Feedback End Block


Item 26 of 40
< D ii 3 '
ORNQMfiUNt—t Question Id: 16304 Previous Next Tutorial Lab Values Notes Calculator Reverse Color Text Zoom
Explanation

This patient has had 23 consecutive pregnancy losses (ie, spontaneous abortions) consistent with recurrent pregnancy
loss. Initial workup for these patients includes pelvic ultrasound, karyotype, and thrombophilia testing because anatomic,
genetic, and thrombotic disorders are the most common causes.

This patient's recurrent pregnancy loss is likely due to her submucosal fibroid, a benign smooth muscle (myometrial)
tumor that can cause abnormal intrauterine anatomy by extending into the cavity or impinging on (ie, disrupting) the
.

O
H

endometrium. This compromises embryo implantation or growth, likely due to decreased endometrial thickness and the
H
HM
.

reduced vascularity of the fibroid (relative to the myometrium).


H
DJ
H

Therefore, the most appropriate treatment for recurrent pregnancy loss due to submuoosal fibroids is hysteroscopic
.

H
A

myomectomy (even if the fibroids are not causing heavy menses). This procedure restores normal intrauterine anatomy,
Ur
H

thereby reducing the risk of future pregnancy loss.


at
.

H
\r
H

(Choice A) Clomiphene is a selective estrogen receptor modulator that treats ovulatory infertility by increasing FSH levels
.

H
a

and promoting ovarian follicular development. Clomiphene does not treat recurrent pregnancy loss and has antiestrogen
u:
H

effects on the endometrium (which decreases endometrial thickness).


.

N
e
l-‘
N

(Choice C) Low—molecular—weight heparin is used in women with recurrent pregnancy loss due to an inherited
.

N
N

thrombophilia; it does not improve live birth rates in women with a negative thrombophilia workup.
u;
N

(Choice D) Uterine artery embolization decreases blood flow to the uterus, thereby reducing fibroid size. It is not
N
a

recommended for patients desiring future fertility because the resuflant decreased uterine perfusion increases the risk of
ur
N
N
or

obstetric complications (eg, fetal growth restriction) and decreases live birth rates.
N
\l

(Choice E) Vaginal progesterone is used in patients with an incidental shortened cervix (525 mm) on ultrasound to
NH
to“

decrease the risk of preterm delivery. It does not reduce the risk of recurrent pregnancy loss.
O
N

Ea
.

31 Female Reproductive System a Breast


Uterine fibroids
0
v Feedback End Block
N
U
u
ORNQMfiUNl—l QUESHO" Id: 16394
IVMark ®
Tutorial
ii
Lab Values
3
Notes Calculator
'
Reverse Color Text Zoom
$3
(Choice A) Clomiphene is a selective estrogen receptor modulator that treats ovulatory infertility by increasing FSH levels
and promoting ovarian follicular development. Clomiphene does not treat recurrent pregnancy loss and has antiestrogen
effects on the endometrium (which decreases endometrial thickness).

(Choice C) Low—molecular—weight heparin is used in women with recurrent pregnancy loss due to an inherited
thrombophilia; it does not improve live birth rates in women with a negative thrombophilia workup.

(Choice D) Uterine artery embolization decreases blood flow to the uterus, thereby reducing fibroid size. It is not
Hl-l
HO

recommended for patients desiring future fertility because the resultant decreased uterine perfusion increases the risk of
obstetric complicalions (eg, fetal growth restriction) and decreases live birth rates.
HH
LON

(Choice E) Vaginal progesterone is used in patients with an incidental shortened cenlix (525 mm) on ultrasound to
NNNNNNHHHHHI—l
urar—‘euawxratur-h

decrease the risk of preterm delivery. It does not reduce the risk of recurrent pregnancy loss.

Educational objective:
Recurrent pregnancy loss (23 consecutive pregnancy losses) may be caused by submucosal fibroids due to impaired
embryo implantation or growth. Patients are managed with hysteroscopic myomectomy, which restores normal intrauterine
anatomy and reduces the risk of future pregnancy loss.

References

- Recurrent pregnancy loss: generally accepted causes and their management.

- Evidence—based guidelines for the investigation and medical treatment of recurrent miscarriage.
toaster
NNNN

- Hysteroscopic myomectomy: techniques and preoperative assessment.


O
N

31 Female Reproductive System 8. Breast


Uterine fibroids
Q
0
v Feedback End Block
N
U
them 27 one I ‘7 Mark ® it Q I
ORNQMfiUND—l QUESHO" Id: 14720 Tutorial Lab Values Notes Calculator Reverse Color

A2—year—old girl is brought to the office by her mother, who noticed an abnormality in the genital area while changing her
diaper The patient has had multiple diaper rashes over the last 6 months but has had regular bowel movements and no
dysuria or hematuria‘ She has no chronic medical conditions and no known allergies. Vital signs are normal. 0n pelvic
examination, the labia minora appear thin, are fused together at the midline, and partially block the urethral meatus.
Multiple excoriations are seen, but there is no rash or perianal involvement. No vaginal discharge is seen at the introitus.
Internal pelvic examination is deferred. Which of the following is the most likely diagnosis in this patient?
.

O
H
H
H

O A. Candida vulvovaginitis [5%]


M
.

O B. Contact dermatitis [4%]


DJ
H
.

H
h

«’0 C. Labial adhesions [69%]


U!
H

O D. Lichen sclerosus[18%]
at
.

H
\l
H

O E Pinworm infection [0%]


.

H
3
u:
H

O F. Urethral prolapse [0%]


.

N
O
l—‘
N
.

N
M

(:0d III 69% 1Seoond _ 01/252020


DJ
N

0° “"5“” Answered correctly Time Spent E Last Updated


.1)
N
U1
N
at
N

Explanation
uNNN
O‘DGI

Female Reproductive System 8. Breast Q


0
._.w

Labial adhesion Feedback End Block


ORNQMfiUNt—t
Item 27 ot40
Question Id: 14720
l P, Mark <
Previous
5 ®
Tutorial
5!
Lab Values
3
Notes Calculator
'
Reverse Color Text Zoom
$3
Explanation

Normal Labial adhesion


.

O
H
H
H
M
.

H
DJ
H
u

Ih
H
Ul
H
u

at
H
\l
H

Vaginal
u

H
3

opening adhesion
ND
H
u

N
6
l—‘
N
u

N
N
0.!

©UWofld
N
N
0

This patient has thin, fused labia minora due to labial adhesions‘ This condition is most commonly seen in prepubertal
U1
N

gills due to low estrogen production. Inflammation from poor hygiene, infection (eg, vaginitis), irritation (eg, diaper rash),
at
.

or trauma (eg, straddle injury) also contributes to the development of adhesions.


\l
.

N
.

N
G

Labial adhesions can be partial (involving a portion of the labia) or complete Partial adhesions, as seen in this patient, are
N
in

often asymptomatic; however‘ some individuals may experience vaginal pain or pulling. Adhesions covering the urethral

Ea
u
O

31 Female Reproductive System a Breast


Labial adhesion
0
v Feedback End Block
N
U
u
Item 27 of40 I P Mark 6;) it Q 9
ORNQMfiUNt—t Question Id: 14720 Tutorial Lab Values Notes Calculator Reverse Color

©UWcrtd

This patient has thin, fused labia minora due to labial adhesions This condition is most commonly seen in prepubertal
girls due to low estrogen production. Inflammation from poor hygiene, infection (eg, vaginitis), irritation (eg, diaper rash),
or trauma (eg, straddle injury) also contributes to the development of adhesions.

Labial adhesions can be partial (involving a portion of the labia) or complete. Partial adhesions, as seen in this patient, are
often asymptomatic; however, some individuals may experience vaginal pain or pulling. Adhesions covering the urethral
.

O
H

meatus can also cause an abnormal urinary stream and an increased risk for recurrent urinary tract infections due to urine
H
H

accumulation.
M
.

H
DJ

Although mild, asymptomatic adhesions require no treatment, topical estrogen is first-line therapy for those with
H
.

H
h

symptoms.
U!
H

(Choice A) Candida vulvovaginitjs presents with a beefy—red rash on the vulva as well as vaginal pruritis and discharge.
at
.

Although patients with repeated yeast infections are more prone to labial adhesions, it is unlikely in this patient with no rash
\l
H

or vaginal discharge.
.

H
3
u:
H

(Choices B and D) Contact dermatitis and lichen sclerosus are types of skin inflammation that can predispose to labial
.

N
O

adhesions. However, contact dermatitis presents with a pruritic, erythematous rash, and lichen sclerosus is characterized
l—‘
N

by hypopigmented labial lesions. This patient has no skin discoloration on examination.


.

N
M
DJ
N

(Choice E) Pinworm infection typically presents in school—age children with perianal itching, particulariy at night. Although
.1)
N

pinwonns can cause vulvovaginitis in some cases, labial fusion is not associated.
U1
N
at

(Choice F) Urethral prolapse presents in prepubertal girls with inflamed, friable tissue in a donut shape at the urethral
N

meatus. This patient's obstructive, fused labia is due to labial adhesions.


uNNN
O‘DGI

Educational objective:

Female Reproductive System a Breast


0
._.w

Labial adhesion Feedback End Block


them 27 of 40 ® 5‘ g '
ORNQMfiUNt—t QUESHO" Id: “720 Tutorial Lab Values Notes Calculator Reverse Color

symptoms.

(Choice A) Candida vulvovaginitis presents with a beefy-red rash on the vulva as well as vaginal pruritis and discharge.
Although patients with repeated yeast infections are more prone to labial adhesions, it is unlikely in this patient with no rash
or vaginal discharge.

(Choices B and D) Contact dermatitis and lichen sclerosus are types of skin inflammation that can predispose to labial
adhesions. However, contact dermatitis presents with a pruritic, erythematous rash, and lichen sclerosus is characterized
Hl-l
HO

by hypopigmented labial lesions. This patient has no skin discoloration on examination.


HH
LON

(Choice E) Pinworrn infection typically presents in school-age children with perianal itching, particularly at night. Although
pinworrns can cause vulvovaginitis in some cases, labial fusion is not associated.
OiUI-bbJNl—‘OUJGMOUI-h
NNNNNNNHHHHHH

(Choice F) Urethral prolapse presents in prepubertal girts with inflamed, friable tissue in a donut shape at the urethral
meatus. This patient's obstructive, fused labia is due to labial adhesions.

Educational objective:
Labial adhesions, or fused labia minora, are most commonly seen in prepubertal girls with low estrogen production. Partial
adhesions are typically asymptomatic, and no treatment is necessary. Topical estrogen is first—line therapy for symptomatic
lesions.

References

- Labial adhesion and urinary tract problems: the importance of genital examination.

- Clinical recommendation: labial adhesions.


N
\1
N
G
LAN
Ora

31 Female Reproductive System 8. Breast


v Labial adhesion
Q
0
Feedback End Block
N
U
9.
ORNQU‘fiUNH
on an
LabVahns Notes Calulator Revetse Color Text Zoom
.

O
H
.

H
H
H
N
.

H
w
.
H
A
U!
.
H
.

H
a
\l
.

H
.

H
a
u:
.

H
.

N
e
|-‘
.

N
.

N
N
u;
.

N
.

N
.5
U!
.

b .un‘.
Q Zoom In a Zoom Out 8 Reset 9‘7 Add To Flash Card

meatus‘ This natient's obstructive fused labia is due to labial adhesions.


Female Reproductive Sys‘em 8. Breast
Labial adhesion E» Feedback
0
End Black
ORNQU‘fiUNH
ltem270lf40
Question Id: 14720 I V “all!
9.
Nata
E
Calodator
'
Revetse Color Text Zoom
$3

Candidal vaginitis
.

O
H
.

H
H
H
N
.

H
w
.
H
A
U1
.
H
a.
.

H
\l
.

H
.

H
a
u:
.

H
.

N
e
l-‘
.

Thick.
.

N
N

W
m‘
o;
.

N
.

N
.5
U1
.

Zoom In Q Zoom Out 9 Reset 9‘7 Add To Flash Card

meatus‘ This natient's obstructive fused labia is due to labial adhesions.


Female Reproductive System 8. Breast
Labial adhesion E» Feedback
0
End Black
ORNQMfiUND—l
Item270lf40
Question Id: 14720
.vualk ‘ >
Lab Valia
9.
Nata
an an
Calulator
'
Reveise Color Text Zoom
$3
Exhibit Display
Lichen sclerosus
.

O
H
.

H
H
M
.

H
DJ
.
H
.

H
h
U!
.

H
a.
.

H
\l
.

H
.

H
3
u:
.

H
.

N
O
l—‘
.

N
.

N
M
DJ
.

N
.1)
.

N
U1
.

N
Oi
.

miss-us
GUM

Zoom In Q Zoom Out 9 Reset 517 Add To Flash Card

meatusi This natienl's obstructive fused labia is due to labial adhesions.


Female Reproductive Sys‘em 8. Breast
Labial adhesion E» Feedback
0
End Black
Exhibit Display

Entembiasis life cycle

“Winmflim
mmmmwm

dmmmbhvews

QmIn QZoomom Cw WToFIashCard


ORNQU‘fiUND—l
Item270lf40
Question Id: 14720
.vualk ‘
Previous
>
Next Lab Valia
9.
Nola
'
Reveise Color Text Zoom
$3
Exhibit Display

Urethral prolapse
.

O
H
H
H
M
.

H
DJ
H
H
u

I.

Urethra
H
U1
H
u

0
H
\l
H
u

3
H
ND

\ ircumferential.
N
u

Labia minora
6

'able lesion
N

’-
l—‘
N
u

N
N
0.!
N
.b
N
U1
N
0‘
.

N
\l
.

9'7 Add To Flash Card


N

®\ Zoom In a Zoom Out 6 Reset


.

G
N
‘0

meatusi This natient's obstructive fused labia is due to labial adhesions.


u


O

Female Reproductive Sys‘em 8. Breast


0
w
l-‘

Labial adhesion Feedback End Black


them 28 one I ‘7 Mark ® it 3 I
ORNQMfiUNt—l QUESHO" Id: 15173 Tutorial Lab Values Notes Calculator Reverse Color

A21—yearrold woman comes to the emergency department due to severe right lower quadrant pain. The pain began 3
weeks ago and was initially intermittent and dull. However, over the last 4 hours the pain has become constant and severe.
She is now having nausea and vomiting, but no diarrhea or dysuria. The patient has no chronic medical conditions and has
had no surgeries. She has regular, monthly menses, and her last menstrual period was 2 weeks ago. Temperature is 37.8
C (100 F), blood pressure is 140/70 mm Hg, and pulse is 93/min. BMl is 19 kg/m‘. Abdominal examination reveals right
lower quadrant tenderness with rebound and guarding. Pelvic examination shows scant physiologic cervical discharge and
Hl-l
HO

no cervical motion tendemess. Atender, right adnexal mass is palpated. Pregnancy test is negative. Pelvic ultrasound
reveals a 6-cm, partially calcified, right ovarian mass with multiple thin, echogenic bands. There is decreased Doppler flow
HH
LON

to the light ovary. Which of the following is the most likely cause of this patient‘s mass?
O‘DWNJOIUl-tl-‘O'DGNOUI-h
UNNNNNNNNNNHHHHHH

«O A. Cystic teratoma [69%]


O B. Endometrioma [2%]

O C. Follicular cyst [9%]

O D. Serous cystadenocarcinoma [4%]

O E. Theca lutein cyst [4%]

O F. Tubo—ovan‘an abscess [10%]

Omitted
C m I" 69% 1 Second — BMW/2020
'
A0 “"5”” Answered correctly Time Spent E Last Updated

31 Female Reproductive System 8. Breast


Ovarian cancer
Q
0
v Feedback End Black
N
U
1 Item 28 of 40 < D
, Mark I1 E p
-A u A
2 Question I“: 15173 IV Previous Next nal Lab Values Notes Calculator Reverse Color Text Zoom fig;
3
4 Explanation
5
6
7 Mature cystic teratoma
s
- Benign ovarian germ cell tumor
9 Pathology _
1o - Endoderrn, mesodenn, ectodenn tissue

11 . Most asymptomatic
12 - Ovarian torsion
13
_ _ - Struma ovarii subtype: hyperthyroidism
14 Clinical features _
15 - Unilateral adnexal mass
16 . Ultrasound: complex, cystic, calcifications
17 . Gross appearance: sebaceous fluid, hair, teeth

18 Management . Ovan'an cystectomy or oophorectomy


19
2° This patient has ovarian torsion, a Misting of the ovary around the supporting ligaments (eg, infundibulopelvic, utero—
21 ovarian) containing the ovarian vessels. Torsion initially impedes venous outflow with continued arterial inflow, leading to
22 vascular congestion and edema. Persistent torsion (as in this patient) causes acute—onset, severe pelvic pain due to
23
24 ovarian ischemia and necrosis from complete ovarian vessel obstruction (eg, decreased ovarian Doppler flow) and can
25 be complicated by peritonitis (eg, fever, nausea, vomiting) and an acute abdomen (eg, rebound, guarding).

26 Any large ovarian mass (25 cm) can induce torsion; however, mature cystic teratomas (also known as derrnoid cysts),
‘ 27 common in premenopausal women, have an elevated risk due to their heterogeneous composition and variable density that
creates an unstable mass prone to rotation. This composition is seen on ultrasound as a partially calcified mass (99,
30 teeth) with multiple thin, echogenic bands (eg, hair). in case of torsion, treatment is prompt surgical detorsion to restore ,

31 Female Reproductive System a Breast Q 0


32 , Ovarian cancer Feedback End Block
Item 23 of40 - \7 Mark ® it Q I
ORNQMfiUND—t QUESHO" Id: 15173 Tutorial Lab Values Notes Calculator Reverse Color

This patient has ovarian torsion, a twisting of the ovary around the supporting ligaments (eg, infundibulopelvic, utero—
ovarian) containing the ovarian vessels. Torsion initially impedes venous outflow with continued arterial inflow, leading to
vascular congestion and edema. Persistent torsion (as in this patient) causes acute-onset, severe pelvic pain due to
ovarian ischemia and necrosis from complete ovarian vessel obstruction (eg, decreased ovarian Doppler flow) and can
be complicated by peritonitis (eg, fever, nausea, vomiting) and an acute abdomen (eg, rebound, guarding).

Any large ovarian mass (25 cm) can induce torsion; however, mature cystic teratomas (also known as dennoid cysts),
Hl-l
HO

common in premenopausal women, have an elevated risk due to their heterogeneous composition and variable density that
creates an unstable mass prone to rotation. This composition is seen on ultrasound as a partially calcified mass (eg,
HH
LON

teeth) with multiple thin, echogenic bands (eg, hair). In case of torsion, treatment is prompt surgical detorsion to restore
circulation and preserve fertility with either an ovarian cystectomy (to prevent recurrence) or oophorectomy (if necrotic).
O‘DWNJOIUl-tl-‘O'DGNOUI-h
UNNNNNNNNNNHHHHHH

(Choice B) Endometriomas are encapsulated collections of old blood from ectopic endometrial implants on the ovary that
appear as homogeneous cystic masses. There are typically no solid (eg, calcified, hyperechoic) components.

(Choice 6) Follicular cysts are physiologic cysts that are usually asymptomatic. Ovulation (ie, follicular cyst rupture) may
cause unilateral pelvic pain mid-cycle (ie, mittelschmerz), with a simple, small, thin—walled cyst 1 free fluid (physiologic).

(Choice D) Serous ovarian cancers (eg, cystadenocarcinoma) appear as complex masses with solid components;
however, they are typically not calcified or hyperechoic.

(Choice E) Theca lutein cysts appear as large, bilateral cystic masses rather than a unilateral mass. They arise from
markedly elevated B-hCG levels (eg, hydatidiforrn mole)—this patient's pregnancy test is negative, making this diagnosis
unlikely.

(Choice F) Tube-ovarian abscesses (TOAs) present with fever and tender, multiloculated, cystic masses with distortion of
normal adnexal structures, Although this patient does have a low-grade fever, a TOA is unlikely because there is no
abnormal discharge or cervical motion tenderness.

31 Female Reproductive System a Breast


Ovarian cancer
Q
0
v Feedback End Black
N
U
hem 23 one I ‘7 Mark ® it 3 I
ORNQMfiUNt—l QUESHO" Id: 15173 Tutorial Lab Values Notes Calculator Reverse Color

(Choice C) Follicular cysts are physiologic cysts that are usually asymptomatic. Ovulation (ie, follicular cyst rupture) may
cause unilateral pelvic pain mid-cycle (ie, mittelschmerz), with a simple, small, thin-walled cyst : free fluid (physiologic)

(Choice D) Serous ovarian cancers (eg, cystadenocarcinoma) appear as complex masses with solid components;
however, they are typically not calcified or hyperechoic.

(Choice E) Theca lutein cysts appear as large, bilateral cystic masses rather than a unilateral mass. They arise from
markedly elevated B-hCG levels (eg, hydatidiforrn mole)—this patient's pregnancy test is negative, making this diagnosis
Hl-l
HO

unlikely.
HH
LON

(Choice F) Tuba—ovarian abscesses (TOAs) present with fever and tender, multiloculated, cystic masses with distortion of
normal adnexal structures. Although this patient does have a low-grade fever, a TOA is unlikely because there is no
O‘DWNJOIUl-tl-‘O'DGNOUI-h
UNNNNNNNNNNHHHHHH

abnormal discharge or cervical motion tenderness.

Educational objective:
Ovarian torsion causes acute-onset, severe pelvic pain due to ovarian rotation impeding circulation and causing ischemia.
Mature cystic teratomas (derrnoid cysts) are common causes of premenopausal adnexal masses and have an elevated risk
of torsion due to their heterogeneous composition,

References

- Detecting ovarian disorders in primary care.

- Clinical risk factors for ovarian torsion.

- Mature cystic teratoma of the ovary: a cutting edge overview on imaging features.

31 Female Reproductive System 8. Breast


ovarian cancer
Q
0
v Feedback End Block
N
U
hem 28 M40 I ‘7 Mark ‘ p 6;) 5| 9‘ '
ORNQU‘fiUNH 0095110" “11 15‘” Previous Next Tulorial Lab Value. Notes Calcdator Revetse Color Text Zoom
€633
. *. l ,

W . Unilateral adnexal mass


Exhibit Display

Ovarian torsion
.

O
H
.

H
H
H
N
.

H
w
H
.

A
H
U!
.

H
.

a
H
\l
.

H
.

a
H
u:
.

N
.

e
N
|-‘
.

N
.

N
N
u;
.

N
.

A
N
U!
.

N
OI
.

N
NJ

0
Feedback End Black
ORNQU‘fiUNH
ltem280f40
Question Id: 15173 <_N>
Pteumus
@ilg‘
Tutorial Lab Vaha Notes Calulator
'-:Jۤ3
Reveise Color Text Zoom

- Unilatera‘ adnexal mass


Exhibit Display
—nEndOIWQIFIBISTFI 8 transverse)

‘I h” . : ’
.

O
H
.

H
H
M
.

H
DJ
.
H
.

H
A
U!
.

H
.

H
a

endometrioma
\l
.

2‘
.

H
a
u:
.

H
.

N
e
|-‘
.

N
.

N
N
u;
.

N
.

N
A
U!
.

N
OI
.

N
xi
.

. 29

@, Cl 0
. 30
. 31 Female Reproductive Sys‘em 8. Breast
. 32 ovarian can cer Feedback End Black
hem 29 one I ‘7 Mark ® 5] g I
ORNQMfiUNt—l QUESHO" Id: 12425 Tutorial Lab Values Notes Calculator Reverse Color

A40—yearrold woman comes to the emergency department due to abdominal pain for the past 2 weeks, The pain has
become worse over the past day and is not relieved by ibuprofen The patient has a fever but no nausea, vomiting,
diarrhea, or dysuria. She has no chronic medical conditions or previous surgery. Her last menstrual period was 2 weeks
ago; menses are regular and last 5 days, with painful cramping and heavy bleeding on the first day. The patient is currently
sexually active with a male partner and uses condoms for contraception. She does not use tobacco, alcohol, or illicit drugs.
Temperature is 38.8 C (102 F), blood pressure is 100/60 mm Hg, and pulse is 92/min. The abdomen has diffuse
.

O
H

tenderness to deep palpation that is worst in the right lower quadrant There is no rebound or guarding Pelvic examination
H
H

reveals a small, mobile, tender uterus and an enlarged, tender right adnexal mass. Leukocyte count is 22,000/mm3, C-
M
.

reaclive protein and CA—125 are elevated. Urine pregnancy test is negative. Pelvic ultrasonography reveals a large, thick—
DJ
H

walled multiloculated mass filled with debris obliterating the right adnexa. Which of the following is the most likely diagnosis
u

uh
H
U!

in this patient?
H
u

at
H

O A. Appendiceal abscess [1 %]
\l
H
u

H
3

Degenerating subserosal uterine leiomyoma [0%]


Iommpow
“000000
ND
H
u

N
6

Endometriosis [1%]
l-‘
N

Infected ectopic pregnancy [0%]


u

N
N
0.1
N

Mature cystic teratoma [14%]


.

N
A

Ovarian serous cystadenocarcinoma [32%]


U!
N
at
N

_ Ovarian torsion [2%]


\l
N

V‘ _ Tube—ovarian abscess [45%]


N
G
N
'0
O
u

31 Female Reproductive System 8. Breast


_ Pelvic inflammatory disease
Q
0
Feedback End Block
N
u
E hem290f40 lvmm q [> G») it a ' €633
ORNQMfiUNt—t Question I“: 12425 Previous Next Tutorial Lab Values Notes Calculator Reverse Color Text Zoom

Explanation

Chlamydia & gonorrhea in women

- Age <25
Risk factors . _ _
10 - High—risk sexual behavror
11
12 - Asymptomatic (most common)
13 . . - Cervicrtis
Manlfestahons .y
14 - Urethritis
15 - Penhepatitis (Fitz-Hugh and Curtis syndrome)
16
17 Diagnosis - Nucleic acid amplification testing
H

Empiric: azithromycin + ceflriaxone


3

I
19 Treatment - Confirmed chlamydia: azithromycin
20 - Confirmed gonorrhea: azithromycin + ceftriaxone
21
22 - Pelvic inflammatory disease
23 _ . - Ectopic pregnancy
24 Compllcatlons ..
- Infertility
25 - Pharyngitis
26
27 Tubo-ovarian abscess (TOA), a complication of pelvic inflammatory disease, presents with fever, abdominal pain, and a
' 2° complex multiloculated adnexal mass with thick walls and internal debris. TOA typically occurs in reproductive~age
women due to a polymicrobial infection of the upper genital tract, a normally sterile environment. Infection extends to the .

31 Female Reproductive System a Breast Q 0


32 _ Pelvic inflammatory disease Feedback End Black
hem 29 of40 . ‘7 Mark ® it Q I
ORNQMfiUNt—t 009550“ Id: 12425 Tutorial Lab Values Notes Calculator Reverse Color

Tuba-ovarian abscess (TOA), a complication of pelvic inflammatory disease, presents with fever, abdominal pain, and a
complex multiloculated adnexal mass with thick walls and internal debris. TOA typically occurs in reproductive—age
women due to a polymicrobial infection of the upper genital tract, a normally sterile environment. Infection extends to the
fallopian tubes and creates an inflammatory exudate, pumlent fluid, and wall thickening, which conglomerate into a complex
mass to create pyosalpinx (eg, infected fallopian tube) and TOA.

Laboratory findings include increases in nonspecific indices (eg, leukocytosis, C—reactive protein, CA—125), which along with
.

O
H

fever, suggest infection rather than malignancy (eg, ovarian serous cystadenocarcinoma) (Choice F). TOA is diagnosed by
H
H

imaging (eg, pelvic ultrasound, CT scan) and treated with broad~spectrum parenteral antibiotics.
M
.

H
DJ
H

(Choice A) An appendioeal abscess can cause a right lower quadrant abscess. However, typical presentation is
u

uh
H

periumbilical pain that migrates to the right lower quadrant with associated nausea, vomiting, and anorexia.
Ul
H
u

at

(Choice B) Degenerating subserosal uterine leiomyoma can present with abdominal pain, fever, and an irregularly
H
\l
H

enlarged, focally tender uterus. Ultrasound findings would reveal a leiomyoma rather than an adnexal mass.
u

H
3

(Choice 6) Endometriosis can present with pelvic pain and an adnexal mass (99, endometrioma) that is typically described
ND
H

as having a "ground glass" appearance an ultrasound. Physical examination reveals an adherent, immobile uterus, which is
u

N
6

not seen in this patient.


l-‘
N
u

N
N

(Choice D) An infected ectopic pregnancy is a rare cause of adnexal abscess. This diagnosis is excluded by a negative
U.)
N

pregnancy test.
.

N
A
Ul
N

(Choice E) A mature cystic teratoma may present with abdominal pain or symptoms of torsion but does not cause fever
at
N

and leukocytosis. Ultrasound reveals an ovarian cyst with solid components, hyperechoic nodules, and calcifications.
\l
N

(Choice G) Ovarian torsion presents with sudden onset of unilateral pelvic pain without fever or leukocytosis. Ultrasound
N
G
N

findings include an adnexal mass with absent Doppler flow to the ovary.
'0
O
u

31 Female Reproductive System a Breast


_ Pelvic inflammatory disease
Q
0
Feedback End Block
N
u
hem 29 of40 I ‘7 Mark ® it 3 I
ORNQMfiUND—t QUESHO" Id: 12425 Tutorial Lab Values Notes Calculator Reverse Color

(Choice A) An appendiceal abscess can cause a right lower quadrant abscess. However, typical presentation is
periumbilical pain that migrates to the right lower quadrant with associated nausea, vomiting, and anorexia

(Choice B) Degenerating subserosal uterine leiomyoma can present with abdominal pain, fever, and an irregularly
enlarged, focally tender uterus. Ultrasound findings would reveal a leiomyoma rather than an adnexal mass.

(Choice 8) Endometriosis can present with pelvic pain and an adnexal mass (eg, endometn'oma) that is typically described
as having a "ground glass" appearance on ultrasound. Physical examination reveals an adherent, immobile uterus, which is
.

O
H

not seen in this patient.


H
H
M
.

(Choice D) An infected ectopic pregnancy is a rare cause of adnexal abscess. This diagnosis is excluded by a negative
DJ
H

pregnancy test.
u

uh
H
U!
H

(Choice E) A mature cystic teratoma may present with abdominal pain or symptoms of torsion but does not cause fever
u

at
H

and leukocytosis. Ultrasound reveals an ovarian cyst with solid components, hyperechoic nodules, and calcificaticns.
\l
H

(Choice G) Ovarian torsion presents with sudden onset of unilateral pelvic pain without fever or leukocytosis. Ultrasound
u

H
3

findings include an adnexal mass with absent Doppler flow to the ovary.
ND
H
u

N
6

Educational objective:
l-‘
N

Tube-ovarian abscess is a complication of pelvic inflammatory disease; it presents with fever, abdominal pain, and a
u

N
N

complex muttiloculated adnexal mass with thick walls and internal debn's on ultrasound.
0.!
N
.

N
A
U!
N

References
at
N

- Sensitivity of ultrasound for the diagnosis of tuba-ovarian abscess: a case report and literature review.
\l
N
N
G
N
'0
O
to

31 Female Reproductive System 8. Breast


_ Pelvic inflammatory disease
Q
0
Feedback End Block
N
u
hem 30 one I ‘7 Mark ® it Q I
ORNQMfiUNt—l QUESHO" Id: 4135 Tutorial Lab Values Notes Calculator Reverse Color

A44—yearrold woman comes to the office due to night sweats and insomnia. For the past month, she has awakened
completely soaked with perspiration almost every night. She has also had difficulty concentrating at work. The patient has
had inegular menstrual periods for the past 6 months. She has a history of hypertension controlled with medication. The
patient quit smoking 5 years ago and does not use illicit dmgs. She typically has an alcoholic drink before bed.
Temperature is 36.7 C (98 F), blood pressure is 140/90 mm Hg, pulse is SO/min, and respirations are 14/min. Skin is normal
and there is no periorbital edema. The thyroid is nonenlarged and nontender, and there are no masses. Abdominal
.

O
H

examination is normal. The uterus is small and anteverted, and the vagina has minimal rugation. There are no palpable
H
H

adnexal masses. Urine pregnancy test is negative. Which of the following is the best next step in management of this
M
.

patient?
DJ
H
.

H
A

O A. Measure 24-hour urinary catecholamines [2%]


Ur
H
or
.

V O B. Measure serum TSH and FSH [56%]


\r
H

O C. Order urine toxicology screen [0%]


.

H
a
u:
H

O D. Prescribe oral hormone replacement [6%]


.

N
e
l-‘

O E. Provide reassurance and education about menopause [33%]


N
.

N
N
o;
N
.

N
A

Omitted ll 56% 1Seoond — 05/220020


Ur
N

Cunect answer I
Answered correctty Tlme Spent E Last Updated
or
.

N
\l
N
an
N

Explanation
u:
N
w
o

31 Female Reproductive System 8. Breast


_ Menopause
Q
0
Feedback End Block
N
u
hem300f40 3
ORNQMfiUNi—t Question Id: 4136
IVMark <rr>
Previous Next
62)
Tutorial
ii
Lab Values Notes Calculate r
'
Reverse Color Text Zoom
$3
Explanation

Menopause

Vasomotor symptoms
Oligomenonhealamenorrhea
.

O
H

Sleep disturbances
H
H

Clinical features Decreased libido


M
.

Depression
DJ
H

Cognitive decline
.

H
A

Vaginal atrophy
Ur
H
at
.

Clinical manifestations
\r

Diagnosis
H

T FSH
.

H
a
u:
H

Topical vaginal estrogen


Treatment
.

N
e

Systemic hormone replacement therapy


l-‘
N
.

N
N

This patient has vasomotor symptoms (eg, hot flashes or "night sweats"), insomnia, and irregular menses that are
o;
N

consistent with either menopausal transition or hyperthyroidism. Menopause and thyroid disorders can present with
.

N
A

overlapping clinical manifestations, and both are common in women age 40—50. Other symptoms of hyperthyroidism
Ur
N

include heat intolerance, tremor, weight loss, hyperrefiexia, diarrhea, hypertension, and palpitations. Patients with
or
.

symptoms concerning for hyperthyroidism should be evaluated with a serum TSH level.
\l
N
an
N

Menopause is defined as absent menses for 12 months. Menopausal transition and initial symptoms may begin a few years
u:
N

before the final menstrual period and commonly occur at age >45. During menopause, the circulating estrogen level
w
o

31 Female Reproductive System a Breast


_ Menopause Ea Feedback
0
End Block
N
u
Item 30 one I ‘7 Mark C?) i) E I
ORNQMfiUNt—t QUESHO" Id: 4135 Tutorial Lab Values Notes Calculator Reverse Color

This patient has vasomotor symptoms (eg, hot flashes or "night sweats"), insomnia, and irregular menses that are
consistent with either menopausal transition or hyperthyroidism. Menopause and thyroid disorders can present with
overlapping clinical manifestations, and both are common in women age 40—50. Other symptoms of hyperthyroidism
include heat intolerance, tremor, weight loss, hyperreflexia, diarrhea, hypertension, and palpitations. Patients with
symptoms concerning for hyperthyroidism should be evaluated with a serum TSH level.

Menopause is defined as absent menses for 12 months. Menopausal transition and initial symptoms may begin a few years
.

O
H

before the final menstrual period and commonly occur at age >45. During menopause, the circulating estrogen level
H
H
M
.

decreases, resulting in a decrease in the feedback inhibition on the hypothalamic—pituitary axis. This results in an elevated
DJ
H

serum FSH level, which can help confirm the diagnosis of menopause. Vasomotor symptoms may be treated with
.

H
h

hormone replacement therapy after a thyroid etiology is excluded (Choice D).


U!
H

(Choice A) Measurement of 24—hour urinary catecholamines can be used to diagnose pheochromocytoma, an uncommon
at
.

H
\l

cause of night sweats and hypertension. Other symptoms of pheochromocytoma, including tachycardia, palpitations, and
H
.

H
3

weight loss consistent with adrenergic hyperstimulation, are not seen in this patient.
ND
H

(Choice C) Urine toxicology screen identifies illicit drugs (eg, cocaine) that can cause diaphoresis and insomnia. These
.

N
6

substances have no associated menstrual irregularities. This patient's symptoms are more suggestive of either menopause
N
H

or hyperthyroidism.
.

N
N
0.!
N

(Choice E) Menopause can be diagnosed clinically in women over age 45 with a 12—month history of amenorrhea without
.

N
A

other physiologic causes. This patient, however, has symptoms concerning for hyperthyroidism, and serum FSH and TSH
U!
N

levels should be measured.


OI
.

N
\l
N

Educational objective:
on
N

Vasomotor symptoms, insomnia, and irregular menses could be due to hyperthyroidism or menopause in middle—age
women. Serum TSH and FSH levels should be measured in atients with these 5 mtoms.
Female Reproductive System a Breast
Menopause
Q
0
Feedback End Block
them 30 M40 < D. '
ORNQMfiUNt—t Question Id: 4135 Previous Next Tutorial Lab Values Notes Calculator Reverse Color Text Zoom
before the final menstrual period and commonly occur at age >45. During menopause, the circulating estrogen level
decreases, resulting in a decrease in the feedback inhibition on the hypothalamic~pituitary axis. This results in an elevated
serum FSH level, which can help confirm the diagnosis of menopause. Vasomotor symptoms may be treated with
hormone replacement therapy after a thyroid etiology is excluded (Choice D).

(Choice A) Measurement of 24-hour urinary catecholamines can be used to diagnose pheochromocytoma, an uncommon
cause of night sweats and hypertension. Other symptoms of pheochromocytoma, including tachycardia, palpitations, and
weight loss consistent with adrenergic hyperstimulation, are not seen in this patient.
.

O
H
H
H

(Choice 6) Urine toxicology screen identifies illicit drugs (eg, cocaine) that can cause diaphoresis and insomnia. These
M
.

substances have no associated menstrual irregularities. This patient's symptoms are more suggestive of either menopause
DJ
H

or hyperthyroidism.
.

H
A
Ul
H

(Choice E) Menopause can be diagnosed clinically in women over age 45 with a 12—month history of amenorrhea without
at
.

other physiologic causes. This patient, however, has symptoms concerning for hyperthyroidism, and serum FSH and TSH
\l
H

levels should be measured.


.

H
a

Educational objective:
H
in
.

Vasomotor symptoms, insomnia, and irregular menses could be due to hyperthyroidism or menopause in middle—age
N
e
l-‘
N

women. Serum TSH and FSH levels should be measured in patients with these symptoms.
.

N
N
o;
N

References
.

N
A
Ul
N

- Hyperthyroidism.
at
.

- Climacteric symptoms in middle—aged women with chronic somatic diseases.


\l
N
an
N
u:
N
w
o

31 Female Reproductive System 8. Breast


_ Menopause Ea Feedback
0
End Block
N
u
1 hem 31 of 40 'I '
. I Mark I g
' 2 Question I“: 12279 V nal Lab Values Notes Calculator Reverse Color
3

-
' 4 A 17—yearrold girl comes to the office for evaluation of heavy menstrual bleeding. She typically has 4 days of heavy
5 bleeding, often requiring 5 or 6 pads a day The patient awakens multiple times a night to change her pad and often soaks
' : through her clothing and bed sheets. She has mild cramping on the first 2 days of her menses that is controlled by
s ibuprofen but has no interrnenstrual bleeding or chronic pelvic pain. Menarche was at age 12; menses were irregular the
9 first couple years but are now regular and occur every 26 days. The patient has no chronic medical conditions and has had
_ 10 no surgery. She has a male partner and uses condoms for contraception. \fital signs are normal. BMI is 23 kg/m2_ On
11 bimanual examination, the uterus is small and mobile; no adnexal mass or tenderness is present. Laboratory results are as
- 12 follows:
13
. 14 Hemoglobin 9.6 g/dL

15 Mean corpuscular volume 76 um3


. 16
17 Platelets 300,000/mm3
. 1s
19 Prothrombin time 13 sec

' 2° Activated PTT 38 sec


21
. 22 Urine pregnancy test is negative. Which of the following is the most likely cause of this patient's heavy menstrual bleeding?
23
- 24 O A. Cyclic bleeding of ectopic endometrial glands [5%]
25
. 26 O B. Immature hypothalamic—pituitary—ovan'an axis [18%]

27 v O c. Impaired platelet adhesion [39%]


28 "
29 D. Inherited factor VIII deficiency [10%]

Female Reproductive System a Breast Q 0


Von willehrand disease Feedback End Black
1 _ hem 31 of40 ' I
= .l g -A u A
' 2 Question I“: 12279 ' ® Lab Values Notes Calculator Reverse Color Text Zoom fig;
3 . . _ . .
. 4
5 .
6 Hemoglobin 9.6 gldL

7 Mean corpuscular volume 76 um3


. s
9 Platelets 300,0001mm3

. 1o
11 Prothrombin lime 13 sec

' 12 Activated PTT 38 sec


13
. 14 Urine pregnancy test is negative. Which of the following is the most likely cause of this palient's heavy menstrual bleeding?
15
. 16 O A. Cyclic bleeding of ectopic endometrial glands [5%]
O O

_ Immature hypothalamic—pituitary—ovarian axis [18%]


\I
...

U!
H
3
u

_ Impaired platelet adhesion [39%]



O
N
u

O D. Inherited factor VIII deficiency [10%]


._.N

O O
3

. Localized hyperplastic overgrt of endometrial glands [20%]


I'I'I
N
0;

Proliferation of myometn'al smooth muscle cells [5%]


e

.""

25

. 26

28
27 '
Smlged “I 39% 2 Seconds — 03/28.:‘2020
one answer Answered correctly Time SDGM E L331 Updated
29 C
. 30 '
Female Reproductive System 8. Breast Q B O
. 32 _ Von willehrand disease Feedback End Black
Item 31 one I < D g ' €633
ORNQMfiUNt—t QUESHO" “'3 12279 Previous Next Tutorial Lab Values Notes Calculator Reverse Color Text Zoom
Explanation ‘

Platelet adhesion 8- activation via vWF


.

O
H
H
H
M
.

§‘
DJ
H
.

A
H

Circulating Factor VIII


protected
ur
H
H

GP llblllla
a
.

\l
H
3
.

H
ND
H
6
.

N
l-‘
N
N
.

N
N
0;

GP = giycoprotein: vWF = von Willebrand factor.


N

©UWorId
.

A
N
UI

This patient with heavy, regular menses; anemia; and normal coagulation studies likely has von Willebrand disease
N
OI
.

(vWD). vWD is a common bleeding disorder that is caused by either a quantitative or qualitative deficiency in von
N
\l

Willebrand factor (vWF) Patients with vWD have impaired platelet adhesion as vWF acts as a bridging giycoprotein
N
a

between platelets and endothelial factors at sites of vascular injury. vWF also serves as a carrier for factor VIII, preventing
N
in

30 its degradation by protein C. .


Female Reproductive System a Breast Q B O
32 V Von willehrand disease Feedback End Block
1 Item 31 of 40 't '
. Mark I 9 -‘ H A]
' 2 Question I“: 12279 IV nal Lab Values Notes Calculator Reverse Color
3
. 4 This patient with heavy, regular menses; anemia; and normal coagulation studies likely has von Willebrand disease
5 (vWD). vWD is a common bleeding disorder that is caused by either a quantitative or qualitative deficiency in von
. 6 Willebrand factor (vWF). Patients with vWD have impaired platelet adhesion as vWF acts as a bridging glycoprotein
7 between platelets and endothelial factors at sites of vascular injury. vWF also serves as a carrier for factor VIII, preventing
' 3 its degradation by protein C.
9
_ 10 Common clinical manifestations of vWD include easy bruising and mucocutaneous bleeding (eg, epistaxis, heavy menstrual
11 bleeding); symptom onset variability (eg, a few years after menarche) is based on disease severity. Patients typically have
- 12 a prolonged bleeding time; however, platelet count and PT are normal Patients may have a prolonged activated PTT, but
13 activated PTI' can be normal in those with mild disease (as in this patient) or in states of increased vWF synthesis (eg,
' 1“ pregnancy, oral contraceptive use, acute stress, thyroid hormone supplementation).
15
. 16 The diagnosis is confirmed by vWF testing (VWF antigen, ristocetin oofactor activity, factor VIII levels). Management, which
17 is rarely needed outside of trauma or surgery, includes desmopressin, vWF replacement therapy, and antifibrinolytics (eg,
. 1s tranexamic acid).
19
20 (Choice A) Endometriosis (ectopic endometrial glands) presents with heavy, painful menses and an immobile uterus. This
21 patient has a mobile uterus and menstrual pain that responds to ibuprofen, making this diagnosis unlikely.

' 22 (Choice B) In the first 1—2 years following menarche, adolescents may have an immature hypothalamic—pituitary—ovarian
23 axis, leading to irregular menses and anovulatory bleeding. Although this patient's menses were initially irregular, they have
' :: subsequentty normalized.

. 26 (Choice D) Inherited factor VIII deficiency (ie, hemophilia A) is an X—linked recessive disorder that is much more common in
27 males. It typically manifests with spontaneous hemarthrosis, intramuscular hematomas, and gastrointestinal or
28 genitourinary bleeding.
29
_ 3° (Choice E) Endometrial polyps (localized hyperplastic growth of endometrial glands) typically present with intermenstrual v
Female Reproductive System 8. Breast Q B O
. 32 ' Von willebrand disease Feedback End Block
'
1
23 “"95““ I“: 12279 '
il
Lab Values
5
Notes Calculator
'
Reverse Color
,. . 1 . _
, 4 patient has a mobile uterus and menstrual pain that responds to ibuprofen, making this diagnosis unlikely.

5 (Choice B) In the first 1-2 years following menarche, adolescents may have an immature hypothalamic-pituitary—ovanan
. 6
7 axis, leading to irregular menses and anovulatory bleeding. Although this patient’s menses were initially irregular, they have
s subsequentty normalized.

9 (Choice D) Inherited factor VIII deficiency (ie, hemophilia A) is an X-Iinked recessive disorder that is much more common in
' 10 males. It typically manifests with spontaneous hemarlhrosis, intramuscular hematomas, and gastrointestinal or
11 genitourinary bleeding.
- 12
13 (Choice E) Endometrial polyps (localized hyperplastic growth of endometrial glands) typically present with interrnenstrual
. 14 bleeding, which is not seen in this patient.
15
16 (Choice F) Uterine Ieiomyomata, the proliferation of myometrial smooth muscle, can cause heavy, regular menses.
17 However, patients typically have a large, irregularty shaped utems rather than a small, mobile utems.

. 18 Educational objective:
19 Von Willebrand disease is a common bleeding disorder that causes impaired platelet adhesion and is the most common
' 20 cause of heavy, regular menses in adolescents. Platelet counts and PT are normal, activated P'l'l' may be normal or
21
. 22 prolonged.

23
. 24 References
25
26 - ACOG committee opinion number 580: von Vlfillebrand disease in women.

27 - Practice bulletin no. 128: diagnosis of abnormal uterine bleeding in reproductive—aged women.
28
29
30
Female Reproductive System 8. Breast Q B O
32 _ Von Willebrand disease Feedback End Black
ORNQMfiUNl—l
Item 31 of 40
Question Id: 12279 .
V
Halli Tutorial
a
Lab Valles
9‘
Nola Calulator
'
Revetse Color Text Zoom
$3

Pelvic endometrlosis

Implants (bladder)
.

O
H
.

H
H
H
N
.

H
w
u

I.
H
U1
-

H
u

H
0
\l
-

H
u

H
3
.

ND
H
u

N
6
l-‘
.

N
u

N
N
0.!
.

N
u

uh
N
U1
.

N
u

a!
N

Implanls (bowel)
\l
-

a Zoom Out 8 Reset 97 Add To Flash Card


.

N
G
u

N
‘0


.

u
O

Female Reproductive Syslem 8. Breast


Von willehrand disease
0
Feedback End Black
ORNQU‘fiUNH
= Item 31 of 40
Question Id: 12279 lm Tmofial Lab Values

Nata
anon
Calulator
'
Reverse Color Text Zoom
$3
'6 norma In 1089 WI ml 0 “59859 as In WIS pa Ien or In S a 95 0 Increase. V" syn eSIS eg,

Exhibit Display
Endometrial polyps
Hl-l
HO
HH
LON
wGNOIUI-tI-‘ONDGMOUI-h
NNNNNNNNNNHHHHHH

Q Zoom In Q Zoom Out 9 Reset [-17 Add To Flash Card


30

32
Female Reproductive Sys‘em 8. Breast
v Von willebrand disease Ea Feedback
0
End Black
ORNQMfiUNl—l
E 9095““
“”3””, “I: 12279
[Plum ® iI 9‘ ' .3 £633
Tulon'al Lab Values Notes Calulator Reverse Color Text Zoom
'6 norma In 1089 WI ml 0 “59859 as In IS pa Ien or In S a 95 O Increaseo V" syn eSIS eg, A

Exhibit Display

Uterlno flbrolds
.

O
H
.

H
H
H
N
.

H
w
.
H
A
U!
.
H
a.
.
H
\l
.
H
.
H
a

Intramural
u:
.
H
N
.

O
N

Sibmuoosal
l—‘
.

N
N
.

N
u;
.

N
A
.

N
U!
.

N
OI
.

OUWGH
\l
.

Q Zoom In Q Zoom Out 9 Reset 517 Add To Flash Card


N
Q
N
to

.30 '
Female Reproductive Sys‘em 8. Breast % D O
. 32 _ Von willebrand disease Feedback End Black
hem 32 one I ‘7 Mark ® it Q I
ORNQMfiUNt—t QUESHO" Id: 12373 Tutorial Lab Values Notes Calculator Reverse Color

Six weeks after a spontaneous, uncomplicated term vaginal delivery, a 32—year—old woman, gravida 1 para 1, comes to the
emergency department due to left breast pain, The patient first noticed pain and redness on her left breast a week ago,
She has continued to breast feed her infant from the unaffected breast, The patient has no chronic medical conditions and
her only medication is a daily multivitamin. Temperature is 38.3 C (101 F). Physical examination shows an area of
erythema extending from the areola to the lateral edge of the left breast and surrounding a well—circumscnbed, 4—cm area of
fluctuance. Axillary lymphadenopathy is present. Which of the following is the best next step in management of this
.

O
H

patient?
H
H
H
N

O O O
.

U o m >

_ Breast binding and acetaminophen [1%]


2

. Cone needle and skin punch biopsies [3%]


H
U1

. Ice packs and use of a supportive bra [3%]


5

m O
H
\l

_ Needle aspiration and antibiotics [68%]


*5
O
a

_ Warm compresses and massage [22%]


. 20
21
. 22 g
23 0mmed III 63% 4Seconds — OMS/2020
. 24 gorredanswer Answered correctly Tune Spem E Last Updated
25
. 26
27 Explanation
. 28
29
30 Breast abscess .
Female Reproductive System 8. Breast Q 0
Breast abscess Feedback End Black
'
1
2
mm
QUESHWMZW“
<I>
prewous
- N ex! Tt'l
®
U one
:1
LbVl
a a Lies
a
Nt
0 es Cl3 CuIt
:4 0f
'
Reverse Clo
O I
me
Tt
e com

3
. 4

5 Explanation
. 6 l
7
- 3 Breast abscess
9
_ 1o - Maternal age >30
11 Risk factors - First pregnancy
- 12 - Tobacco use
13
14 - Fever
15 Clinical features - Focal inflammation
. 16 - Fluctuant, tender mass

17 Diagnosis - Breast ultrasound


. 1s
19 - Antibiotics
Management _
. 20 - Drainage
21
, 22 The patient has a breast abscess based on the presence of signs of mastitis (fever, localized erythema/pain) accompanied
23 by a unilateral fiuctuant, tender, palpable mass. Breast abscesses typically develop from untreated mastitis (as seen in
- 24 this patient), which is typically due to incomplete emptying of the breast (eg, poor latch, alternating of breast and bottle
25 feeds) and resultant milk stasis. In turn, persistent or severe mastitis can lead to a focal collection of punilent fluid
' 26 (abscess). Mastitis and breast abscess are typically caused by Staphylococcus aureus, which enters the breast through
:; nipple trauma (eg, chafing/blistering),

29 The diagnosis of breast abscess is made clinically, but ultrasound may be required to differentiate severe mastitis from an

Female Reproductive System a Breast Q 0


Breast abscess Feedback End Black
1 hem 32 one - P Mark ® it Q 9
' 2 Question Id: 12678 Tutorial Lab Values Notes Calculator Reverse Color
3
4 The patient has a breast abscess based on the presence of signs of mastitis (fever, localized erythema/pain) accompanied
5 by a unilateral fluctuant, tender, palpable mass. Breast abscesses typically develop from untreated mastitis (as seen in
_ 6 this patient), which is typically due to incomplete emptying of the breast (eg, poor latch, alternating of breast and bottle
7 feeds) and resultant milk stasis In turn, persistent or severe mastitis can lead to a focal collection of pumlent fluid
- 8 (abscess), Mastitis and breast abscess are typically caused by Staphylococcus aureus, which enters the breast through
9 nipple trauma (eg, chafing/blistering),
. 1o
11 The diagnosis of breast abscess is made clinically, but ultrasound may be required to differentiate severe mastitis from an
_ 12 abscess if a mass is deep within the tissue. Management includes drainage of the abscess with either needle aspiration
13 (usually under ultrasound guidance) or incision and drainage. Empiric antibiotics against methicillin-sensitive S aureus
. 14 (eg, dicloxacillin, cephalexin) are added for treatment of the surrounding mastitis. Continued breastfeeding is
15 recommended for continued milk drainage.
. 16
17 (Choice A) Breast binding is not recommended for breast engorgement, mastitis, or abscess because it increases pain and
, 13 facilitates ductal blockage.

19 (Choice B) Core needle and skin punch biopsies are indicated in the evaluation of inflammatory breast cancer, which can
' :2 present similar to mastitis or abscess (eg, unilateral breast pain, axillary lymphadenopathy) However, patients typically
22 have peau d'orange skin thickening and a solid (rather than fluctuant) mass.

23 (Choice C) Ice packs and a supportive bra are indicated in the management of breast pain (ie, mastalgia). Patients with
' 2“ mastalgia have bilateral breast pain with no associated mass.
25
, 26 (Choice E) Warm compresses and massage are indicated in the management of a clogged lactiferous duct, which can
27 present with unilateral breast pain; however, the mass is typically located in the subareolar region and has no associated
- 28 axillary lymphadenopathy.
29
30 Educational objective: ,

. 31 Female Reproductive System a Breast Q 0


_ Breast abscess Feedback End Block
1 them 32 of 4D 1 '
. I Mark I 5
' 2 Question I“: 12W“ V nal Lab Values Notes Calculator Reverse Color
3
. 4 (Choice A) Breast binding is not recommended for breast engorgement, mastitis, or abscess because it increases pain and
5 facilitates ductal blockage.
. 6
7 (Choice B) Core needle and skin punch biopsies are indicated in the evaluation of inflammatory breast cancer, which can
8 present similar to mastitis or abscess (eg, unilateral breast pain, axillary lymphadenopathy). However, patients typically
9 have peau d'orange skin thickening and a solid (rather than fluctuant) mass.

' 1" (Choice C) Ice packs and a supportive bra are indicated in the management of breast pain (ie, mastalgia). Patients with
11 mastalgia have bilateral breast pain with no associated mass.
- 12
13 (Choice E) Warm compresses and massage are indicated in the management of a clogged lactiferous duct, which can
. 14 present with unilateral breast pain; however, the mass is typically located in the subareolar region and has no associated
15 axillary lymphadenopathy.
. 16
17 Educational objective:
. 18 Breast abscesses present with a unilateral fluctuant, tender, palpable breast mass with fever, surrounding
19 erythemalpain, and associated axillary lymphadenopathy. Management includes drainage (eg, needle aspiration) and
. 20 empiric antibiotics (eg, dicloxacillin, cephalexin),
21
. 22
References
23
- 24 - Breast infection: a review of diagnosis and management practices.
25
26 - Breast abscess: evidence based management recommendations.

27 - Treatments for breast abscesses in breastfeeding women.


. 28
29
30
. 31 Female Reproductive System 8. Breast Q 0
_ Breast abscess Feedback End Black
ltem32olf4o ® 5| P {Q
ORNQU‘fiUNH
Question Id: 12078 Reverse Color Text Zoom

Exhibit Display
Inflammatory breast cancer
.

O
H
.

H
H
H
N
.

H
w
H
.

A
H
U!
.

H
a.
.

H
\l
.

H
.

a
H
u:
.

N
.

e
N
|-‘
.

N
.

N
N
u;
.

N
.

A
N
U!
.

WWI
N
OI
.

N
\l
.

N
.

a
N
u:
.

u
.

Female Reproductive Sys‘em 8. Breast % 0


w
.

Breast abscess Feedback End Black


hem 33 one I ‘7 Mark ® it Q I
QUESHO" Id: 394‘ Tutorial Lab Values Notes Calculator Reverse Color

A 23—yearrold woman comes to the emergency department due to pelvic pain that started immediately after sexual
intercourse 3 hours earlier The pain is sharp and stabbing in nature and has intensified significantly. She has no fever or
nausea. She had a laparoscopic appendectomy 5 years ago. The patient has no other medical problems and takes no
medications. Her last menstmal period was 3 weeks ago. Temperature is 36.7 C (93 F), blood pressure is 110/70 mm Hg,
and pulse is 98/min. The abdomen is soft and tender to palpation in the light lower quadrant. Laboratory results are as
follows:

Hemoglobin 12.2 gldL

Platelets 260,0ClDImm3

Leukocytes 8,200/mm3

Urine pregnancy test is negative. Pelvic ultrasound shows a 4x5 cm right cystic ovarian mass with a moderate amount of
free fluid in the pelvis. Doppler velocimetry is normal. Which of the following is the most likely diagnosis?

O A. Adnexal torsion [10%]

O B. Endometriosis [2%]

O C. Mittelschmerz [1%]

VO D. Ovarian cyst rupture [84%]

O E. Tubo—ovan‘an abscess [1%]

Ormted

Female Reproductive System 8. Breast


ovarian cyst
Q
0
Feedback End Block
9 _ hem33of40 IVMark < D Q) ii 3 p .3 €333
10 Question I“: 89‘" Previous Next Tutorial Lab Values Notes Calculator Reverse Color Text Zoom
11
12 Explanation
13
14
15 Acute abdominal/pelvic pain in women

16 Diagnosis Clinical presentation Ultrasound findings


17
1s _ - Recurrent mild & unilateral mid—cycle pain pn'or to ovulation _ _
Mlttelschmerz , Not Indicated
19 . Pain lasts hours to days
20
Ectopic - Amenorihea, abdominal/pec pain & vaginal bleeding _ _
21 _ . No intrautenne pregnancy
22 pregnancy - Posmve B—hCG

23 - Sudden-onset, severe, unilateral lower abdominal pain; Enlarged ovary with


24 .
25 Ovarian torsion nausea & vomiting decreased or absent blood
26 - Unilateral, tender adnexal mass on examination flow

27 Ruptured - Sudden—onset, severe, unilateral lower abdominal pain _ _


23 . . . . _ , PeIVIc free flund
ovarian cyst Immedlately followmg strenuous or sexual actIVIty
29
30 Pelvic _ _ _ _ _
. - Fever/chills, vaginal discharge, lower abdominal pain & .
31 Inflammatory _ , i Tuboovanan abscess
cerVIcal motion tenderness
- 32 disease

. 34 This patient‘s presentation suggests a ruptured ovarian cyst, a cause of acute pelvic pain affecting women of
35 reproductive age. Symptoms are caused by peritoneal irritation from leaking of cyst contents. Patients typically develop
35 sudden onset of unilateral lower abdominal pain, often after strenuous activity or sexual intercourse.
37
as Physical examination shows tenderness of the lower abdomen, and an adnexal mass is sometimes palpable A complete .

39 Female Reproductive System a Breast Q 0


40 v Ovarian W“ Feedback End Block
Item 33 one I \7 Mark C?) i! E I
. 10 QUESHO" Id: 39‘" Tutorial Lab Values Notes Calculator Reverse Color
11

- 12
This patient's presentation suggests a ruptured ovarian cyst, a cause of acute pelvic pain affecting women of
13
reproductive age. Symptoms are caused by peritoneal irritation from leaking of cyst contents. Patients typically develop
. 14
sudden onset of unilateral lower abdominal pain, often after strenuous activity or sexual intercourse.
15
. 16 Physical examination shows tenderness of the lower abdomen, and an adnexal mass is sometimes palpable. A complete
17 blood count should be ordered to assess for anemia due to acute blood loss (ovarian bleeding from cyst rupture), and a
u 18
pregnancy test should be obtained to exclude ectopic pregnancy. Pelvic ultrasound usually shows pelvic free fluid from
19
leaking cyst contents, but an adnexal mass may be absent in the case of complete rupture.
u 20

21 An uncomplicated cyst rupture with no fever, hypotension, tachycardia, or signs of hemoperitoneumlinfection can be
. 22 managed conservatively with analgesics on an outpatient basis. Patients who are hemodynamically unstable or have
23 significant hemopelitoneum require surgical intervention.
. 24

25 (Choice A) Adnexal torsion also presents with sudden onset of unilateral lower abdominal pain. Although an ovarian mass
. 26 is often present, pelvic ultrasound with Doppler velocimetry shows enlarged, edematous ovaries with decreased or absent
27 blood flow.
. 28
(Choice B) Endometriosis typically presents with chronic pelvic pain, dysmenorrhea, deep dyspareunia, infertility, or
29
30
bowel/bladder dysfunction. Pelvic ultrasound may show an adnexal mass (eg, ovarian endometrioma), but free fluid is

31 typically not present.


. 32
(Choice 6) Mittelschmerz is midcycle pain due to follicular rupture with ovulation. The pain is typically mild, unilateral, and
lasts less than a day; it occurs 2 weeks into the menstrual cycle, corresponding with the time of ovulation.
. 34

35 (Choice E) Tubo—ovan'an abscess is unlikely without a fever and leukocytosis. Ultrasound usually shows a complex
36 multilocular adnexal mass rather than the simple cyst seen in this patient.
37
. 38
Educational objective:

.
39
4D
Female Reproductive System a Breast
V Ovarian cyst
Q
0
Feedback End Block
. _
Question “1- 89‘"
ll 3 '
' Tutorial Lab Values Notes Calculator Reverse Color
1 u -

An uncomplicated cyst rupture with no fever, hypotension, tachycardia, or signs of hemoperitoneum/infection can be
managed conservatively with analgesics on an outpatient basis. Patients who are hemodynamically unstable or have
significant hemopelitoneum require surgical intervention.

(Choice A) Adnexal torsion also presents with sudden onset of unilateral lower abdominal pain. Although an ovarian mass
is often present, pelvic ultrasound with Doppler velocimetry shows enlarged, edematous ovaries with decreased or absent
blood flow.

(Choice B) Endometriosis typically presents with chronic pelvic pain, dysmenon’nea, deep dyspareunia, infertility, or
bowel/bladder dysfunction. Pelvic ultrasound may show an adnexal mass (eg, ovarian endometrioma), but free fluid is
typically not present.

(Choice C) Mitlelschmerz is midcycle pain due to follicular rupture with ovulation. The pain is typically mild, unilateral, and
lasts less than a day; it occurs 2 weeks into the menstrual cycle, corresponding with the time of ovulation.

(Choice E) Tubo-ovan'an abscess is unlikely without a fever and leukocytosis. Ultrasound usually shows a complex
multilocular adnexal mass rather than the simple cyst seen in this patient.

Educational objective:
A ruptured ovarian cyst presents with acute onset of unilateral pelvic pain, often precipitated by strenuous activity or sexual
intercourse. Pelvic ultrasound can confirm the diagnosis by showing free fluid in the pelvis from leaking cyst contents

References

- Diagnosis and management of ovarian cyst accidents.

39 Female Reproductive System 8. Breast


Y ovarian cyst
Q
0
Feedback End Block
hem340f40
QUESHO" Id: 12439
IVMark ®
Tutorial
5!
Lab Values
g
Notes Calculator
'
Reverse Color Text Zoom
ۤ3
A45—yearrold woman comes to the office due to increasing difficulty sleeping for the past few months. She is unable to
concentrate at work and is not exercising due to fatigue The patient reports a 4.5«kg (9.9-lb) weight gain and is now unable
to wear her normally fitting clothing. She has no mood changes, hair loss, weakness, or changes in bowel or bladder
function. She has no chronic medical problems or previous surgeries. The patient's menstrual periods have become
irregular over the last 2 years; her last period was 3 months ago. She is currently sexually active. She does not use
tobacco, alcohol, or illicit drugs. Blood pressure is 120/70 mm Hg and pulse is 80/min. BMI is 31 kglm? The patient has
moist mucous membranes with normal capillary refill. There is no acanthosis nigricans. The abdomen is obese; there is no
hepatosplenomegaly or ascites. On bimanual examination, the uterus is symmetrically enlarged. There are no palpable
adnexal masses. Which of the following is the best next step in management of this patient?

O A. Abdominal CT scan [2%]

O B. FSH level [11%]

«Q o. hCG level [63%]


O D. Nocturnal polysomnography [1%]

O E. TSH level [20%]

Omitted
I" 63% 1Second — 05/20/2020
Correct answer
Answered correctly Tlme Spent E Last Updated

Explanation

39 Female Reproductive System 8. Breast


V undiagnosed pregnancy
Q
0
Feedback End Black
hem340f40
Question Id: 12439
IVMark <
Previous
D
Next
Q)
Tutorial
ii
Lab Values
3
Notes Calculator
'
Reverse Color Text Zoom
$3
Explanation

This patient has insomnia, fatigue, weight gain, amenorrhea, and an enlarged uterus consistent with pregnancy.
Women begin the menopausal transition (ie, perimenopause) years before the final menstrual period and onset of
menopause (average age 51). Ovulation still occurs during this time, and women may become pregnant despite lower
conception rates. Symptoms of penmenopause include irregular menstrual periods, insomnia, fatigue, and weight gain,
which often overlap with symptoms of early pregnancy. All women of reproductive age with amenorrhea and signs of
pregnancy should be evaluated with an hCG level.

(Choice A) Ovarian cancer may present with fatigue, increased abdominal circumference due to ascites, and weight loss
(rather than weight gain). Physical examination findings include ascites and an adnexal mass, rather than a uniformly
enlarged uterus. Evaluation of ovarian cancer includes an abdominal CT scan and pelvic ultrasound.

(Choice B) Menopause can cause symptoms similar to those of pregnancy. However, patients typically have a decreased
uterine size. Decreased ovarian function from menopause can be confirmed with an elevated FSH level.

(Choice D) Nocturnal polysomnography evaluates for the presence of sleep disorders (eg, sleepwalking, nocturnal
epilepsy), This patient's insomnia and fatigue are common in pregnancy and require no further evaluation.

(Choice E) Thyroid disorders may present with fatigue and menstrual disturbances; diagnosis is with abnormal TSH
levels. This patient does not have other typical symptoms of hypothyroidism - mood changes, hair loss, weakness, and
changes in bowel function. In addition, thyroid disorders do not cause uterine enlargement.

Educational objective:
Pregnancy can present in a perimenopausal woman with insomnia, fatigue, weight gain, amenorrhea, and an enlarged
uterus. Patients with amenorrhea and other signs of pregnancy should be evaluated with an hCG level.

39 Female Reproductive System a Breast


V undiagnosed pregnancy
Q
0
Feedback End Block
hem 35 one I ‘7 Mark ® it Q I
QUESHO" Id: 12393 Tutorial Lab Values Notes Calculator Reverse Color

A 19—yearrold woman comes to the office due to severe acne. She had expected the acne to improve after high school, but
instead, it is worsening The patient has tried multiple remedies without significant improvement She is concerned that the
acne is scarring her face. Since the patient started college last year, she has gained 13.6 kg (30 lb). She sings in a local
church choir and has not noticed any voice changes. The patient has no chronic medical conditions, has had no surgeries,
and takes no medications. BMl is 29 kg/mz. Physical examination shows nodulocystic acne on the face, arms, back, and
forehead. The scalp is normal, with no alopecia. Tanner stage is 5. There is no skin discoloration or striae. No masses are
palpable on abdominal examination. A urine pregnancy test is negative. Which of the following additional information is
required to establish the most likely diagnosis?

O A. Family history of hypothyroidism [1%]

O B. Food intake diary [4%]

O C. History of an eating disorder [0%]

O D. Intravenous drug use [0%]

«’0 E. Menstmal history [93%]

O F. Number of lifetime sexual partners [0%]

Omitted II 93% 1Se00nd — OMB/2020


Correct answer I
Answered oorreclty Time Spear E Last Updated

Explanation

39 Female Reproductive System 8. Breast


V Polycystic ovary disease
Q
0
Feedback End Black
Imm35of40 IVMark < D. 3 '
$3
Question Id: 12303 Previous Tutorial Lab Values Notes Calculate r Reverse Color Text Zoom
Next

Explanation

Hyperandrogenism
Hirsutism
Nodulocystic acne
Clinical features
Androgenic alopecia
T Semm testosterone

Polycystic ovary syndrome


Androgen—secreting tumor
Differential diagnosis
Cushing syndrome
Nonclassical CAH

CAH = congenital adrenal hyperplasia.

Severe nodulocystic acne occurring on the arms and upper back is a sign of hyperandrogenism. A common cause of
hyperandrogenism in young women, particularty in conjunction with obesity, is polycystic ovary syndrome (PCOS). In
addition to either clinical (eg, severe acne, hirsutism, androgenic alopecia) or laboratory (eg, elevated serum testosterone)
evidence of hyperandrogenism, the diagnosis of PC08 requires a history of irregular menses indicating anovulation and/or
polycystic ovaries on ultrasound. When 2 of these 3 criteria are met and other causes of hyperandrogenism (eg,
Cushing syndrome, nonclassical congenital adrenal hyperplasia) are excluded, the diagnosis of P008 is established.

(Choice A) Autoimmune hypothyroidism is linked to family history and may cause irregular menses and weight gain.
Typical skin findings are coarse hair, skin discoloration, and, occasionally, alopecia, not nodulocystic acne.

39 Female Reproductive System a Breast


V Polycystic ovary disease Ea Feedback
0
End Black
hem 35 one I ‘7 Mark ® it 3 I
QUESHO" Id: 12393 Tutorial Lab Values Notes Calculator Reverse Color

Severe nodulocystic acne occurring on the arms and upper back is a sign of hyperandrogenism. Acommon cause of
hyperandrogenism in young women, particularly in conjunction with obesity, is polycystic ovary syndrome (PCOS). In
addition to either clinical (eg, severe acne, hirsutism, androgenic alopecia) or laboratory (eg, elevated serum testosterone)
evidence of hyperandrogenism, the diagnosis of P008 requires a history of irregular menses indicating anovulation and/or
polycystic ovaries on ultrasound. When 2 of these 3 criteria are met and other causes of hyperandrogenism (eg,
Cushing syndrome, nonclassical congenital adrenal hyperplasia) are excluded, the diagnosis of PC08 is established.

(Choice A) Autoimmune hypothyroidism is linked to family history and may cause irregular menses and weight gain.
Typical skin findings are coarse hair, skin discoloration, and, occasionally, alopecia, not nodulocystic acne.

(Choices B and C) Patients Willi eating disorders do not typically have signs of hyperandrogenism. In addition, there is no
established causal relationship between certain foods and acne.

(Choice D) Intravenous drug use may cause localized skin infections (eg, folliculitis) but not diffuse nodulocystic acne.

(Choice F) The number of lifetime sexual partners is used to assess the risk for cervical dysplasia and sexually transmitted
infections; it is not relevant in the dilferential diagnosis of acne.

Educational objective:
Severe nodulocystic acne may be the initial sign of hyperandrogenism in patients with polycystic ovary syndrome. The
diagnosis is based on the presence of 2 of 3 manifestations: clinical or laboratory evidence of hyperandrogenism, menstrual
irregularities, and polycystic ovaries on ultrasound.

References

- Genetic, hormonal and metabolic aspects of PCOS: an update.

39 Female Reproductive System 8. Breast


Y Polycystic ovary disease
Q
0
Feedback End Black
- 39 Female Reproductive Sys‘em 8. Breast % O
_ 40 V Polycysfic ovary disease Feedback End Black
hem as one I ‘7 Mark ® it 3 I
QUESHO" Id: 3339 Tutorial Lab Values Notes Calculator Reverse Color

A32—yearrold woman, gravida 4 para 4, comes to the office for a routine checkup 6 weeks after an uncomplicated vaginal
delivery of a healthy boy. The patient has no concerns and would like to discuss contraception options. She heard from a
friend that some options can make losing weight challenging. Prior to this pregnancy, the patient had a history of
increasingly heavy menses and was found to be mildly anemic. Iron was prescribed, but she often forgot to take it; she
oflen forgot her prenatal vitamins too. The patient is breastfeeding exclusively. Pelvic examination shows a small, mobile
uterus with no abnormalities. Breast examination shows no masses, tenderness, or nipple injury. Which of the following is
the preferred method of contraception for this patient?

O A. Combined estrogen—progestin oral contraceptives [3%]

O B. Copper intrauterine device [24%]

V0 C. Levonorgestrel-containing intrauterine device [57%]

O D. Medroxyprogesterone injection [6%]

O E. No contraception needed while breastfeeding [7%]

Omitted
cm III 57% 28econds _ 04/18/2020
C° “"5”” Answered correctly Tlme Spent E Last Updated

Explanation

Levonorgestrel IUD

Female Reproductive System 8. Breast


Contraception
Q
0
Feedback End Black
Question Id: 3339
IVMark <
Previous
D
Next
Q)
Tutorial
i!
Lab Values
3
Notes Calculator
'
Reverse Color Text Zoom
$3
Explanation

Levonorgestrel IUD

Strings

Thickens cervical mucus (blocks spenh entry)


Thins uterine lining (decreases menstrual bleeding)
IUD = Intrauterine device.
ouwma

The IevonorgestreI-containing intrauterine device (IUD) is a long-acting, reversible contraceptive that prevents
pregnancy by releasing levonorgestrel (a progestin), which creates a physical barrier by thickening cervical mucus and
impairing implantation through decidualization of the endometrium‘ It has an efficacy of >99% and is approved for use in

Female Reproductive System a Breast


contraception Ea Feedback
0
End Block
hem as our) . ‘7 Mark < D C?) it 5- 9
Question I“: 3339 Previous Next Tutorial Lab Values Notes Calculator Reverse Color
The levonorgestrel-containing intrauterine device (IUD) is a long-acting, reversible contraceptive that prevents
pregnancy by releasing levonorgestrel (a progestin), which creates a physical barrier by thickening cervical mucus and
impairing implantation through decidualization of the endometrium. It has an efficacy of >99% and is approved for use in
the United States for up to 5 years. A common side effect is amenorrhea, which can be used to improve anemia and
abnormal uterine bleeding. A small percentage of women experience systemic side effects (eg, mood changes, breast
tenderness, headaches). Weight gain is not a side effect.

(Choice A) Combined oral contraceptives can be used for contraception in patients >6 weeks postpartum while
breastfeeding. They are an effective contraceptive method that can improve hypennenorrhea but should be reserved for
patients who reliably take daily medications.

(Choice B) The copper IUD, another long—acting, reversible contraceptive option, is approved in the United States for up to
10 years The copper IUD can cause heavy menstnial bleeding and should not be placed in women with hypermenon'hea
or anemia as it may exacerbate these conditions.

(Choice D) Medroxyprogesterone is an option for long—tenn contraception, with patients receiving injections every 3
months. Because weight gain is a common side effect, medroxyprogesterone is not optimal for this patient, who is
concerned about difficulties with postpartum weight loss.

(Choice E) Lactation can cause anovulation and therefore some degree of contraception as high prolactin levels inhibit the
release of gonadolropin—releasing hormone from the hypothalamus. Lactation, however, is not considered a reliable form of
birth control because ovulation can resume while a mother is still breastfeeding.

Educational objective:
The levonorgestrel intrauterine device (IUD) provides highly efficacious, long—acting, reversible contraception that works by
thickening cervical mucus and impairing implantation. Progestin—containing IUDs have the added benefit of amenorrhea
and minimal systemic side effects.

39 Female Reproductive System a Breast


V Contraception
Q
0
Feedback End Block
hem 37 one I ‘7 Mark ® it Q I
QUESHO" Id: 15140 Tutorial Lab Values Notes Calculator Reverse Color

A 16—yeareold girl comes to the office for a routine health maintenance examination. The patient is in high school, is
performing well academically, and plays on the tennis team. She has no concems today. On review of systems, the patient
has not reached menarche, but she says that her mother did not start menstruating until this age. She recently started
wearing contact lenses for myopia and uses an over—the—counter cream for acne. Height is at the 70th percentile and
weight is at the 25th percentile for age. Blood pressure is 120l70 mm Hg and pulse is 60/min. Breast development is
sexual maturity rating (Tanner stage) III. Pelvic examination shows sexual maturity rating (Tanner stage) III pubic hair
development and normal external genitalia. Speculum examination reveals a well-rugated vagina but no cervix. FSH levels
are within the normal pubertal range. Karyotype is 46, XX. Pelvic ultrasound reveals an absent uterus. Which of the
following is the best next step in management of this patient?

O A. Dihydrotestosterone and testosterone level [30%]

O B. Echocardiogram [5%]

O C. Hymenal incision [1%]

O D. MRI of the brain [3%]

«Q E. Renal ultrasound [53%]


O F. TSH and prolactin level [5%]

Omitted
C em I" 53% 1 Second — 03/15/2020
'
E°" “"5”” Answered correctly Time Spent E Last Updated

39 Female Reproductive System 8. Breast


Amencrrhea
Q
0
Feedback End Black
hem 37 am
Question Id: 15140
I P, Mark <
Previous
D
Next
Q)
Tutorial
ii
Lab Values
3
Notes Calculator
'
Reverse Color Text Zoom
$3
Explanation

Urogenital development

Kidney
Parameeonephric
(mullerian) duct

Urogenital
sinus
\ Uterus
Meeonephric
(wolflian) duct Ureter
Bladder
Lower %
bud of vagina

Q UWofld

This patient has an absent uterus and cervix with otherwise normal secondary sexual characteristics (eg, breasts,
external genitalia) consistent with miillerian agenesis (ie, Mayer—Rokitansky—Kiister—Hauser syndrome), abnormal mullen’an
duct development. The ovaries and external genitalia develop independently of the mullerian duct system; therefore,
patients have normal FSH levels (ie, normal ovarian function), normal external genitalia, and the lower 2/3 of the vagina (ie,
blind vaginal pouch).

The internal genitalia derive from structures of the intermediate mesoderm, which develops into the paramesonephric (ie,

dback End Block


Item 37 or 40 - ‘7 Mark < D Q) i] g- 9
Question Id: 15140 Previous Next Tutorial Lab Values Notes Calculator Reverse Color
This patient has an absent uterus and cervix with otherwise normal secondary sexual characteristics (eg, breasts,
external genitalia) consistent with mtillerian agenesis (ie, Mayer—Rokitansky—Kiister—Hauser syndrome), abnormal mullerian
duct development. The ovaries and external genitalia develop independently of the mullerian duct system; therefore,
patients have normal FSH levels (ie, normal ovarian function), normal external genitalia, and the lower 2/3 of the vagina (ie,
blind vaginal pouch).

The internal genitalia derive from structures of the intermediate mesoderm, which develops into the paramesonephric (ie,
mullerian) and mesonephric (ie, wolffian) ducts. The paramesonephric duct forms the uterus, fallopian tubes, cervix, and
upper 1/3 of the vagina, and the mesonephric duct forms the primitive kidney. Because of their common embryologic
source and synchronous development in the first trimester, internal genital anomalies are often concurrent with renal
abnormalities. Therefore, women with mullerian agenesis should undergo evaluation of the renal tract (ie, ureters, kidney)
with a renal ultrasound. Common abnormalities include unilateral renal agenesis, pelvic kidneys, and duplications of the
collecting system.

(Choice A) Testosterone and dihydrotestosterone levels are used to diagnose 5—alpha—reductase deficiency and androgen
insensitivity syndrome (AIS). In 5—alpha—reductase deficiency, genotypic males (46,XY) appear phenotypically female until
puberty; elevated testosterone levels at puberty then cause virilization (eg, clitoromegaly) and lack of breast development,
which are not seen in this patient. In AIS, genotypic males appear phenotypically female through puberty and have breast
development. In contrast to this patient, those with AIS have nolminimal acne or pubic hair development due to an
abnormal androgen receptor.

(Choice B) Echocardiogram is performed in patients with Turner syndrome to evaluate for cardiac malformations (e9,
bicuspid aortic valve, aortic coarctation). This patient has a normal karyotype and FSH level (ie, normal ovarian function),
making Turner syndrome unlikely.

(Choice 0) Hymenal incision is performed in patients with imperforate hymen, which causes primary amenorrhea. Patients
have a uterus and a blue—tinged vaginal mass due to accumulated menstrual blood (ie, hematocolpos).

39 Female Reproductive System a Breast


Amenormea
Q
0
Feedback End Block
them 37 M40 - < D» '
Question Id: 15140 Previous Next Tutorial Lab Values Notes Calculator Reverse Color
puberty; elevated testosterone levels at puberty then cause virilization (eg, clitoromegaly) and lack of breast development,
which are not seen in this patient. In AIS, genotypic males appear phenotypically female through puberty and have breast
development. In contrast to this patient, those with AIS have no/minimal acne or pubic hair development due to an
abnormal androgen receptor.

(Choice B) Echocardiogram is performed in patients with Turner syndrome to evaluate for cardiac malformations (eg,
bicuspid aortic valve, aortic coarctation). This patient has a normal karyotype and FSH level (ie, normal ovarian function),
making Turner syndrome unlikely.

(Choice 6) Hymenal incision is performed in patients with imperforate hymen, which causes primary amenorrhea. Patients
have a uterus and a blue-tinged vaginal mass due to accumulated menstmal blood (ie, hematocolpos).

(Choices D and F) TSH and prolactin levels are ordered in women with primary amenormea who have a uterus and
Iowlnonnal FSH. MRI of the brain is indicated in patients with a lowlnormal FSH, high prolactin, or visual field defects (eg,
hemianopsia) to evaluate hypothalamic and pituitary causes (eg, sellar mass).

Educational objective:
Mullen‘an agenesis causes primary amenonhea due to the failed development of the uterus, cervix, and upper 1/3 of the
vagina. Urogenital development is from a common embryologic source; therefore, renal malformations are common and
patients require evaluation with a renal ultrasound.

References

- ACOG Committee opinion: no. 728: M&#252;llerian agenesis: diagnosis, management, and treatment.

- Mayer-Rokitansky—K&#252,ster—Hauser syndrome: complications, diagnosis and possible treatment options: a review.

39 Female Reproductive System 8. Breast


Amenorrhea Ea Feedback
0
End Block
. 9 A
10 —
Lab Values Nata Calulator Revels-e Color
puberty; elevated testosterone levels at puberty then cause virilization (eg, clitoromegaly) and lack of breast development
Exhibit Display

"M

Q Zoom In Q Zoom Out 9 Reset 517 Add To Flash Card

- 39 Female Reproductive Sys‘em 8. Breast % O


. 40 v Amenarrhea Feedback End Black
Item as one I ‘7 Mark ® it 3 I
QUESHO" Id: 3933 Tutorial Lab Values Notes Calculator Reverse Color

A 62—yearrold nulligravid woman comes to the office for follow—up after right adnexal enlargement was found on routine
pelvic examination 2 weeks ago. Pelvic ultrasound reveals a 5-cm right ovarian cyst Menopause occurred at age 52, and
the patient has had no postmenopausal spotting or bleeding. She had an abnormal Papanicolaou test in her 205 that
subsequently tested as normal, and her medical history is otherwise unremarkable. The patient has never had any
surgeries. There is no family history of ovarian or breast cancer. Her most recent mammogram 2 months ago showed no
abnormalities. Which of the following is the most appropriate initial course of action for this patient?

O A. Endometn'al biopsy [3%]

O B. Laparoscopy [26%]

O C. Needle aspiration for cytology [25%]

O D. Papanicolaou test [4%]

v0 E. Serum CA—125 level [40%]

Omitted
Ill 40% 1 Second _ 03/26r2020
Ewe“ am” Answered correctly Tlme Spent E Last Updated

Explanation

Ovarian cancer risk increases with age, particularly after menopause, and any ovarian mass in a postmenopausal patient
requires investigation. Risk factors for this malignancy include age, use of fertility drugs, uninterrupted ovulation (eg,

Female Reproductive System a Breast Q 0


ovarian cancer Feedback End Block
hem 38 am I ‘7 Mark < D Q) i g '
Question Id: 3903 Previous Next Tutorial Lab Values Notes Calculator Reverse Color

Explanation

Ovarian cancer risk increases with age, particularty after menopause, and any ovarian mass in a postmenopausal patient
requires investigation. Risk factors for this malignancy include age, use of fertility drugs, uninterrupted ovulation (eg,
nulligravidity), and BRCA mutation. Cancer antigen 125 (CA-125) is a biomarker for epithelial ovarian cancer.
Elevations are also caused by common gynecologic conditions (eg, leiomyomata, endometn'osis) that are more likely
present in premenopausal patients; therefore, the specificity of CA—125 levels is much greater in postmenopausal women.

CA-125 levels are measured in conjunction with pelvic ultrasonography findings to categorize an ovarian mass as likely
malignant or benign. This postmenopausal patient has no medical conditions that may cause a false elevation of CA—125,
so measurement is the best next step to investigate the risk of malignancy for this ovarian cyst, CA—125 levels can also be
used to monitor for recurrence of a proven malignancy after treatment.

In a postmenopausal patient, an elevated CA-125 level in the context of any adnexal mass is highly suspicious for
malignancy. If the adnexal mass has no malignant features on ultrasound (eg, small size, simple cyst) and the CA—125 level
is normal, the patient can be observed with penodic ultrasound, but observation without further workup is inappropriate in a
postmenopausal patient. If there are suspicious features on ultrasound (eg, large mass, solid components, septations)
and/or if the CA—125 level is elevated, the patient should undergo further imaging (eg, MRI, CT scan) to assess the extent of
disease. Knowledge of the presence of metastatic implants from imaging is important to guide surgical exploration (Choice
B), which is recommended if malignancy is suspected based on clinical, CA<125, and imaging findings.

(Choice A) Endometrial biopsy is performed to investigate for endometrial carcinoma and is indicated in patients with
postmenopausal bleeding, abnormal uterine bleeding over age 45, or thickened endometrial stripe with an ovarian mass.
These findings are absent in this patient.

(Choice C) Needle aspiration is contraindicated in postmenopausal women with an adnexal mass due to risk of spreading

39 Female Reproductive System a Breast


Ovarian cancer
Q
0
Feedback End Block
them 38 M40 < D. '
. 10 Question Id: 8903 Previous Next Tutorial Lab Values Calculator Reverse Color
11 postmenopausal patient. If there are suspicious features on ultrasound (eg, large mass, solid components, septations)
- 12
and/or if the CA—125 level is elevated, the patient should undergo further imaging (eg, MRI, CT scan) to assess the extent of
13
disease Knowledge of the presence of metastatic implants from imaging is important to guide surgical exploration (Choice
. 14
B), which is recommended if malignancy is suspected based on clinical, CA—125, and imaging findings.
15
. 16 (Choice A) Endometrial biopsy is performed to investigate for endometrial carcinoma and is indicated in patients with
17 postmenopausal bleeding, abnormal uterine bleeding over age 45, or thickened endometrial stripe with an ovarian mass.
u 18
These findings are absent in this patient.
19
u 20 (Choice 6) Needle aspiration is contraindicated in postmenopausal women with an adnexal mass due to risk of spreading
21 potentially malignant cells should the mass prove cancerous.
. 22
(Choice D) Papanicolaou testing is a screen for cervical cancer and is not diagnostic of ovarian cancer. A distant history of
23
an abnormal test does not increase ovarian cancer risk, and this test is not indicated in a workup of an adnexal mass.
. 24

25 Educational objective:
. 26
Ovarian cancer risk increases alter menopause, and an ovan'an mass in a postmenopausal patient is highly conceming for
27
malignancy. Investigation by pelvic ultrasonography and CA—125 measurement is necessary. Even if the mass has no
. 28
malignant features on ultrasound, an elevated CA—125 level is concerning and requires further imaging and possible
29
. 30
surgical exploration.

31
. 32 References
33
. 34
- The value of ultrasound monitoring of adnexal masses for early detection of ovarian cancer.

35 - Adnexal masses: characterization and imaging strategies.


36
. 37

. 39

40
Female Reproductive System 8. Breast
Ovarian cancer Ea Feedback
0
End Block
hem as one I ‘7 Mark ® it Q I
QUESHO" Id: 13355 Tutorial Lab Values Notes Calculator Reverse Color

A21—yearrold woman is brought to the emergency department by her roommate. During a first date a week ago, the patient
blacked out after a single glass of wine. She woke up naked in her date's apartment and could not remember how she got
there. The patient has been anxious and embarrassed about the event and reluctantly told her roommate what happened
after she was found crying in the bathroom today. She does not want to involve the police because she wants to "move
on." The patient's last menstrual period was 3 weeks ago. Vital signs are normal and physical examination is
unremarkable. Urine pregnancy testing is negative. The patient is prescribed emergency contraception and postexposure
prophylaxis for sexually transmitted infections. Which of the following is the best next step in management of this patient?

O A. Discuss date rape prevention [5%]

O B. Notify the police [12%]

O C. Perform urine toxicology testing [13%]

O D. Provide reassurance and follow—up in 2 weeks [10%]

«O E. Recommend psychosocial counseling [57%]

O F. Start antidepressant [0%]

0m med
Correct answer
Ill 57% 1 Second — 02/0512020
Answered correctly Tlme Spent E Last Updated

Explanation

38

40
Female Reproductive System 8. Breast
Sexual assault Q
0
Feedback End Black
9 _ mamasouo lvmm < D G») it 3 9 £633
. 10 Question Id: 18366
Previous Next Tutorial Lab Values Notes Calculator Reverse Color Text Zoom
11
Explanation
- 12
13
. 14
15 Sexual assault
. 16 _ . Physical and forensic examination (eg, hair, semen)
Evaluatlon _
17 - Psychologrc assessment
. 1s
_ 19 . Mental health disorders (eg, PTSD)
, 20 Consequences - Unintended pregnancy
21 - Sexually transmitted infections
. 22
23 . Postexposure prophylaxis
_ 24 Management - Emergency contraception
25 - Psychosocial counseling

' 26 PTSD = posttraumatic stress disorder.


27
- 23 This patient is likely a victim of date rape. Rape is sexual assault involving unwanted vaginal, oral, or anal penetration
29 through force or lack of consent. Most cases occur between acquaintances and up to half involve intoxication with
' 30 alcohol or the use of date rape drugs (eg, flunitrazepam [Rohypnol]) that cause loss of consciousness and amnesia, as in
31 _ .
this patient.
. 32
33 Among adolescents and young adults, there are multiple cultural myths about date rape, including that it is not 'real" rape
- 34 and that "by agreeing to go home with a man, a woman is agreeing to sex.” Because of these myths, date rape victims are
35 more likely to feel partially responsible for their assault, which can lead to long—tenn psychologic consequences ranging
36 from decreased appetite and sleep disturbances to posttraumatic stress disorder and suicidality.
37
38 Therefore, management includes both medical care (eg. postexposure prophylaxis, emergency contraception) and v

40 v Sexual assault Feedback End Black


9 _ hem as one - \7 Mark ® it g. I
. 10 Question Id: 18356
Tutorial Lab Values Notes Calculator Reverse Color
11 .
12 This patient is likely a victim of date rape. Rape is sexual assault involving unwanted vaginal, oral, or anal penetration
13 through force or lack of consent. Most cases occur between acquaintances and up to half involve intoxication with
_ 14 alcohol or the use of date rape drugs (eg, flunitrazepam [Rohypnol]) that cause loss of consciousness and amnesia, as in
15 this patient.

' 16 Among adolescents and young adults, there are multiple cultural myths about date rape, including that it is not 'real' rape
i; and that "by agreeing to go home with a man, a woman is agreeing to sex." Because of these myths, date rape victims are
19 more likely to feel partially responsible for their assault, which can lead to long—term psychologic consequences ranging
. 20 from decreased appetite and sleep disturbances to postlraumatjc stress disorder and suicidality,

21 Therefore, management includes both medical care (eg, postexposure prophylaxis, emergency contraception) and
' 22 psychosocial counseling, which should take place as soon alter the assault as possible (Choice D). Patients should be
23
24 reassured that they were not responsible and be provided multiple forms of support (eg, support group referral, individual
25 therapy).

' 2‘ (Choice A) Date rape prevention may be discussed at the annual well—woman visit, but this advice is insensitive
27 immediately after an assault and places blame on the patient.
. 23
29 (Choice B) Although mandatory reporting occurs for child and elder abuse, sexual assault of an adult does not in most
. 30 junsdictions. In addition, notifying the police against this patient's wishes is inappropriate.
31
32 (Choice C) Urine toxicology can identify date rape drugs, but testing must be performed soon after an assault due to a
33 short half—life. Because this patient's assault was a week ago, testing is unlikely to be effective.

' 34 (Choice F) Antidepressant therapy may be indicated in patients who develop depression or post'lraumatic stress disorder
35 due to sexual assault; however, additional psychosocial evaluation is required in this patient before starting medical therapy.
36
37 Educational objective:
38 \fictims of sexual assault 9-. date race are at risk for Iono—term as cholooic conse-uences ea, osttraumatic stress v
Female Reproductive System a Breast Q 0
Sexual assault Feedback End Black
9 _ imam as one . \7 Mark ® 5‘ g. I
' 10 Question I“: ”356 Tutorial Lab Values Notes Calculator Reverse Color
11 . , . . , . , . . . . .
. 12 psychosocial counseling, which should take place as soon after the assault as possible (Choice D). Patients should be
13 reassured that they were not responsible and be provided multiple forms of support (eg, support group referral, individual
. 14 therapy).
15
16 (Choice A) Date rape prevention may be discussed at the annual well—woman visit, but this advice is insensitive
17 immediately after an assault and places blame on the patient.

' 13 (Choice B) Although mandatory reporting occurs for child and elder abuse, sexual assault of an adult does not in most
19 junsdictions. In addition, notifying the police against this patjent’s wishes is inappropriate.
. 20
21 (Choice 8) Urine toxicology can identify date rape drugs, but testing must be performed soon after an assault due to a
. 22 short half-life. Because this patient's assault was a week ago, testing is unlikely to be effective.
23
24 (Choice F) Antidepressant therapy may be indicated in patients who develop depression or postlraumatjc stress disorder
25 due to sexual assault; however, additional psychosocial evaluation is required in this patient before starting medical therapy.

' 26 Educational objective:


27 Victims of sexual assault (eg, date rape) are at risk for long—term psychologic consequences (eg, posttraumatic stress
. 23
29 disorder). Therefore, they should be offered psychosocial counseling in addition to medical care (eg, postexposure
_ 30 prophylaxis, emergency contraception).

31
. 32 References
33
_ 34 - Sexual assault of women.

35 - ACOG committee opinion no. 777 summary: sexual assault.


36
37
38
Female Reproductive System 8. Breast
Q B O
40 v Sexual assault Feedback End Black
9 _ Item400f40 'VMark ® it 3 I
. 10 Question Id: 16588
Tutorial Lab Values Notes Calculator Reverse Color
11

-
' 12 A27—yearrold woman comes to the office for advice regarding contraception. The patient has been using condoms
13 consistently but now wants something more reliable, She has regular monthly menses, typically with 6 days of bleeding that
' :2 requires wearing both a pad and a tampon simultaneously to prevent blood from getting on her clothing. Last year, the
16 patient's mother was diagnosed with a pulmonary embolus and found to be a heterozygous factor V Leiden carn'er. At that
_ 17 time, the patient also undenlvent screening and was also found to be a heterozygous factor V Leiden carrier. She has no
, 13 personal history of venous thromboembolism and no other medical conditions. The patient does not use tobacco, alcohol,
19 or illicit drugs. Blood pressure is 120/76 mm Hg and pulse is 70/min. BMI is 20 kglmz. Physical examination is normal.
- 20 Laboratory results are as follows:
21
_ 22 Complete blood count

23 Hemoglobin 10 g/dL
. 24
25 Hematocrit 30%

' :: Platelets 280,000/mm3


. 23 Leukocytes 4,100Imma
29
_ 30 Which of the following is the most appropriate contraceptive option for this patient?
. 31
_ 32 O A. Copper—containing intrauterine device [21%]

33 O B. Diaphragm and spermicide [4%]


. 34
35 O C. Estrogen-progestin oral contraceptives [3%]
. 36
37 D. Estrogen—progestin vaginal ring [0%]

Female Reproductive System a Breast Q 0


Contraception Feedback End Black
9 ‘ them 40 or 40 < D p
. 10 Question Id: 16588
Previous Next Tutorial Lab Values Notes Calculator Reverse Color Text Zoom
11
personal history of venous thromboembolism and no other medical conditions The patient does not use tobacco, alcohol,
- 12
or illicit drugs. Blood pressure is 120/76 mm Hg and pulse is "IO/min. BMI is 20 kg/m? Physical examination is normals
13
Laboratory results are as follows:
. 14

15 Complete blood count


. 16
17 Hemoglobin 10 gIdL

' 1" Hematocrit 30%


19
. zo Platelets 280,0001mm3
21
Leukocytes 4,1(10/mm“I
. 22

23 Which of the following is the most appropriate contraceptive option for this patient?
. 24
25 O A. Copper—containing intrauterine device [21%]
. 26
27 O B. Diaphragm and spermicide [4%]
. 23 O C. Estrogen-progestin oral contraceptives [3%]
. 29
_ 30 O D. Estrogen-progestin vaginal ring [0%]
31
VO E. Progestin—releasing intrauterine device [69%]
. 32
33
. 34
35 Omitted
C d I" 69% 7 Seconds — (HMO/2020

36 Eorre “SW” Answered correctly Time Spent E Last Updated

37
38 v

39 Female Reproductive System 8. Breast Q 0


n contraception Feedback End Block
-
IVMark < D Q) ii 3 ' €533
' 10 “"95“" I“: 16533 Previous Next Tutorial Lab Values Notes Calculator Reverse Color Text Zoom
11
- 12
Explanation
13
. 14

1: Combined hormonal contraception contraindications


17 Absolute Relative
. 1s
19 - Migraines with aura
. 20 - Severe hypertension
, 21 - lschemic heart disease, stroke - Mild or medication—controlled hypertension
. 22 - Age 235 & smoking 215 cigarettes/day - Age 235 & smoking <15 cigarettes/day
23 - <3 weeks postpartum - Certain medications (eg, lamotrigine, rifampin)
' 24 - Thromboembolism - Inherited thrombophilia carrier (& family member with
25 - Thrombophilia (eg, factor V Leiden, APLS) thrombophilia plus thromboembolism)
' :: - Active breast cancer
23 - Active or severe liver disease

29 APLs = antiphospholipidantibody syndrome.


. 30
31 Contraceptive counseling should be patient—centered and take into consideration patient—specific factors (eg, medical
. 32 history), the risk/benefit profiles of various contraception methods, and the patient's future fertility desires. In this patient,
33 the most appropriate contraceptive option is the progestin-releasing intrauterine device (IUD) because it takes into
' 3‘ consideration this patient's:
. 35
_ 36 - increased risk of thromboembolism because she is a heterozygous factor V Leiden carrier who has a family
37 member with inherited thrombophilia and prior thromboembolism. Estrogen-containing contraceptives are avoided in
38 atients at increased risk ofthromboembolism due to estrooen—induced h Icercoaulabili Choices C and D , In '
Female Reproductive System a Breast Q 0
camraception Feedback End Black
iigu'IJ éB
g A

' 10 Question I“: 16558 Tutorial Lab Values


Notes Calculator Reverse Color Text Zoom
11 " —. IOIlIl—. II'IV‘IO‘ ‘

- 12
13 Contraceptive counseling should be patient—centered and take into consideration patient—specific factors (eg, medical
_ 14 history), the risk/benefit profiles of various contraception methods, and the patient's future fertility desires. In this patient,
15 the most appropriate contraceptive option is the progestin-releasing intrauterine device (IUD) because it takes into
. 16 consideration this patient‘s:

i; - increased risk of thromboembolism because she is a heterozygous factorV Leiden carrier who has a family
19 member with inherited thrombophilia and prior thromboembolism. Estrogen-containing contraceptives are avoided in
. 20 patients at increased risk of thromboembolism due to estrogen—induced hypercoagulabilily (Choices (2 and D). In
21 contrast, the progestin—releasing IUD does not affect thromboembolic risk.

' 22 - heavy menstrual bleeding associated with anemia, which will not improve or may potentially worsen with the use of
23
24 certain contraception methods such as the copper-containing IUD (Choice A). The progestin—releasing IUD causes
25 endometrial atrophy, which treats heavy vaginal bleeding and anemia.

' 25 - desire for future fertility, which is possible with IUD removal at any time the patient desires pregnancy.
. 27
_ 23 In addition, the progestin—releasing IUD is one of the most effective methods of pregnancy prevention, with an efficacy of
29 >99% and duration of use up to 5 years.

' 30 (Choice B) Adiaphragm with spermicide is a nonhorrnonal barrier method that does not affect thromboembolic risk.
31
32 However, it has poor contraceptive efficacy, making it an inferior option for this patient seeking reliable contraception.

33 Educational objective:
' 34 Estrogen-containing contraception methods are contraindicated in patients at increased risk for thromboembolism.
35 Progestin-only contraceptives (eg, IevonorgestreI-releasing intrauterine device) and the copper-containing intrauterine
' 2: device may be offered to these patients.

38 v
- 39 Female Reproductive System a Breast Q 0
n v Contraception Feedback End Block
$3
9
10
Item400lf40
4» 9‘ '
Quesfionldnfifilm Lab Value. Nola Calcdator Reveise Color Text Zoom
Previous Next
11 ‘ ‘ —- I O O 0 II—.

12 Exhibit Display
13
14
15
16
17
18
19
20
21
22
23
24
25
26
27
28
29
30
31
32
33 “Ileana cervical mucus (blocks spam wiry)
Thins mains lining (dam monstrud bleeding)
34
“93mm.
35 OW
36
37
a Zoom In Q Zoom Out 9 Reset 517 Add To Flash Card
38

“v common
39 Female Reproductive Sysiem 8. Breast
Ea Feedback
0
End Black
- 9 ‘ Item 40 M40 4 D 6‘) 5| 9‘ ' £633
' 10 Question Id: 16588 Pfevioug Next Lab Value. Nata Calcdator Reveise Color Text Zoom
_ 11 “ —- IOOOII—. II'IV‘II‘ A

' 12 Exhibit Display


' 13
. 14
Copper IUD
. 15
. 16
. 17
. 18
. 19
. 20
. 21
. 22
. 23
. 24
. 25
. 26
. 27
. 28
. 29
. 30
. 31
. 32
. 33 Cases endometrial inflammation
_ 34 (basic to sperm & ova)
Ill) 3 mm- m.
. 35 OM
' Z: a Zoom In Q Zoom Out 9 Reset % Add To Flash Card

. 38 '

“v common
- 39 Female Reproductive Sys‘em 8. Breast % O
Feedback End Black

You might also like